Neuro NBME Form 1 – Flashcards

Unlock all answers in this set

Unlock answers
question
Neuro 1 An 84-year-old man is evaluated for a 5-year history of a gradually worsening gait and a 2-year history of cognitive impairment and urinary incontinence. Twelve years ago, he sustained a closed head injury that caused a mild traumatic subarachnoid hemorrhage and a 5-hour loss of consciousness. Medications include zolpidem (when needed as a sleep aid) and a daily multivitamin. On physical examination, temperature is 36.2 °C (97.2 °F), blood pressure is 128/78 mm Hg, pulse rate is 76/min, respiration rate is 14/min, and BMI is 27. The patient's gait is slow and unsteady and is marked by small, shuffling steps. His level of alertness, speech, posture, arm swing, and muscle tone are all normal, and he has no tremor. He scores 24/30 on the Folstein Mini-Mental State Examination, losing one point in the orientation portion for incorrectly stating today's date, three points in the serial calculation portion, and two points in the recall portion. Results of a complete blood count, a basic metabolic panel, serum vitamin B12 measurement, thyroid function tests, and a urinalysis are normal. An MRI of the brain is shown . Which of the following is the most likely diagnosis? A Alzheimer dementia B Normal pressure hydrocephalus C Parkinson disease D Vascular dementia
answer
B The triad of gait apraxia, dementia, and urinary incontinence, especially when accompanied by enlarged ventricles, is suggestive of normal pressure hydrocephalus. This patient exhibits the classic triad of gait impairment (specifically, gait apraxia), dementia, and urinary incontinence that typifies the potentially reversible syndrome of normal pressure hydrocephalus (NPH). This triad of symptoms eventually occurs in most patients with dementia, and the diagnosis of NPH is often considered but much less often proved to be the correct diagnosis. In this patient, however, strong evidence supports a diagnosis of NPH, including the MRI evidence of ventriculomegaly. Although Alzheimer dementia (AD) is also associated with cognitive impairment and impaired gait, gait does not improve after removal of cerebrospinal fluid in AD as it does in NPH. AD is so common in elderly patients with cognitive impairment that excluding it as a cause can delay the diagnosis of NPH; this delay may help explain some of the eventual shunt failures that occur even in patients with well-diagnosed NPH. Therefore, recognizing reversible dementia syndromes as soon as possible is imperative because of the therapeutic opportunity these syndromes represent. The only symptom this patient has that is shared by patients with Parkinson disease is a shuffling gait. Otherwise, his presentation—normal posture, arm swing, and muscle tone and the absence of a tremor—is quite different. Likewise, this patient has no history of or symptoms suggesting stroke or vascular disease, such as sudden onset of neurologic signs or symptoms, which makes vascular dementia unlikely. Although coincident vascular, Alzheimer-type, and Parkinson-type pathology is a common finding in autopsy studies, even in neurologically unimpaired healthy elderly adults, this fact should not constitute the basis for a diagnosis in the setting of a classic reversible dementia syndrome.
question
Neuro 2 An 18-year-old male college student is evaluated for a single generalized tonic-clonic seizure that began when he was asleep in his dormitory and resolved uneventfully. He has no history of head trauma, meningitis, or prior seizure and no family history of epilepsy. He takes no medications. Results of physical examination, including a neurologic examination, are normal. Results of laboratory studies, including a complete blood count, a serum electrolyte panel, and a urine toxicology screen, are also normal. An MRI of the brain and an electroencephalogram show no abnormalities. Which of the following is the most appropriate management of this patient's seizure? A Initiate no drug therapy at this time B Initiate therapy with carbamazepine C Initiate therapy with lamotrigine D Initiate therapy with valproic acid E Refer for epilepsy surgery evaluation
answer
A Unless special circumstances exist, drug therapy is generally not started in patients with a single unprovoked seizure. Drug therapy should not be initiated in this patient at this time. After a single unprovoked seizure, the risk of recurrence in the subsequent 2 years has been reported to be 30% to 40%. The risk of recurrence is greatest in patients with status epilepticus on presentation, with an identifiable underlying neurologic cause, or with abnormal results on an electroencephalogram (EEG). Patients with a partial seizure who are age 65 years or older or who have a family history of epilepsy may also be in a higher-risk category. The appropriate recommendation for this young patient, who has experienced a single idiopathic seizure but has no personal or family history of epilepsy, no identified neurologic cause of his seizure, and normal results on an EEG, is that no medication be started. As with all medical treatment recommendations, patient preference must be taken into account, and some patients in the low-risk group may elect to start therapy after a single seizure, particularly if they have a high-risk occupation. If a second seizure occurs in the future, the recurrence risk is greater than 60%, and antiepileptic medical therapy should be recommended at that time. Of note, driver's license privileges are restricted in every state in the United States for persons who have experienced a seizure. Specific restrictions vary by state, with typical requirements of a seizure-free period of 3 to 12 months in order to again operate a motor vehicle; a few states make exceptions for a single seizure. Reinstatement of driving privileges depends on demonstrating freedom from seizures for the specified period and there being a reasonable expectation of future seizure control. Initiation of antiepileptic medication is not required by law. Epilepsy surgery is reserved for patients who have disabling seizures that cannot be controlled with medication. Such surgery is not indicated for this patient, who had a single seizure that resolved uneventfully.
question
Neuro 3 A 36-year-old woman is evaluated in the office for a history of migraine, with and without aura, since age 16 years. She has an average of three attacks each month and consistently experiences an attack 2 days prior to menstruation; this headache is more difficult to treat than those not associated with menstruation. Although she typically obtains pain relief within 2 hours of taking sumatriptan, the headache recurs within 24 hours after each dose during the period of menstrual flow. Sumatriptan, orally as needed, is her only medication. Results of physical examination are unremarkable. Which of the following is the most appropriate perimenstrual treatment for this patient's headaches? A Estrogen-progestin contraceptive pill B Mefenamic acid C Sumatriptan plus naproxen, orally D Sumatriptan, subcutaneously E Topiramate
answer
B Evidence supports the use of mefenamic acid for perimenstrual prophylaxis of menstrually related migraines, with treatment starting 2 days prior to the onset of flow or 1 day prior to the expected onset of the headache and continuing for the duration of menstruation. This patient should be treated with mefenamic acid. She has migraine with aura, migraine without aura, and menstrually related migraine. Her menstrually related headaches are less responsive to acute therapy than are the non-menstrually related attacks, and headache recurs daily throughout menses. The best management for this patient is, therefore, the perimenstrual use of a prophylactic agent. There is evidence that supports the use of mefenamic acid for perimenstrual prophylaxis, with treatment starting 2 days prior to the onset of flow or 1 day prior to the expected onset of the headache and continuing for the duration of menstruation. In this patient, that would mean beginning 3 days before the onset of menstrual flow and continuing throughout menstruation. The use of combined oral contraceptive therapy (estrogen plus progestin) is contraindicated in this woman because of her history of migraine with aura. Women with migraine with aura are at a two-fold increased risk of ischemic stroke, ischemic myocardial infarction, and venous thromboembolism. The risk of stroke is increased further, up to eight-fold, in women with migraine with aura who use combined oral contraceptive pills. No evidence supports the oral use of either sumatriptan plus naproxen sodium or topiramate for the perimenstrual prophylaxis of menstrually related migraine. Similarly, there is no evidence supporting the subcutaneous use of sumatriptan in this setting. In fact, the higher recurrence rate with the subcutaneous formulation may prove counterproductive.
question
Neuro 4 A 52-year-old woman is evaluated for a 2-year history of burning feet. Symptoms are constant and are worse at night. The patient is overweight and has a history of hypertension treated with lisinopril. There is no known family history of peripheral neuropathy. On physical examination, the patient is afebrile; blood pressure is 134/88 mm Hg, pulse rate is 66/min, respiration rate is 12/min, and BMI is 28. Neurologic examination shows diminished pinprick and temperature sensation on the dorsal and plantar surfaces of both feet. Cranial nerve examination and testing of manual muscle strength, deep tendon reflexes, proprioception, and coordination reveal no abnormalities. Laboratory studies show a fasting plasma glucose level of 102 mg/dL (5.7 mmol/L). Results of a complete blood count, vitamin B12 measurement, and serum protein electrophoresis are all normal. Electromyographic testing shows a mild reduction in the sensory nerve action potential in the legs, compatible with a mild peripheral neuropathy. An MRI of the lumbar spine is normal. Which of the following is the most appropriate next diagnostic test? A Cerebrospinal fluid examination B Genetic testing for Charcot-Marie-Tooth disease C Glucose tolerance test D Skin biopsy
answer
C A history of burning or lancinating distal extremity pain and examination findings showing only sensory loss suggest a small-fiber peripheral neuropathy, which is most frequently associated with diabetes mellitus and impaired glucose tolerance. A 2-hour glucose tolerance test to detect diabetes mellitus or impaired glucose tolerance is the most appropriate next diagnostic study for this patient. The patient's history of burning feet, in conjunction with neurologic examination findings showing only distal sensory loss (with normal reflexes and muscle strength), suggests a small-fiber peripheral neuropathy. The most common identifiable cause of small-fiber peripheral neuropathy is diabetes or impaired glucose tolerance. This patient's fasting plasma glucose level, which is just over the upper limit of normal, should prompt a 2-hour glucose tolerance test. This test is more sensitive and can detect patients with diabetes and a normal fasting plasma glucose level. Approximately 30% of patients with normal fasting plasma glucose levels will have an oral glucose tolerance test diagnostic for diabetes. A result of 140 mg/dL to 199 mg/dL (7.8 mmol/L to 11.0 mmol/L) from this test establishes a diagnosis of impaired glucose tolerance, and a result of 200 mg/dL (11.1 mg/dL) or greater is diagnostic of diabetes. An examination of cerebrospinal fluid (CSF) obtained on lumbar puncture is not indicated in this patient. CSF studies should be considered in patients with acute or rapidly progressive neuropathy or polyradiculoneuropathy and in patients with severe weakness, sensory loss, or absent deep tendon reflexes. In patients with Guillain-Barré syndrome or chronic inflammatory polyradiculoneuropathy, CSF examination typically shows a normal cell count with elevated CSF protein level (albuminocytologic dissociation). CSF testing in such patients also helps to exclude infectious disorders, such as West Nile virus, HIV, and polyradiculoneuropathy caused by cytomegalovirus, which can present similarly. Given the symptom of burning feet and the absence of high arches, hammertoes, or abnormalities on strength and reflex testing, Charcot-Marie-Tooth disease is not likely. Although Charcot-Marie-Tooth disease is a common cause of peripheral neuropathy, patients with this disorder typically do not have neuropathic pain, and many patients also do not have sensory symptoms, despite having sensory loss on examination. Many patients in whom Charcot-Marie-Tooth disease is ultimately diagnosed are unaware of a family history of peripheral neuropathy, so the lack of family history in this patient does not necessarily exclude the diagnosis. Genetic testing is commercially available for many of the different forms of this disease, but genetic testing should be considered only in those patients with a known family history or in those with long-standing neuropathic symptoms and high arches and hammertoes on examination. Other inherited causes of small-fiber peripheral neuropathy include the hereditary burning feet syndrome, hereditary amyloidosis, and the hereditary sensory autonomic neuropathies. A skin biopsy is not indicated in this patient, given that the electromyographic (EMG) study was abnormal and showed objective evidence of peripheral neuropathy. Abnormalities on EMG testing in patients with suspected small-fiber peripheral neuropathy are seen in only 25% to 30% of patients. When EMG studies are normal, skin biopsy or autonomic nervous system testing should be considered to establish the diagnosis of a small-fiber peripheral neuropathy. Skin testing for small-fiber peripheral neuropathy is commercially available and can show reduced numbers of nerve fibers in the epidermis. Because small-fiber nerves also make up the autonomic nervous system, autonomic nervous system testing can also establish the presence of a small-fiber peripheral neuropathy.
question
Neuro 5 A 41-year-old man is evaluated in the emergency department for a 3-day history of confusion and visual loss and a 10-day history of gradually increasing headache. Two weeks ago, he had a 3-day history of severe gastroenteritis with diarrhea, nausea, and vomiting. A review of medical records shows that he is a factor V Leiden heterozygote. On physical examination, vital signs are normal. The patient has papilledema, a very mild right pronator drift, right homonymous hemianopsia, and fluent aphasia. Results of laboratory studies and a CT scan of the head are normal. Which of the following is the most appropriate next diagnostic test in this patient? A Carotid ultrasonography B Electroencephalography C Lumbar puncture D Magnetic resonance venography
answer
D Cerebral venous sinus thrombosis should be considered in the differential diagnosis of any patient with headache, features of elevated intracranial pressure, progressive reduction in the level of consciousness, and seizures, especially if there is a known risk of hypercoagulability. This patient should undergo magnetic resonance venography. The most likely diagnosis, given his known hypercoagulability, likely dehydration, symptoms of mounting intracranial pressure, and eventual focal deficits, is cerebral venous sinus thrombosis with possible venous ischemia. Cerebral venous sinus thrombosis may present with signs and symptoms of intracranial hypertension, such as headache, papilledema, and visual problems; focal neurologic findings or seizures; and mental status changes, stupor, and coma. Major risk factors for cerebral venous sinus thrombosis in adults include conditions that predispose to spontaneous thromboses, such as inherited or acquired thrombophilia, pregnancy, oral contraceptive use, malignancy, sepsis, and head trauma. Magnetic resonance venography can readily detect obstruction of the venous sinuses by a thrombus and the damage to the brain caused by the resultant increased pressure and so is the most appropriate next diagnostic test. Of all the possible imaging modalities, magnetic resonance venography is the most sensitive test for detecting the thrombus and the occluded dural sinus or vein. Carotid ultrasonography assesses the extracranial vasculature but does not assist in the assessment of the intracranial vasculature. The patient's clinical presentation suggests elevated intracranial pressure from venous stasis. The intracranial venous sinuses need assessment, not the extracranial carotid arteries. Seizures are a recognized complication of cerebral venous sinus thrombosis. However, electroencephalography does not assist with the establishment of a preliminary diagnosis and should not be performed next. A lumbar puncture is contraindicated in this patient with evidence of elevated intracranial pressure.
question
Neuro 6 A 45-year-old woman is evaluated in the emergency department for a 2-day history of increased leg weakness, ataxia, fatigue, and urinary incontinence. She has no other systemic symptoms. The patient has a 20-year history of multiple sclerosis, which has followed a progressive course over the past 5 years. Her baseline neurologic status, recorded 1 month ago, includes moderate bilateral spastic leg weakness, bilateral extensor plantar responses, mild sensory loss, and ambulation with a cane. Her only medication is interferon beta-1a; her dosage has been stable for the past 3 years. On physical examination, temperature is 38.1 °C (100.6 °F), blood pressure is 110/70 mm Hg, pulse rate is 90/min, and respiration rate is 16/min. The patient is unable to ambulate because of severe leg weakness. There is no costovertebral angle tenderness. Results of general physical and neurologic examinations are otherwise normal, including tests of mental status, cranial nerve function, and upper extremity neurologic status. Results of a complete blood count show a leukocyte count of 11,000/µL (11 × 109/L). Her urine is cloudy and is positive for nitrites and leukocyte esterase on urinalysis. Results of serum electrolyte measurement, liver chemistry studies, and renal function tests are all normal. A radiograph of the chest reveals no abnormalities. Which of the following is the best drug therapy for this patient? A Baclofen B Ciprofloxacin C Methylprednisolone D Prednisone
answer
B Pseudorelapses in multiple sclerosis are the worsening of neurologic symptoms because of another cause, such as a systemic infection requiring antibiotic treatment or supportive care, and should be differentiated from true relapses, which may require corticosteroid treatment. Ciprofloxacin is the best drug therapy for this patient. In light of her new urinary incontinence, fever, and abnormal results on urinalysis, she most likely has a urinary tract infection, which is manifesting clinically as a worsening of her baseline neurologic deficits (leg weakness and ambulatory impairment). She is experiencing a pseudorelapse of her multiple sclerosis (MS)—a neurologic deterioration caused by a change in general health status or by factors in the external environment (such as heat). Pseudorelapses are usually caused by systemic infections or medications. Treatment of the underlying cause (in this patient, with the antibiotic agent ciprofloxacin), obtaining a urine culture to ensure the appropriate antibiotic choice, and supportive care with an antipyretic agent constitute the appropriate treatment. Pseudorelapses need to be differentiated from true MS relapses, which are not associated with systemic symptoms, such as fever. Patients with chronic, progressive forms of MS and moderate or severe baseline neurologic impairment are at high risk for pseudorelapses. Occult urinary tract infection is a common culprit. Symptomatic therapy with antipyretic and antispasticity drugs only is not appropriate for this patient because her primary infection would be left untreated. Additionally, antispasticity drugs, such as baclofen, are likely to worsen her leg weakness. Pseudorelapses are typically self-limited, and spontaneous neurologic improvement usually occurs during or shortly after successful treatment of the underlying infection. Therefore, inclusion of a corticosteroid at treatment onset is unnecessary and could interfere with the treatment of the infection. Because corticosteroids do not treat and may aggravate the primary infection, high-dose methylprednisolone or prednisone should never be given as the only therapy to patients with MS and a suspected urinary tract infection. Occasionally, a systemic infection triggers a new MS relapse; in patients with such a relapse, corticosteroids could be used to treat the neurologic impairment after cure of the infection and exclusion of other causes of deterioration have been documented.
question
Neuro 7 A 50-year-old woman is evaluated in the emergency department after having a witnessed generalized tonic-clonic seizure. Her husband reports hearing her fall and finding her on the kitchen floor convulsing. The event lasted 2 minutes and stopped spontaneously. The patient has never experienced a similar episode, has no personal history of head trauma or meningitis, and has no family history of a seizure disorder. She has a history of hypertension treated with hydrochlorothiazide. On examination, the patient is lethargic but cooperative. She is afebrile, with a blood pressure of 146/80 mm Hg and a pulse rate of 70/min. She has mild weakness of the left face and arm that resolves over the next 3 hours. Results of laboratory studies, including measurement of plasma glucose level, are normal. An MRI of the brain shows chronic small-vessel ischemic disease but no acute abnormalities. Which of the following findings predicts a greater risk of a future seizure in this patient? A Age B Hypertension C Presence of secondary generalization D Small-vessel ischemic changes on an MRI of the brain E Todd paralysis
answer
E After a single unprovoked seizure, a greater risk of recurrence is predicted if the event was a partial seizure or if the patient has Todd paralysis, status epilepticus on presentation, an age greater than 65 years, or abnormal findings on neurologic examination. The presence of Todd paralysis predicts a greater risk of future seizures in this patient. After a single unprovoked seizure, recurrence risk in the subsequent 2 years has been reported to be 30% to 40%. Many historical or examination findings predict a higher risk of recurrence, including presentation in status epilepticus, age greater than 65 years, known underlying neurologic disorder(s) or structural lesion(s), and Todd paralysis. Partial-onset seizures are also more likely to recur, perhaps because of the increased likelihood of there being an underlying causative structural lesion. Todd paralysis is a transient unilateral or focal weakness often seen after partial seizures, with or without secondary generalization. Todd paralysis or other focal abnormalities on the neurologic examination predict a greater risk of a future seizure. Patients with an electroencephalogram or head imaging scan showing a potentially epileptogenic lesion are also in the higher-risk category. Elderly patients have been shown to have a higher risk of recurrence after a first seizure compared with younger adults. This 50-year-old woman would not be in a higher risk category on the basis of age alone. Cerebrovascular disease, including prior stroke and intracranial hemorrhage, is a commonly identified cause of epilepsy in older adults. However, the presence of hypertension and other cerebrovascular risk factors does not predict a greater risk of future seizures. The presence or absence of secondary generalization does not affect the rate of future events and so is not relevant for determining this patient's risk. The relatively nonspecific finding of small-vessel ischemic disease on an MRI of the brain is not significantly correlated with seizure risk. More concerning vascular lesions include acute or chronic stroke or hemorrhagic lesions involving cortical structures.
question
Neuro 8 A 70-year-old woman is evaluated in the emergency department for a 6-week history of back pain that has become increasingly severe over the past 6 days and a 2-day history of progressive bilateral leg weakness. Two years ago, the patient was treated for breast cancer with lumpectomy and local radiation therapy. She has no other pertinent personal or family medical history and takes no medications. On examination, temperature is 37.1 °C (98.8 °F), blood pressure is 140/85 mm Hg, and pulse rate is 88/min. She had moderate spastic paraparesis and bilateral extensor plantar responses = Babinski. A T8 sensory level, impaired bilateral lower extremity proprioception, and moderate gait ataxia are also noted. Results of a complete blood count, erythrocyte sedimentation rate determination, and coagulation studies are all normal. An MRI of the spinal cord reveals an epidural mass causing destruction of the T6 vertebral body with spinal instability and moderately severe compression of the adjacent spinal cord. Which of the following is the most appropriate immediate treatment of this patient's spinal cord compression? A Decompressive surgery B Intravenous dexamethasone followed by decompressive surgery and chemotherapy C Intravenous dexamethasone followed by decompressive surgery and radiation therapy D Intravenous dexamethasone followed by radiation therapy and chemotherapy
answer
C Epidural spinal cord compression usually requires immediate therapy with intravenous corticosteroids followed by decompressive surgery and radiation therapy. This patient has a typical presentation of epidural spinal cord compression, most likely from recurrence of the patient's previously treated breast cancer. The best treatment is intravenous dexamethasone followed by decompressive surgery and radiation therapy. The corticosteroid infusion likely reduces the tumor-related mass effect and edema, surgery provides immediate physical decompression of the spinal cord and the opportunity to stabilize the spine (if necessary), and radiotherapy targets the residual macroscopic and microscopic tumor burden. Observational studies support the use of intravenous corticosteroids in all patients with epidural spinal cord compression. Surgery is also warranted because of the concern about spinal instability. Furthermore, a randomized, multicenter, controlled clinical trial of therapy for epidural spinal cord compression, which included patients with metastatic breast cancer, showed that the combination of decompressive surgery and radiotherapy was associated with better ambulatory outcome than either treatment alone. In some scenarios, such as early spinal cord compression by a very radiosensitive tumor (including lymphoma and myeloma), intravenous dexamethasone plus radiotherapy may be appropriate. However, patients with such tumors were not included in the previously mentioned trial. There is no role for chemotherapy in the immediate treatment of epidural spinal cord compression. Chemotherapy may be indicated as part of a comprehensive program to control metastatic disease but will not provide the urgent relief needed for this cancer emergency.
question
Neuro 9 An 88-year-old woman is evaluated in the emergency department 1 hour after the acute onset of language disturbance and right-sided weakness. The family members who witnessed the onset say that the symptoms progressed over a few minutes and that there were accompanying symptoms of nausea and vomiting; they describe the patient holding her head as if in pain. She has a 20-year history of hypertension but no other medical problems; her only medication is lisinopril. During the examination, the patient becomes increasingly difficult to arouse and vomits repeatedly. She is afebrile, blood pressure is 220/110 mm Hg, pulse rate is 110/min, and respiration rate is 20/min. There is no nuchal rigidity. Carotid upstrokes are normal; there are no bruits and no jugular venous distention. Other than tachycardia, the cardiopulmonary examination is unremarkable. Global aphasia and right hemiplegia are noted. On the basis of her preliminary clinical evaluation, which of the following is the most likely diagnosis? A Intracerebral hemorrhage B Ischemic stroke C Meningitis D Transient ischemic attack
answer
A The classic presentation of intracerebral hemorrhage is the sudden onset of a focal neurologic deficit with subsequent symptomatic progression over minutes to hours. This patient most likely has had an intracerebral hemorrhage. The classic presentation of an intracerebral hemorrhage is the sudden onset of a focal neurologic deficit with subsequent symptomatic progression over minutes to hours. Headache, vomiting, hypertension, and an impaired level of consciousness are its most common clinical accompaniments and help distinguish an intracerebral hemorrhage from an ischemic stroke. Intracranial hemorrhage, which accounts for 11% of stroke deaths, has symptoms similar to those of ischemic stroke at presentation; therefore, it cannot always be reliably distinguished from ischemic stroke by clinical criteria alone. The definitive diagnostic study is CT of the head or MRI of the brain. Imaging provides diagnostic and prognostic information. The volume of intracranial hemorrhage and level of consciousness are the two most powerful predictors of outcome in this setting. A severe hemispheric ischemic stroke can cause severe focal neurologic dysfunction but generally does not progress or worsen as rapidly as occurred with this patient. Meningitis, although sometimes fulminant, is rarely this acute in presentation. As with subarachnoid hemorrhage, it may be associated with nuchal rigidity. Encephalitis, but not meningitis, can cause cerebral dysfunction, such as aphasia. A transient ischemic attack is a brief episode of focal neurologic dysfunction, without progressive worsening, that usually lasts less than 30 minutes and so is unlikely to be the correct diagnosis in this patient.
question
Neuro 10 A 61-year-old man is evaluated in the office for a 6-month history of progressive weakness of the lower extremities. He says he has difficulty rising from a seated position and walking up stairs and also has episodes of dry eyes, dry mouth, and erectile dysfunction. The patient reports no ptosis, diplopia, dysphagia, or dyspnea. He has a 15-year history of hypertension and a 42-pack year smoking history. Family history is unremarkable. His only medication is hydrochlorothiazide. On physical examination, vital signs are normal. Manual muscle strength testing shows weakness in the proximal upper and lower limb muscles. Deep tendon reflexes are absent diffusely. Plantar responses are flexor. A sensory examination shows no abnormalities, and cranial nerve function is normal. Laboratory studies show normal serum levels of sodium, potassium, calcium, creatinine, glucose, and creatine kinase. Results of liver chemistry studies are also normal. Which of the following is the best diagnostic test for this patient? A Measurement of acetylcholine receptor antibody level B Measurement of parathyroid hormone level C Measurement of voltage-gated P/Q-type calcium channel antibody level D Muscle biopsy
answer
C The diagnosis of Lambert-Eaton myasthenic syndrome, a neuromuscular junction disorder that causes progressive proximal muscle weakness and areflexia, precedes the clinical recognition of cancer in up to 50% of patients. This patient most likely has Lambert-Eaton myasthenic syndrome, as suggested by his history of proximal upper and lower limb weakness, the presence of autonomic symptoms (dry eyes/mouth, erectile dysfunction), and the finding of absent deep tendon reflexes on examination. These are characteristic signs and symptoms of the syndrome. Lambert-Eaton myasthenic syndrome is a neuromuscular junction disorder caused by disordered calcium channel function on the presynaptic nerve terminal. In most patients with this disorder, antibodies to voltage-gated P/Q-type calcium channel receptors exist. Lambert-Eaton myasthenic syndrome is typically a paraneoplastic syndrome, caused by or associated with an underlying malignancy, particularly small cell lung cancer. The diagnosis of Lambert-Eaton myasthenic syndrome precedes the clinical diagnosis of cancer in up to 50% of affected patients; therefore, in patients with newly diagnosed Lambert-Eaton myasthenic syndrome, a thorough search for an underlying cancer should be performed. If no evidence of malignancy is found, these patients should be evaluated serially for occult malignancy. In addition to elevated levels of voltage-gated P/Q-type calcium channel antibodies, the diagnosis can be confirmed through electrodiagnostic studies, particularly repetitive nerve stimulation studies, which show an increase in the muscle action potential (increment) after brief exercise. Elevated levels of antibodies against acetylcholine receptors are present in 90% of patients with generalized myasthenia gravis. Myasthenia gravis is an autoimmune disorder that results in neuromuscular transmission failure, causing weakness of limb and cranial muscles. The diagnosis is confirmed through electrodiagnostic testing, including repetitive nerve stimulation studies and, in some patients, single-fiber electromyography. The presence of an elevated acetylcholine receptor antibody level may provide additional confirmatory evidence supporting the diagnosis of myasthenia gravis. In this patient, the absence of any bulbar signs or symptoms, such as ptosis, visual symptoms (blurred vision or diplopia), or dysphagia, in conjunction with absent deep tendon reflexes, would argue against myasthenia gravis. Hyperparathyroidism, either primary or secondary, can result in proximal limb weakness. The normal calcium level in this patient would argue against a significant parathyroid disorder. Additionally, absent deep tendon reflexes would not be expected in a patient with hyperparathyroidism. Measurement of the parathyroid hormone level is therefore not indicated. Muscle biopsy is not likely to offer any additional diagnostic information in this patient with normal serum creatine kinase levels. Muscle biopsy is indicated primarily in patients with suspected inflammatory myopathies, such as polymyositis, dermatomyositis, or inclusion body myositis, and in certain hereditary myopathic disorders. Symptom onset in the seventh decade argues against a hereditary myopathy, as does the normal creatine kinase level. Although serum creatine kinase levels can be normal in patients with inclusion body myositis, deep tendon reflexes are typically normal, and weakness is most prominent in quadriceps and deep finger flexor muscles.
question
Neuro 11 An 84-year-old man is evaluated for the gradual onset of progressive memory loss over the past 2 years. In the past 4 months, he has twice been unable to find his way home after going to the local supermarket; his wife now goes with him whenever he leaves the house. His wife also has assumed responsibility for the household finances after the patient overdrew their checking account for the third time because of subtraction errors in their checkbook. He has hypertension treated with hydrochlorothiazide and hypothyroidism treated with levothyroxine. His mother had onset of Alzheimer dementia at age 79 years and died at age 86 years. His only other medication is a daily multivitamin. On physical examination, temperature is 36.9 °C (98.4 °F), blood pressure is 130/80 mm Hg, pulse rate is 72/min, respiration rate is 14/min, and BMI is 25. His level of alertness, speech, and gait are normal. His score on the Folstein Mini-Mental State Examination is 24/30, including 0/3 on the recall portion. Results of laboratory studies, including a complete blood count, serum vitamin B12 measurement, thyroid function tests, and a basic metabolic panel, are normal. An unenhanced MRI of the brain shows no abnormalities. Which of the following is the most appropriate treatment at this time? A Donepezil B Memantine C Quetiapine D Sertraline E Discontinuation of all current medications
answer
A First-line pharmacotherapy for mild Alzheimer dementia is an acetylcholinesterase inhibitor. This patient should receive donepezil. The Folstein Mini-Mental State Examination (MMSE) discriminates well between the major stages of dementia used for prognosis and management purposes. The MMSE score range of 21 to 25 corresponds to mild dementia, 11 to 20 to moderate dementia, and 0 to 10 to severe dementia. This patient has Alzheimer dementia and is at a mild to moderate stage of impairment. The most appropriate medication with which to begin treatment is an acetylcholinesterase inhibitor of which there are currently three: donepezil, rivastigmine, and galantamine. Multiple large, prospective, randomized, double-blind, placebo-controlled studies have shown in patients with mild, moderate, or severe Alzheimer dementia the efficacy of donepezil (and its superiority to placebo) in the preservation of instrumental and functional activities of daily living and in the reduction of caregiver stress. Other studies have found that patients treated with donepezil have improved cognitive function compared with those treated with placebo. Donepezil was safe and well tolerated in this patient group. Memantine is also used to treat Alzheimer dementia, but only in patients with moderate to severe impairment. There is no evidence that memantine has any effect in earlier stages of Alzheimer dementia or that it alters the course of the disease. With a score of 24/30 on the MMSE, this patient has mild dementia, which makes memantine an inappropriate treatment. In patients with severe dementia, memantine can be used alone or added to an acetylcholinesterase inhibitor. Quetiapine is an antipsychotic drug, and sertraline is an antidepressant agent. Although both can be used in patients with Alzheimer dementia, their use is limited to treatment of behavioral symptoms of psychosis and depression, respectively, neither of which this patient has exhibited. However, if these medications are to be used in such patients, the risks must first be carefully weighed against the benefits. The U.S. Food and Drug Administration has reported that the use of second-generation antipsychotic medications (aripiprazole, olanzapine, quetiapine, and risperidone) in elderly patients with dementia is associated with increased mortality. Although it is important to consider the potential cognitive side effects of prescription (and nonprescription) medications, those taken by this patient are not associated with such effects. Therefore, there is no need to risk potential harm to this patient by discontinuing his blood pressure and thyroid medications.
question
Neuro 12 A 30-year-old woman is evaluated in the office for a 6-week history of severe left facial pain. She says the pain occurs multiple times each day, is confined to the left cheek and jaw, and is stabbing in quality. Each pain episode lasts only 2 seconds, but she may experience multiple consecutive episodes. The pain can be triggered by drinking cold fluid, chewing on the left side of the mouth, or touching the left cheek. Three years ago, she had intermittent paresthesias of the hands and feet, which led to a diagnosis of anxiety. She has been taking ibuprofen for pain control since her current symptoms began, but it has been ineffective. On physical examination, vital signs are normal. Pain is triggered during the examination by touching the left cheek. She has no long-tract pyramidal signs or pathologic reflexes. Facial sensation and strength are normal, as are findings of funduscopic examination. Which of the following is the most appropriate next step in management? A Administration of baclofen B CT of the head C Lumbar puncture D MRI of the brain E Nerve conduction studies of the extremities
answer
D Because trigeminal neuralgia usually presents after age 40 years, its diagnosis in a younger patient should prompt an evaluation for secondary causes, such as multiple sclerosis, posterior fossa tumors, and vascular or aneurysmal compression of the trigeminal nerve. This patient, whose history is most compatible with a diagnosis of trigeminal neuralgia, should have an MRI of the brain. In 90% of patients with trigeminal neuralgia, idiopathic disease presents after age 40 years. Trigeminal neuralgia in a patient this young should prompt evaluation for secondary causes, such as multiple sclerosis, posterior fossa tumors, and vascular or aneurysmal compression of the trigeminal nerve. In light of her history of transient paresthesias 3 years ago, multiple sclerosis is high among the diagnostic possibilities for this patient. An MRI can show the demyelination typical of multiple sclerosis. Baclofen, although sometimes used in this scenario, is not a first-line drug for treatment of trigeminal neuralgia and should not be initiated at this stage because there is insufficient evidence from randomized, controlled trials to show significant benefit from non-antiepileptic drugs in trigeminal neuralgia. CT of the head is not the imaging procedure of choice for this patient. The resultant scan will not show demyelinating lesions and may not detect posterior fossa or skull-based lesions involving the trigeminal nerve. Although the detection of abnormalities on cerebrospinal fluid examination can be useful in supporting a diagnosis of multiple sclerosis, brain-imaging procedures should precede lumbar puncture, especially as long as other structural intracranial lesions are still part of the differential diagnosis. Once such lesions are excluded by the results of imaging studies, lumbar puncture may be appropriate to perform. Nerve conduction studies of the extremities are inappropriate for this patient because the paresthesias have been transient, the neurologic examination discloses no evidence of a peripheral neuropathy, and the structural and demyelinating causes of trigeminal neuralgia are best excluded with an MRI of the brain.
question
Neuro 13 A 62-year-old woman is evaluated in the emergency department for a 2-week history of progressive shortness of breath, which has culminated over the past 3 days in a change in the sound of her speech and occasional swallowing difficulties. She says that approximately 8 weeks ago, she began to have blurred vision late in the day, particularly when driving home from work, and occasional weakness in the upper and lower limbs. Three weeks ago, she had a urinary tract infection treated with ciprofloxacin. The patient takes no other medications On physical examination, blood pressure is 118/56 mm Hg, pulse rate is 68/min, respiration rate is 22/min, and arterial oxygen saturation on ambient air is 98%. Her lungs are clear to auscultation. She has moderate bilateral proximal weakness in the upper and lower limbs, with mild weakness distally. Deep tendon reflexes are normal, and there is no appendicular or truncal ataxia. She has fluctuating ptosis and diminished facial strength. Speech is nasal and slurred; her language is normal. Which of the following is the best treatment option for this patient? A Plasma exchange B Prednisone C Pyridostigmine D Repeat ciprofloxacin therapy
answer
A Myasthenic crisis, a potentially life-threatening neurologic emergency characterized by muscle weakness severe enough to necessitate intubation, typically requires plasma exchange therapy. This patient has a rapidly progressive neurologic disorder with weakness of limb, bulbar, and respiratory muscles and should undergo plasma exchange. Given the absence of sensory loss and normal deep tendon reflexes, myasthenia gravis is the most likely diagnosis. Because of the rapid progression of the disease and the involvement of respiratory and bulbar muscles, the patient should be admitted to the hospital for management of a myasthenic crisis. Myasthenic crisis is a potentially life-threatening neurologic emergency characterized by weakness that is severe enough to necessitate intubation. Electromyographic studies, including repetitive stimulation of motor nerves, are indicated to establish a diagnosis of myasthenia gravis. Plasma exchange is the treatment of choice in patients with myasthenic crisis. Myasthenia gravis is an autoimmune disorder that results from antibody-mediated attacks on the postsynaptic neuromuscular junction. This process may impair neuromuscular transmission by functional blockade of the acetylcholine receptor, by accelerating the degradation of acetylcholine receptors, and by causing damage to the postsynaptic membrane of the neuromuscular junction. Plasma exchange presumably removes these circulating antibodies. Prednisone is an effective therapy in the treatment of patients with myasthenia gravis. However, it is not appropriate as the initial, sole therapy in patients in the midst of a myasthenic crisis. A worsening of muscular weakness is seen in some patients within the first 3 weeks of the initiation of this medication. Even if this complication does not occur, the beneficial effects from prednisone typically do not occur for 3 to 4 weeks after drug initiation. In a patient with myasthenic crisis, prednisone is often initiated after the patient has been stabilized and is improving with plasma exchange. Used in this fashion, prednisone should reach maximum efficacy by the time the beneficial effects of plasma exchange have waned (6-8 weeks). Pyridostigmine is an inhibitor of aceylcholinesterase, which increases the amount of acetylcholine in the neuromuscular junction. This medication can be given orally or intravenously and may offer symptomatic benefit in patients with myasthenia gravis. Patients with mild symptoms due to myasthenia gravis can occasionally be treated with pyridostigmine alone. Although this medication may be of some benefit in patients with myasthenic crisis and should be considered, definitive immune-modulating therapy is indicated as the most appropriate treatment. Ciprofloxacin and other fluoroquinolone medications are contraindicated in patients with myasthenia gravis; the ciprofloxacin the patient received 3 weeks ago to treat a urinary tract infection may have precipitated her current myasthenic crisis. Other medications that should be avoided in patients with myasthenia gravis include lithium, aminoglycosides, magnesium, and macrolide antibiotics.
question
Neuro 14 A 33-year-old man is evaluated for a 3-year history of progressive gait dysfunction accumulating in a stepwise manner, with periods of rapid worsening followed by a plateau in symptoms until the next relapse. He has no personal history of other medical problems. On physical examination, vital signs are normal. Severe spastic paraparesis, moderate loss of all sensory modalities in the lower extremities, and a T10 sensory level are noted. An MRI of the thoracic spinal cord is shown . An MRI of the brain shows no abnormalities. Cerebrospinal fluid analysis detects no oligoclonal bands. Which of the following is the most appropriate next diagnostic test? A Repeat lumbar puncture B Repeat MRI of the brain C Spinal angiography D Spinal cord biopsy
answer
C Spinal dural arteriovenous fistula should be suspected in any patient with a chronic or subacute progressive myelopathy with episodes of more rapid, stepwise clinical deterioration. The most appropriate next diagnostic test for this patient is spinal angiography for the specific purpose of confirming the presence of a spinal dura-based arteriovenous fistula. Dural arteriovenous fistula should be suspected in any patient with a chronic or subacute progressive myelopathy with episodes of more rapid, stepwise clinical deterioration. In this patient, the diagnosis is suggested by the clinical history of stepwise spinal cord dysfunction without significant recovery and the MRI finding of a swollen spinal cord with dorsal flow voids, which is indicative of tortuous blood vessels. The spinal cord lesion represents cord congestion due to an abnormal connection between the high-pressure arterial and low-pressure venous drainage systems serving the spinal cord circulation. Patients with such spinal fistulas are often mistakenly diagnosed as having multiple sclerosis (MS), which is the most common nontraumatic disabling disease of young adults. Although repeating the cerebrospinal fluid analysis and the MRI of the brain would be a reasonable way to reevaluate the validity of an MS diagnosis, the spinal cord MRI findings in this patient are not typical of MS and make these repeated studies unnecessary. Spinal cord biopsy would be useful if there were a strong clinical suspicion of an unusual inflammatory myelopathy or a cord tumor. Nothing in the patient's examination or laboratory and imaging findings suggests either possibility.
question
Neuro 15 A previously healthy 50-year-old woman is admitted to the hospital after three recent, transient episodes of nonfluent aphasia and right-hand numbness and weakness. One week before onset of these focal neurologic symptoms, she developed aching left jaw pain, which has persisted. She has no other medical problems and takes no medications. On physical examination, vital signs are normal. A left carotid bruit is heard on auscultation. Left miosis and left ptosis are noted. Results of laboratory studies and a CT scan of the head are normal. Which of the following is the most likely diagnosis? A Cluster headache B Giant cell arteritis C Spontaneous left internal carotid artery dissection D Spontaneous left vertebral artery occlusion
answer
C Dissection of the cervical arteries, although an infrequent occurrence, is a leading cause of stroke in young and otherwise healthy persons. The most likely clinical diagnosis to explain this patient's focal neurologic symptoms is a left internal carotid artery dissection with resultant transient ischemic attacks (TIAs). Carotid dissection characteristically develops after head or neck trauma but may occur spontaneously. Manifestations of this condition include ipsilateral throbbing neck, head, or orbital pain with possible Horner syndrome. The pain from carotid dissection may occur suddenly or develop gradually. Such a dissection can cause a TIA or ischemic stroke by one of two mechanisms: either the mural hematoma expands to the point of occluding the lumen, or a thrombus forms and embolizes to cause distal arterial branch occlusion. Dissection of the cervical arteries, although an infrequent occurrence, is a leading cause of stroke in young and otherwise healthy persons. Magnetic resonance angiography of the carotid arteries and carotid duplex ultrasonography are indicated in the clinical evaluation of possible carotid dissection. A cluster headache is an excruciating unilateral headache of extreme intensity, with a typical duration of 15 minutes to 3 hours. Its pain is lancinating or boring in quality and is located behind the eye (periorbital) or in the temple, sometimes radiating to the neck or shoulder. The cardinal symptoms of a cluster headache attack are ptosis (drooping eyelid), conjunctival injection (redness of the conjunctiva), lacrimation (tearing), rhinorrhea (a runny nose), and, less commonly, facial blushing, swelling, or sweating. These features are known as the autonomic symptoms. The neck is often stiff or tender in the aftermath of a headache, with jaw or tooth pain sometimes present. Although this patient had ptosis and jaw pain, there were no other features to support a diagnosis of cluster headache. Additionally, one would not expect a patient with a cluster headache to have focal cortical dysfunction (such as aphasia). Giant cell arteritis (or temporal arteritis) is a granulomatous vasculitis that predominantly affects the extracranial branches of the carotid artery. This condition occurs almost exclusively in patients older than 50 years of age and has a female predominance. Clinical manifestations of giant cell arteritis include headache, jaw or tongue claudication, scalp tenderness, systemic symptoms, and fever. Involvement of the primary branches of the aorta also may cause limb claudication. Giant cell arteritis is not associated with pain along the carotid artery or cerebral hemispheric symptoms and would be an unusual diagnosis in this relatively young patient. The patient's clinical symptoms are aphasia, right hemiparesis, and right hemisensory loss; all are referable to the middle cerebral artery territory of the dominant left hemisphere and thus are consistent with an anterior circulation stroke event. There are no posterior circulation symptoms (in the vertebrobasilar artery territory) to suggest that a vertebral artery occlusion has occurred. Although an ipsilateral Horner syndrome can accompany a lateral medullary infarction secondary to a vertebral artery occlusion, a patient with this condition would also be expected to have dysphagia, hoarseness, a reduced gag reflex, vertigo, nystagmus, vomiting, ipsilateral cerebellar findings, ipsilateral loss of pain and temperature sensations in the face, and contralateral loss of pain and temperature sensations in the body, none of which this patient has.
question
Neuro 16 A 75-year-old woman is seen for a follow-up evaluation. Two weeks ago, she was brought to the emergency department after slipping in the bathroom and hitting her head on the tub. A CT scan of the head obtained at that time showed a small calcified lesion over the right sphenoid wing. A subsequent outpatient contrast-enhanced MRI of the brain showed a 2-cm dura-based lesion with homogeneous enhancement, consistent with a meningioma; no mass effect or edema was detected. She is otherwise healthy, has no general or neurologic symptoms, and takes no medications. Vital signs and physical examination findings are normal. Which of the following is the best management option for this patient? A Chemotherapy B Serial MRIs of the brain C Stereotactic radiosurgery D Surgical resection
answer
B Small, asymptomatic meningiomas in older adults should be monitored with serial imaging. This patient should have serial MRIs of the brain. Meningiomas are benign, slow-growing tumors that are often discovered as an incidental finding when head imaging is obtained for evaluation of unrelated symptoms. Their appearance on CT scans and MRIs is diagnostic. Appropriate management depends on the age of the patient and the size and location of the tumor. For this patient, who is elderly and has a small, asymptomatic meningioma, the most appropriate management strategy is to follow up with serial MRIs of the brain. In patients with incidentally diagnosed meningiomas, most clinicians perform imaging studies every 3 to 6 months during the first year. If the tumor remains stable, yearly imaging may be performed for the next 5 to 10 years. Densely calcified meningiomas are particularly indolent. Chemotherapy is not warranted in this patient because meningiomas are usually benign in histology and behavior. In patients with large or symptomatic meningiomas, surgical resection is the treatment of choice, whereas observation usually is appropriate for small, asymptomatic meningiomas until evidence of progressive enlargement or symptoms develop. Stereotactic radiosurgery, which selectively irradiates a sharply defined target, is used for symptomatic meningiomas that are not amenable to surgical resection. Surgical resection is generally recommended only for patients who are symptomatic or for young patients in whom it is likely that symptoms will develop over time with a slow-growing tumor.
question
Neuro 17 A 36-year-old woman is evaluated in the office for a 6-year history of headache. The patient says her headaches occur approximately three times each month, are associated with occasional nausea and pain that can be moderately severe and disabling, have a squeezing quality, and begin in the neck, radiating forward to involve the frontal head region bilaterally. Her headaches are preceded by yawning and irritability, last up to 24 hours, and can be triggered by stress or changes in barometric pressure. She has a history of mild depression treated with fluoxetine. Her mother and sister have a history of sinus headaches. On physical examination, blood pressure is 124/86 mm Hg, pulse rate is 78/min, and BMI is 21. Results of general physical and neurologic examinations are normal. The patient's mood appears euthymic. Which of the following is the most likely diagnosis? A Chronic migraine B Medication overuse headache C Migraine without aura D Sinus headache E Tension-type headache
answer
C This patient most likely has migraine without aura. Approximately 90% of patients in an ambulatory care setting with a chief symptom of recurrent, moderately severe headache have a form of migraine. Her 6-year history of episodic headache meets the International Classification of Headache Disorders criteria for migraine. There is no single criterion that is necessary or sufficient to make the diagnosis of migraine. In this particular example, this patient does not have unilateral or throbbing pain, and there is no associated photophobia, phonophobia, or vomiting, features that are often considered necessary for the diagnosis. Her headache is preceded by premonitory symptoms (yawning and irritability), which occur in approximately 75% of patients with migraine. In addition, her attacks are triggered by stress and changes in barometric pressure, both of which are frequent triggers for migraine. Depression is a comorbid disorder that often leads to a misdiagnosis of tension-type headache. Chronic migraine (previously referred to as "transformed" migraine) is a very common cause of chronic daily headaches and is defined as headaches occurring on more than 15 days per month that meet the diagnosis of migraine without aura or respond to ergots or triptans (migraine-specific drugs) on at least 8 of those days. This patient does not meet the diagnostic criteria for this type of headache. Another chronic daily headache often confused with chronic migraine is medication overuse headache. This type of headache, which typically occurs when (or soon after) a patient awakens, is defined as more than 15 headache days per month and more than 10 days of use of acute headache medication per month. Analgesic therapy may decrease headache intensity, but the headache usually will not completely remit. Although the symptoms of this condition may be identical to those of migraine or tension-type headache, the efficacy of migraine-specific therapy in patients with medication overuse headache is intermittent or poor. Patients with this condition respond only when the medication overuse is addressed. Again, the patient described in the vignette does not meet the diagnostic criteria for this type of headache. Headache triggered by changes in barometric pressure is often mistakenly diagnosed as sinus headache. Patients with acute rhinosinusitis and sinus headache have symptoms consisting of nasal congestion, purulent nasal secretions, sinus tenderness, and facial pain. These findings are not present in this patient. Tension-type headache is a dull, bilateral, or diffuse headache that is often described as a pressure or squeezing sensation of mild to moderate intensity. This type of headache has no accompanying migraine features (such as nausea), and its pain does not prohibit activity, characteristics that do not match this patient's symptoms. Headache triggered by stress is often mistaken as tension-type headache, even though stress is one of the most commonly reported migraine triggers. Up to 75% of patients with migraine report neck pain that precedes or occurs during the attack, and this scenario is frequently mistaken for tension-type headache.
question
Neuro 18 A 70-year-old woman is evaluated for recent loss of consciousness. She says she had a "strange feeling" before losing consciousness. Her husband, who accompanied her on this visit and witnessed the event, reports that she fell to the ground and exhibited generalized stiffening and then shaking. He further states that her eyes rolled back in her head and that she was unresponsive for 1 minute, had urinary incontinence, and was profoundly confused, speaking garbled, nonsensical sentences, for 2 to 3 minutes after regaining consciousness. She had a similar episode 2 months ago; evaluation at that time, which included electrocardiography, stress electrocardiography, and a continuous-loop event electrocardiographic recorder, revealed no abnormal findings. The patient takes no medications. Results of a physical examination, including a neurologic examination, are normal. Results of MRI of the brain and electroencephalography during waking and sleeping are also normal. Which of the following is the most likely diagnosis? A Cardiac arrhythmia B Partial seizure with secondary generalization C Transient ischemic attack D Vasovagal syncope
answer
B Normal findings on an electroencephalogram and MRI do not rule out a diagnosis of seizure. The history and provided eyewitness account are most consistent with the diagnosis of a partial seizure with secondary generalization. This type of seizure is characterized by sensory, motor, autonomic, or psychic (déjà vu, depersonalization) phenomena with or without altered awareness at onset followed by generalized tonic-clonic activity. Accurate classification of a seizure relies predominantly on the clinical history, with diagnostic testing used to confirm or clarify the suspected cause. When a patient has loss of consciousness with urinary incontinence and stiffening and shaking lasting 1 to 2 minutes, seizure should always be very strongly considered as the cause. When there is a high clinical suspicion of epilepsy, as in this patient, normal findings on an MRI of the brain and electroencephalogram (EEG) do not rule out that diagnosis. An MRI shows an epileptogenic lesion in only 10% to 20% of patients with epilepsy, and a routine EEG shows abnormalities in only 25% to 50% of patients with confirmed epilepsy. Approximately one third of syncopal episodes are reflex mediated or neurocardiogenic. Cardiac arrhythmias are responsible for approximately 15% of syncopal episodes and are most common in patients with cardiac risk factors or evidence of structural heart disease. Onset is usually sudden, may or may not be associated with palpitations, and can occur in any position. However, this patient's previous normal cardiac evaluation decreases the likelihood of an arrhythmia, and incontinence rarely results from causes of transient loss of consciousness other than seizure. The extended postictal period of confusion and speech impairment seen in this patient is also atypical of cardiac arrhythmia. Patients recovering from loss of consciousness due to arrhythmia are more typically alert or at most mildly confused after the loss of consciousness. Although the premonitory aura this patient experienced is most suggestive of seizure, such an aura has also been reported in cases of cerebral hyperperfusion from cardiac arrhythmia or neurocardiogenic syncope. A transient ischemic attack is not associated with this patient's findings and is not associated with loss of consciousness. Vasovagal syncope occurs as the result of sudden vasodilatation and bradycardia resulting in hypotension and cerebral hypoperfusion and is the most common cause of neurocardiogenic syncope. Syncope with pain, emotional stress, cough, micturition, or defecation supports a vasovagal cause, which is often preceded by warning symptoms of lightheadedness, nausea, and diaphoresis. Generalized stiffening and shaking lasting seconds can occur with vasovagal syncope due to reduced cerebral blood flow, but this type of syncope does not explain the aura, prolonged loss of consciousness, postictal confusion, and incontinence experienced by the patient. Thus, this diagnosis is unlikely.
question
Neuro 19 A 53-year-old woman is evaluated in the office for a 1-week history of paresthesias that began symmetrically in the feet and progressed to involve the distal legs and, more recently, the hands. She says she is unsteady when walking, has lower limb weakness, and has difficulty going upstairs. The patient has no history of pain or bowel or bladder impairment. Personal and family medical history is noncontributory, and she takes no medications. On physical examination, vital signs are normal. Weakness of distal lower extremity muscles is noted, with stocking-glove sensory loss and areflexia. Deep tendon reflexes are absent. Plantar responses are flexor, and gait is unsteady. No sensory level is present across the thorax. Mental status, language, and cranial nerve function are normal. Complete blood count results, erythrocyte sedimentation rate, serum creatinine and creatine kinase levels, and liver chemistry test results are normal. A chest radiograph shows no abnormalities. Which of the following is the most appropriate next diagnostic test? A Electromyography B MRI of the spinal cord C Serologic testing for West Nile virus D Sural nerve biopsy
answer
A Guillain-Barré syndrome is a disorder associated with rapidly progressive extremity weakness, paresthesias, and areflexia. This patient should undergo electromyography (EMG). She has a rapidly progressive disorder affecting the peripheral nervous system, most compatible with a clinical diagnosis of Guillain-Barré syndrome. Patients with Guillain-Barré syndrome typically develop paresthesias distally in the lower extremities that are followed by limb weakness and gait unsteadiness. In addition to sensory loss and limb weakness, deep tendon reflexes are characteristically absent or markedly reduced. The diagnosis is confirmed by EMG, which usually shows a demyelinating polyradiculoneuropathy. Cerebrospinal fluid (CSF) analysis characteristically shows albuminocytologic dissociation, whereby the spinal fluid cell count is normal but the spinal fluid protein level is elevated. CSF analysis may also yield normal results early in the course of the disease. However, a normal CSF cell count is useful in excluding other infectious conditions, such as polyradiculoneuropathies associated with HIV and cytomegalovirus infection, infection due to West Nile virus, and carcinomatous or lymphomatous nerve root infiltration. By definition, symptoms in patients with Guillain-Barré syndrome peak within 4 weeks of onset. Poor prognostic features include rapid symptom progression, respiratory failure, EMG evidence of axonal degeneration, and advanced age. Intravenous immune globulin and plasma exchange are equally efficacious in the treatment of Guillain-Barré syndrome. MRI of the spinal cord would be inappropriate as the next diagnostic test. The patient's presentation of distal extremity sensory loss with areflexia suggests a disorder of the peripheral nervous system. A spinal cord lesion (myelopathy) would be an unlikely cause of the symptoms noted on clinical examination. The absence of bowel or bladder impairment, the lack of a sensory level across the thorax, and the upper and lower limb areflexia argue against a central nervous system disorder affecting the spinal cord. West Nile virus infection should be considered in every patient with symptoms of extremity weakness that begin acutely and progress over days to weeks. However, the presence of paresthesias and sensory loss on examination are not typical of West Nile virus infection. This enteroviral illness affects the anterior horn cells, causing limb weakness in the absence of sensory loss. Most cases of West Nile virus infection cause only minor symptoms that are indistinguishable from those of other viral illnesses. Biopsy of the sural nerve is considered in the diagnostic evaluation of patients with a marked peripheral neuropathy of unknown cause. The sural nerve is a sensory nerve that supplies sensation to the lateral distal leg and lateral aspect of the foot. Sural nerve biopsy is appropriate in patients with suspected vasculitis or amyloidosis and occasionally in patients with chronic inflammatory demyelinating polyradiculoneuropathy; it is also used to exclude neuropathic conditions resulting from neoplastic infiltration or other inflammatory conditions, such as sarcoidosis. Symmetric signs and symptoms and diffuse areflexia are not typical of vasculitis. The acute onset and rapid symptom progression would argue against amyloidosis, chronic inflammatory demyelinating polyradiculoneuropathy, or other infiltrating peripheral nerve disorders. Sural nerve biopsy is not necessary in patients with suspected Guillain-Barré syndrome.
question
Neuro 20 A previously healthy 42-year-old woman is evaluated in the emergency department for the sudden onset of a severe occipital headache during defecation 8 hours ago, followed by two episodes of vomiting. She reports no neck stiffness or neurologic symptoms. Her mother and two sisters have a history of migraine. On physical examination, temperature is 36.8 °C (98.2 °F), blood pressure is 148/88 mm Hg, pulse rate is 90/min, and respiration rate is 20/min. The patient is in obvious distress as a result of the pain. No evidence of meningismus, papilledema, or focal neurologic signs is found. Cerebrospinal fluid examination reveals a normal level of protein and glucose and no leukocytes or erythrocytes. A noncontrast CT scan of the head shows no abnormalities. Which of the following is the most appropriate next step in management? A Admission to the hospital for overnight observation B Administration of sumatriptan, subcutaneously C CT angiography of the head and neck D Repeat lumbar puncture
answer
C Thunderclap headache is a potential neurologic emergency that requires urgent imaging of the cerebral vasculature with either magnetic resonance or CT angiography after a noncontrast CT scan of the head and a lumbar puncture have excluded subarachnoid hemorrhage. This patient should undergo CT angiography of the head and neck. She has experienced a thunderclap headache, which is a severe and explosive headache that is maximal in intensity at or within 60 seconds of onset. Every thunderclap headache must be immediately evaluated to detect potentially catastrophic conditions, especially subarachnoid hemorrhage. Most of the other causes of thunderclap headache, such as an unruptured cerebral aneurysm, a carotid or vertebral artery dissection, cerebral venous sinus thrombosis, and reversible cerebral vasoconstriction syndrome, can be excluded by noninvasive angiography. Therefore, CT angiography of the head and neck is the most appropriate next step in management. CT angiography can detect unruptured aneurysms as small as 3 mm in diameter and thus is adequate to exclude this diagnosis. Magnetic resonance angiography (MRA) would also be appropriate in this setting. Both CT angiography and MRA can be performed with a venous phase to exclude cerebral venous sinus thrombosis. Given that most causes of thunderclap headache can be excluded by such noninvasive angiography and that prior cerebrospinal fluid analysis has shown no evidence of a subarachnoid hemorrhage in this patient, conventional cerebral angiography, in which a catheter is inserted into a large artery and advanced through the carotid artery, is unnecessary. Because of the potential for neurologic morbidity associated with several of the causes of thunderclap headache, admission for observation without evaluation of the cerebral vasculature would not be the best management option. Similarly, treatment with a vasoconstrictive drug, such as sumatriptan, would not be appropriate until the other causes of thunderclap headache have been excluded. Drugs with the potential to constrict extracranial and intracranial cerebral vessels can precipitate or exacerbate the cerebral ischemia that may be associated with arterial dissection and reversible cerebral vasoconstriction syndromes. Although it may take up to 6 hours for subarachnoid blood to sediment into the lumbar thecal sac, this patient's first lumbar puncture occurred more than 8 hours after the onset of symptoms. Repeating the lumbar puncture would, therefore, be unnecessary.
question
Neuro 21 A 74-year-old man is brought to the emergency department by ambulance 1 hour after he had an acute witnessed onset of aphasia and right hemiparesis. He has a history of hypertension and atrial fibrillation. His current medications are hydrochlorothiazide, metoprolol, and warfarin. On physical examination, blood pressure is 178/94 mm Hg and pulse rate is 80/min and irregular. Neurologic examination confirms nonfluent aphasia, a right pronator drift, a right leg drift, and an extensor plantar response on the right. Results of laboratory studies performed within 1 hour of his arrival at the emergency department show an INR of 1.5. An electrocardiogram obtained on the patient's arrival at the emergency department confirms atrial fibrillation. A CT scan of head obtained within 1 hour of his arrival reveals no early ischemic changes. Which of the following is the best treatment? A Aspirin B Continuous intravenous heparin C Intra-arterial recombinant tissue plasminogen activator (rtPA) D Intravenous labetalol E Intravenous rtPA
answer
E A patient with acute ischemic stroke who is taking warfarin but who has a subtherapeutic INR (≤1.7) is still eligible to receive thrombolysis with intravenous recombinant tissue plasminogen activator within the recommended window of 3 hours from stroke onset. This patient should receive intravenous recombinant tissue plasminogen activator (rtPA). He has clinical symptoms and signs and radiologic evidence of an acute left hemispheric stroke. The probable mechanism of stroke is a cardioembolism, given his history of atrial fibrillation and his subtherapeutic INR. He was brought to the emergency department within 1 hour of the witnessed onset of stroke symptoms, and his evaluation is completed 1 hour later. He does not appear to have any clinical, radiologic, or laboratory contraindication to receiving the preferred treatment of intravenous rtPA, and he can receive it within the recommended window of 3 hours from stroke onset. The standard dose is 0.9 mg/kg. Although he is on warfarin, rtPA is not contraindicated because his INR is 1.5; an INR less than or equal to 1.7 is required for rtPA administration in a patient on anticoagulation. Aspirin is indicated for acute ischemic stroke in patients who are not eligible for rtPA, as this patient is. For patients with acute stroke who are eligible for thrombolysis, aspirin should be withheld in the emergency department and for 24 hours after rtPA administration. Although long-term anticoagulation is an effective treatment for prevention of cardioembolic stroke in patients with atrial fibrillation, acute anticoagulation with heparin has not been shown to be beneficial in patients with acute ischemic stroke. Intra-arterial administration of rtPA is indicated for selected patients with acute stroke who have major intracranial artery occlusion within 6 hours of symptom onset and who are not otherwise candidates for intravenous administration of rtPA. This patient was eligible for intravenous administration of rtPA within 3 hours of stroke onset. The latter treatment takes precedence. Elevated blood pressure is common at the time of initial stroke presentation, even among patients without chronic hypertension. Rapid lowering of blood pressure may further impair cerebral blood flow and worsen the ischemic injury. Elevated blood pressure often will resolve spontaneously or improve gradually during the first few days after a stroke. The threshold for acute blood pressure lowering in patients with acute stroke who are eligible for thrombolysis is 185/110 mm Hg. In such a setting, preferred agents include intravenous infusions of labetalol or nicardipine. Because this patient's blood pressure is already below that threshold, there is no indication for intravenous use of labetalol at this time.
question
Neuro 22 A 65-year-old man is evaluated for worsening gait unsteadiness and falls. He first noticed the unsteadiness 1 year ago while walking and has started to fall recently, falling four times in the past 2 weeks. Approximately 3 years ago, he developed erectile dysfunction and has had increasing constipation ever since that time. On physical examination, vital signs are normal except for the supine blood pressure, which is 190/105 mm Hg; blood pressure decreases to 76/50 mm Hg when he stands without a compensatory increase in the pulse rate. Results of mental status testing are normal. He has mildly slurred speech. Testing of cranial nerve function, including testing of extraocular movements, reveals no abnormalities. Manual muscle strength in the upper and lower extremities is normal, but he has mild rigidity of the extremities and mild appendicular ataxia. His gait is slow with a reduced stride length and arm swing, and he has marked postural instability. Which of the following is the most likely diagnosis? A Dementia with Lewy bodies B Multiple system atrophy C Parkinson disease D Progressive supranuclear palsy
answer
B Multiple system atrophy is a sporadic, heterogeneous, neurodegenerative disorder that causes impairment of multiple neurologic systems, including the autonomic nervous system, the extrapyramidal system, and the cerebellum. Multiple system atrophy is the most likely diagnosis in this patient. He has a progressive neurologic disorder characterized by signs and symptoms that suggest impairment of multiple neurologic systems; these include the autonomic nervous system (orthostatic hypotension, erectile dysfunction, constipation), the extrapyramidal system (rigidity, impaired gait), and the cerebellum (limb ataxia). Multiple system atrophy is a progressive, ultimately fatal neurodegenerative disorder that typically causes dysautonomia, parkinsonism, and ataxia, in some combination, in affected patients. Multiple system atrophy is a clinical diagnosis that is suggested by the presence of these various features in the same patient. Dementia with Lewy bodies is also typically associated with parkinsonian features and should be considered in the differential diagnosis of this patient. Dementia with Lewy bodies is associated with cognitive impairment, parkinsonian signs and symptoms, and possible evidence of dysautonomia. However, gait or limb ataxia is not expected, and the degree of dysautonomia typically is not as severe as that seen in multiple system atrophy. The prominent dysautonomia, early falls, absence of a resting tremor, and presence of appendicular ataxia in this patient argue against Parkinson disease as the diagnosis. Early multiple system atrophy can, however, be difficult to distinguish from Parkinson disease, especially because some affected patients may respond initially to carbidopa-levodopa, a medication used to treat parkinsonian symptoms in Parkinson disease. Progressive supranuclear palsy should also be part of the differential diagnosis in this patient. A rare neurodegenerative disorder, progressive supranuclear palsy is associated with parkinsonian signs and early falls due to marked postural instability. However, significant dysautonomia and ataxia are not expected. Marked impairment in vertical gaze is a hallmark of progressive supranuclear palsy.
question
Neuro 23 A 60-year-old man is admitted to the hospital for generalized tonic-clonic seizures that began 30 minutes ago. Clinical seizure activity continues for another 60 minutes, during which time the patient is intubated, placed on a ventilator, and given lorazepam and fosphenytoin, both intravenously. After his seizures stop, he is transferred to the intensive care unit, where he remains comatose. The patient has been receiving chemotherapy and whole-brain radiation therapy for recently diagnosed small cell lung cancer. He indicated at the time of his diagnosis that he wants everything possible done to prolong his life ("full code"). On examination, the patient is comatose, with no response to deep pain stimulation. Cranial nerve examination is significant for reactive pupils. Results of laboratory studies are noncontributory. Other than sinus tachycardia, the electrocardiogram is unremarkable. Which of the following is the most appropriate next step in management? A Continuous electroencephalographic monitoring B Discussion of withdrawal of care with the family C Intravenous phenobarbital D MRI of the brain
answer
A Continuous electroencephalographic monitoring is indicated for patients who remain unresponsive after resolution of clinical status epilepticus to distinguish nonconvulsive status epilepticus from a postictal state. This patient should undergo continuous electroencephalographic monitoring. Generalized convulsive status epilepticus, which this patient initially exhibited, is a neurologic emergency with significant morbidity and mortality. Complications include hemodynamic instability, acidosis, rhabdomyolysis, pulmonary edema, respiratory failure, and irreversible brain injury. His early treatment with lorazepam was appropriate as first-line therapy. Over time, the clinical manifestations of status epilepticus can become increasingly subtle, but the electrical seizures can continue unabated, a state referred to as subtle or nonconvulsive status epilepticus. Motor manifestations may be limited to rhythmic muscle twitching in the face or eyes or to low-amplitude multifocal myoclonus in an obtunded or comatose patient. The unwary or inexperienced observer may mistake subtle generalized convulsive status epilepticus for postictal obtundation. Failure to diagnose and treat ongoing status epilepticus increases the risk of an adverse outcome, even with ventilatory and critical care support. Ongoing electrical seizure activity will lead to increasing physiologic derangement and neuronal injury. In this patient and others who remain unresponsive or have persistent altered mentation after treatment for clinical status epilepticus, it may be impossible to distinguish on clinical grounds alone the effect of ongoing electrical seizure activity from postictal lethargy or the side effects of medication. For this reason, continuous electroencephalographic monitoring is strongly advocated for optimal management. If ongoing status epilepticus is demonstrated, additional intravenous antiepileptic drugs should be administered at doses sufficient to terminate the electroencephalographic seizures. Withdrawal of care at this point is premature because the patient has not yet had an adequate evaluation. Adding phenobarbital or other medications to this patient's drug regimen carries the risk of complications, such as hypotension, and thus would not be the best choice, unless the patient were still experiencing seizures. Obtaining an MRI of the brain should never delay other necessary diagnostic tests or therapies required to treat the status epilepticus. MRI is also unlikely to provide useful additional information in this patient whose cerebral metastasis constitutes a known underlying cause of his seizure activity.
question
Neuro 24 A 71-year-old woman is admitted to the hospital after having a witnessed morning seizure. She has a 3-week history of increasing gait unsteadiness and daytime somnolence and a 3-month history of progressive confusion and headaches; she typically naps 3 hours daily. Six month ago, the patient had fungal pneumonia caused by Coccidioides immitis infection. She has a 20-year history of type 2 diabetes mellitus and no relevant family history. Current medications are insulin glargine, metformin, and intravenous fosphenytoin (started on hospital admission). On physical examination, temperature is 38.2 °C (100.8 °F), blood pressure is 116/70 mm Hg, pulse rate is 96/min, respiration rate is 18/min, and BMI is 32. The patient is somnolent but arouses to voice. She is disoriented to time and place and scores only 12/30 on the Folstein Mini-Mental State Examination. There is mild diffuse symmetric hyperreflexia, and plantar responses are extensor bilaterally; the patient moves all four limbs equally. There is no papilledema. Leukocyte count is 14,000/µL (14 × 109/L), and erythrocyte sedimentation rate is 64 mm/h. All other results of laboratory studies, including measurement of serum vitamin B12 level, a basic metabolic panel, thyroid function tests, and urinalysis, are normal. A noncontrast CT scan of the head shows mildly dilated ventricles. An electroencephalogram shows moderately severe diffuse slowing but no epileptiform activity. Which of the following is the most appropriate next diagnostic test for this patient? A Apolipoprotein E genotyping B Cerebral CT angiography C Cisternography D Lumbar puncture and cerebrospinal fluid analysis E Repeat electroencephalography
answer
D Cerebrospinal fluid analysis should be considered in any patient with potentially reversible causes of impaired cognition, particularly if meningoencephalitis is suspected. This patient has fungal meningitis due to Coccidioides immitis infection and thus should undergo lumbar puncture and subsequent cerebrospinal fluid analysis to detect the potentially reversible cause of her cognitive and other impairments. A 71-year-old woman with declining cognition may seem an obvious candidate for Alzheimer dementia, but any pathologic process affecting the brain can impair cognition. The symptoms described—subacute onset, headache, fluctuating level of alertness, fever, peripheral blood markers of inflammation, and mildly dilated ventricles on the CT scan—should all provoke suspicion of an infectious, inflammatory, or otherwise reversible cause rather than a degenerative one. The scenario described is of a patient with encephalopathy whose clinical, laboratory, and radiologic findings suggest some type of meningitic process. Because of the time course, an atypical infectious agent, such as fungal meningitis, should be considered, especially in light of her history of fungal pneumonia in the past. Other causes of chronic or subacute meningoencephalitis, such as autoimmune encephalopathy and carcinomatous meningitis, need also be considered. Apolipoprotein E (APOE) genotyping will not provide useful information in this patient. Specifically, discovering if this patient carries the APOE e4 allele and thus has an elevated risk for Alzheimer dementia would not help explain or treat her current symptoms and signs but could delay finding the probable infectious, inflammatory, or related cause of her symptoms. Although cerebral vasculitis can produce a clinical picture similar to this one, cerebral CT angiography is not sufficiently sensitive to diagnose vasculitis of the central nervous system. Making that diagnosis usually requires either invasive intra-arterial angiography or brain and leptomeningeal biopsy. The more usual application of cerebral CT angiography is in the assessment of acute stroke, of which there is no evidence in this patient. The finding of mildly dilated ventricles on this patient's CT scan is not relevant to her signs and symptoms, so further confirmatory testing by cisternography is unlikely to reveal the underlying cause of her findings. It is highly unlikely that a repeat electroencephalogram (EEG) will give much further insight into this patient's condition, given that the EEG obtained during the patient's symptomatic period showed no epileptiform activity. Seizures are a potential complicating feature of any meningitic or encephalitic process, but the occurrence of a seizure or EEG finding of occasional interictal epileptiform activity will not provide any additional information about the underlying cause of this patient's symptoms.
question
Neuro 25 A 30-year-old man has a recent diagnosis of multiple sclerosis (MS). He experienced two transient neurologic episodes in the past 6 months, one involving optic neuritis and the other minor partial myelitis; he recovered completely from both events and is currently asymptomatic. MS was diagnosed after an MRI of the brain showed white matter lesions typical of the disease. He has no other pertinent personal or family medical history. Which of the following MS subtypes best describes the course of his disease? A Benign B Primary progressive C Relapsing-remitting D Secondary progressive
answer
C Multiple sclerosis begins as a relapsing-remitting disorder in 85% of patients and a primary progressive disorder in 15% of patients; those with the relapsing-remitting type have a greater than 50% risk of developing a secondary progressive disease course. This patient experienced an episode of transient neurologic dysfunction (an attack) at disease onset and thus has relapsing-remitting multiple sclerosis (MS). Eighty-five percent of patients with MS have this type of disease onset. In relapsing-remitting MS, relapse frequency declines over time, and relapses do not become more severe with increasing disease duration; however, recovery from individual events may be slower and less complete. Primary progressive MS, which the other 15% of patients with MS have at disease onset, is defined as gradually worsening neurologic function over more than 1 year without recovery. This patient has recovered completely from two neurologic episodes and is currently asymptomatic. Therefore, he cannot be classified as having primary progressive MS. Secondary progressive MS is characterized by gradual, unremitting development of new symptoms over months to years in a patient who previously had a relapsing-remitting course. The lifetime risk of conversion from relapsing-remitting to secondary progressive disease is greater than 50%, but the onset and rate of the progressive phase are highly variable and not predictable for individual patients. The median time from MS onset until conversion to the secondary progressive phase typically ranges from 10 to 15 years. The establishment of a secondary progressive course is a risk factor for substantial disability, such as loss of independent ambulatory function; the median time from MS onset to the point at which unilateral gait assistance (such as a cane) is required is 15 to 25 years. This patient first had transient symptoms only 6 months ago and thus cannot be characterized as having secondary progressive disease. Benign MS is defined loosely as no or minimal neurologic impairment 15 or more years after MS onset. This category may encompass as many as 20% of all patients with MS. The definition of benign MS is controversial because continued follow-up of such patients often uncovers late progressive disease and disability accrual. However, a small minority of patients with MS live a long and essentially unrestricted life. The factors that predict a benign course early in the disease have not yet been identified. This patient's MS diagnosis is too recent to be categorized as benign at this point.
question
Neuro 26 A 22-year-old man is evaluated in the emergency department for a 12-hour history of mild headache, nausea, and vomiting. His roommate had similar symptoms the previous day. He is given intravenous fluids and prochlorperazine and begins to feel better until his head suddenly becomes stiff and turns to the right; he cannot move it to the midline or to the left, and he reports cramping and aching in the right neck muscles. Neurologic examination shows the head to be turned to the right with sustained contraction of the left sternocleidomastoid muscle but is otherwise unremarkable. Which of the following is the best treatment for this patient? A Benztropine B Botulinum toxin C Phenytoin D Recombinant tissue plasminogen activator E Tetanus immune globulin
answer
A Medications that block dopamine receptors can cause acute dystonic reactions, which can be readily treated with benztropine. This patient is experiencing a prochlorperazine-induced dystonic reaction, which is best treated with benztropine. Acute dystonic reactions are characterized as torticollic (twisted or a turned head, neck, or face), oculogyric (deviated or rolling gaze), buccolingual (pulling sensation of the tongue), or opisthotonic (trunk or entire-body spasm). Medications, such as prochlorperazine, that block dopaminergic receptors in the extrapyramidal system can result in acute dystonic reactions. Intravenous anticholinergic agents, such as benztropine or diphenhydramine, are the treatment of choice in the acute treatment of dystonic reactions. Benzodiazepine medications can also be helpful in the acute setting. Oral anticholinergic medications are continued for 48 to 72 hours to prevent relapse. Cervical dystonia, or spasmodic torticollis, is a chronic condition associated with abnormal head posturing and tremor. Botulinum toxin is often beneficial for this condition. Acute dystonic reactions can be misdiagnosed as seizures, tetanus, stroke, and other conditions. Phenytoin is an antiseizure medication, tissue plasminogen activator is appropriate treatment for strokes under certain conditions, and tetanus immune globulin is administered for tetanus prophylaxis. None of these drugs is indicated for acute dystonic reactions.
question
Neuro 27 A 41-year-old man is evaluated in the emergency department for a 3-hour history of a severe headache. The headache began abruptly while he was at work and involved the left frontal temporal region and the left ear and jaw. The pain is pulsatile and severe. There are no associated symptoms. He has a history of migraine without aura. On physical examination, temperature is 36.2 °C (97.2 °F), blood pressure is 146/88 mm Hg, pulse rate is 68/min, respiration rate is 18/min, and BMI is 26. The patient has mild left ptosis and mild anisocoria (3 mm pupil on the left and 4 mm on the right). There is no meningismus. The remainder of the neurologic examination and funduscopic examination findings are normal. An electrocardiogram shows normal findings. Laboratory studies show a normal complete blood count, erythrocyte sedimentation rate, and serum chemistry levels. Results of a cerebrospinal fluid analysis are normal. An unenhanced CT scan of the head and neck shows no abnormalities. A magnetic resonance angiogram of the neck (left) and an MRI of the brain (right) are shown . Which of the following is the most appropriate next step in treatment? A Intra-arterial nimodipine B Intravenous dihydroergotamine C Intravenous heparin D Intravenous nitroglycerin E Stent-assisted aneurysm coiling
answer
C Acute unilateral headache with associated Horner syndrome represents acute carotid dissection until proved otherwise. This patient has a carotid dissection, for which he should receive heparin. Acute headache with oculosympathetic paresis (Horner syndrome) is a common presentation of carotid dissection and must be assumed to be carotid dissection until proved otherwise. Such a headache may precede the onset of cerebral ischemia (transient ischemic attack or stroke) by days to weeks. Additional manifestations of this condition may include ipsilateral throbbing neck or orbital pain. The pain from carotid dissection may occur suddenly or develop gradually. Neurologic abnormalities due to ischemia in the ipsilateral hemisphere (causing contralateral numbness or weakness) or retina (causing ipsilateral monocular visual symptoms) also may occur. Magnetic resonance angiography of the carotid arteries and carotid duplex ultrasonography are indicated in the clinical evaluation of possible carotid dissection. Although the management of carotid dissection is controversial, most experts agree on the intravenous use of heparin in the acute setting to prevent distal embolism and stroke. Acute severe headache always requires imaging of the neck and cerebral blood vessels to rule out a vascular cause. This patient's magnetic resonance angiogram of the neck shows a long, tapered stenosis of the left internal carotid artery in the neck, which indicates dissection. Although this patient's CT scan of the head and neck had normal results and MRI of the brain reveals no abnormalities of the brain parenchyma, the MRI does show dissection of the internal carotid artery. There is no evidence to support the use of intra-arterial nimodipine or intravenous nitroglycerin in this setting. The carotid stenosis in this patient is due to the mural hematoma in the wall of a blood vessel, not to an arterial spasm. In addition, these drugs may decrease systemic arterial pressure and cerebral perfusion pressure distal to the severe carotid stenosis. Despite his migraine history and his severe unilateral and pulsatile headache, this patient's current headache does not fulfill the diagnostic criteria for migraine; there are no associated features of nausea, vomiting, photophobia, or phonophobia. Because of its potent α-adrenergic stimulation, dihydroergotamine, a migraine therapy, is contraindicated in patients with conditions that predispose them to vasospastic reactions. This medication is thus not appropriate in the presence of a carotid dissection. Stent-assisted aneurysm coiling is not indicated in this patient because he does not have an intracerebral aneurysm, according to the results of his imaging studies.
question
Neuro 28 A 73-year-old retired woman is evaluated in the emergency department 6 hours after experiencing the sudden, explosive onset of a severe headache. The patient has hypertension controlled by diet and exercise. There is no relevant family history. She has no allergies and takes no over-the-counter medications. On physical examination, she is in obvious distress from the headache. Temperature is normal, blood pressure is 179/108 mm Hg, pulse rate is 119/min, and respiration rate is 14/min. There is no meningismus. No subhyaloid retinal hemorrhages are noted. Neurologic examination shows a normal level of consciousness and no focal abnormalities. Results of laboratory studies and a CT scan of the head without contrast are normal. Which of the following is the most appropriate next diagnostic test? A CT angiogram of the head B Lumbar puncture C Magnetic resonance angiogram of the brain D Magnetic resonance venogram of the brain E MRI of the brain
answer
B A lumbar puncture with subsequent cerebrospinal fluid analysis is necessary in any patient with thunderclap headache and normal findings on a CT scan to fully evaluate a possible subarachnoid hemorrhage. This patient should have a lumbar puncture. A thunderclap headache is a severe and explosive headache that is maximal in intensity at or within 60 seconds of onset. CT scanning is the first test to be conducted in a patient with thunderclap headache in whom a subarachnoid hemorrhage is suspected; a ruptured intracranial aneurysm is the most serious cause of such headaches. The ability to detect subarachnoid hemorrhage is dependent on the amount of subarachnoid blood, the interval after symptom onset, the resolution of the scanner, and the skills of the radiologist. On the day of the hemorrhage, extravasated blood will be present in more than 95% of patients, but in the following days, this proportion falls sharply. If an initial CT scan of the head reveals nothing, a lumbar puncture should be performed next in patients with this presentation. In an important minority (<5%) of patients with thunderclap headache who have no abnormalities on a CT scan and no blood in the cerebrospinal fluid (CSF), the CSF contains metabolites of hemoglobin that may be detected by spectrophotometry. This finding is diagnostic for subarachnoid hemorrhage. Subsequent angiography can confirm the presence of a ruptured aneurysm. It should be noted that spectrophotometry is not universally available in all hospitals. CT angiography and magnetic resonance angiography have similar test characteristics. Their sensitivity for detecting a ruptured aneurysm, with the more invasive conventional angiography as the gold standard, is currently 95%. CT angiography can be performed faster than can magnetic resonance angiography, but the latter uses no radiation and no contrast injection. Both modalities can play a role in the work-up of a patient with undiagnosed thunderclap headache when lumbar puncture and standard CT are nondiagnostic. Conditions other than aneurysm in the differential diagnosis of thunderclap headache can also be detected by CT angiography or magnetic resonance angiography, including arterial dissection and reversible cerebral vasoconstriction syndrome. Magnetic resonance venography may be necessary in a patient with thunderclap headache, but only when cerebral venous sinus thrombosis is part of the differential diagnosis. Cerebral venous sinus thrombosis is more common in young adults. Major risk factors for such thrombosis in adults include conditions that predispose to spontaneous thromboses, including inherited or acquired thrombophilia, pregnancy, oral contraceptive use, malignancy, sepsis, and head trauma. An MRI is as sensitive as a CT scan in the acute phase of subarachnoid hemorrhage but is not superior. Any subarachnoid blood not evident on a CT scan is unlikely to be seen on an MRI. A few days after the hemorrhage, an MRI is better for detecting blood, with fluid-attenuated inversion recovery (FLAIR) and T2-star images being the most sensitive. An MRI can also help in the detection of other conditions in the differential diagnosis, such as an acute ischemic stroke, pituitary apoplexy, a third-ventricle colloid cyst, and an acute hypertensive crisis.
question
Neuro 29 A 25-year-old woman comes to the office to ask about discontinuing her epilepsy medication because she is concerned about its potential long-term adverse effects. She has a 9-year history of generalized tonic-clonic seizures and rapid, shock-like body jerks consistent with myoclonic seizures on awakening. After her first seizure, she had an MRI with normal findings and an electroencephalogram showing generalized polyspike and wave abnormalities. She was started on lamotrigine and has done relatively well, having only rare seizures when she does not get adequate sleep or indulges in binge alcohol drinking. Her last generalized tonic-clonic seizure occurred 1 year ago. Results of physical examination, including neurologic examination, are normal. Which of the following is the best advice regarding discontinuation of the lamotrigine? A Begin a slow taper over the next 3 months B Continue life-long treatment C Discontinue after she is seizure free for 2 years D Discontinue now
answer
B Juvenile myoclonic epilepsy requires lifelong antiepileptic drug therapy. This patient requires life-long treatment of her epilepsy. Appropriate recognition of specific epilepsy syndromes is essential because it will guide the clinician in selecting the appropriate therapy, making the correct prognosis, and, in some instances, providing genetic counseling. A history of myoclonic and generalized tonic-clonic seizures on awakening with onset in adolescence is highly suggestive of the syndrome of juvenile myoclonic epilepsy, which is one of the most commonly encountered forms of epilepsy, possibly affecting 5% to 10% of all patients with epilepsy. Seizures in juvenile myoclonic epilepsy are generally well controlled with medication, but relapses can be provoked by sleep deprivation, alcohol, or exposure to flickering lights. If medication is withdrawn, seizures will recur in 75% to 100% of patients. Therefore, in this patient with a history of juvenile myoclonic epilepsy, medication withdrawal is not recommended either now, 3 months from now, or in 2 years; juvenile myoclonic epilepsy requires lifelong therapy.
question
Neuro 30 An 81-year-old man is evaluated for the gradual onset and progression of memory loss over the past year. He says he has difficulty recalling the names of familiar people, has misplaced his wallet on numerous occasions, and is slower to find his car in large, crowded parking lots. He continues to manage his finances, travel with his wife, and perform the activities of daily living without difficulty. He has borderline hyperlipidemia that is managed by diet alone. A paternal uncle developed Alzheimer dementia at age 74 years. His only medications are aspirin and a daily multivitamin. On physical examination, temperature is 36.7 °C (98.1 °F), blood pressure is 126/82 mm Hg, pulse rate is 68/min, respiration rate is 14/min, and BMI is 26. His level of alertness, speech, and gait are normal. He scores 26/30 on the Folstein Mini-Mental State Examination, losing all three points on the recall portion and one point on the orientation section for incorrectly stating today's date. Results of a complete blood count, serum vitamin B12 measurement, thyroid function tests, and a basic metabolic panel are normal. An MRI of the brain without contrast shows no abnormalities. Which of the following is the most likely diagnosis at this time? A Alzheimer dementia B Dementia with Lewy bodies C Frontotemporal dementia D Mild cognitive impairment E Vascular dementia
answer
D This patient has mild cognitive impairment (MCI), which denotes abnormal cognitive decline that is not severe enough to produce disability. His self-reported memory loss, which is confirmed by his performance on the Folstein Mini-Mental State Examination, is his only symptom; there are no other signs of dementia. Memory loss is nonspecific and is part of many dementia syndromes. However, the lack of any functional impairment in this patient makes MCI the most likely diagnosis at this time. Although there are no universally accepted criteria for MCI, the disorder has been defined as a memory abnormality corroborated by objective memory impairment on standardized tests, without general cognitive impairment or an effect on functional independence. The rate of progression to dementia is approximately 10% to 15% per year. Alzheimer dementia is the most common cause of MCI involving memory loss. Because this patient has no functional disabilities and thus does not meet the criteria for frank dementia, Alzheimer dementia is an incorrect diagnosis at this point. He may eventually develop the disease, given that the conversion rate of MCI to dementia is roughly 10% to 15% per year and that, at autopsy, approximately 80% of patients originally diagnosed with MCI have Alzheimer dementia. Early-stage symptoms that are characteristic of frontotemporal dementia include changes in behavior and personality, such as increasing apathy, disinhibition, or perseverative (repetitive to an exceptional degree) fixations. This patient has exhibited no such changes. The onset of dementia with Lewy bodies could also be characterized by memory loss. Besides clearly not having dementia of any sort at this stage of his illness, this patient lacks any of the other symptoms of dementia with Lewy bodies, such as parkinsonism, visual hallucinations, psychomotor slowing, and dream enactment behavior. Typical manifestations of vascular dementia include psychomotor slowing, a stepwise progression, and a history of stroke, none of which pertains to this patient.
question
Neuro 31 A 31-year-old woman is evaluated for an 8-month history of diurnal fatigue. She has a 3-year history of relapsing-remitting multiple sclerosis (MS) treated with interferon. The fatigue, which is worse in the early afternoon, has been present since her last MS attack, from which she has otherwise recovered completely. The patient, who has two young children, reports no current problems with sleep or mood and no interferon-related side effects. She exercises regularly. She has a history of depression and hypothyroidism. Current medications are interferon beta-1a and levothyroxine. On physical examination, temperature is 36.9 °C (98.4 °F), blood pressure is 95/70 mm Hg, pulse rate is 76/min, and BMI is 23. Results of general physical and neurologic examinations are normal. Results of laboratory studies are also normal, including a complete blood count, erythrocyte sedimentation rate, liver chemistry studies, and serum folate, thyroid-stimulating hormone, and vitamin B12 levels. Which of the following is the most appropriate next step in treatment? A Amantadine administration B Corticosteroid therapy C Discontinuation of the interferon beta-1a D Vitamin B12
answer
A Adequate rest and regular physical exercise can reduce multiple sclerosis-related fatigue, but many patients require treatment with stimulants, such as amantadine. This patient should receive a course of amantadine. Fatigue is the most common symptom of multiple sclerosis (MS) but is also highly prevalent in the general population. The fact that the patient's fatigue began with a relapse strengthens its association with her MS, but evaluation for other causes is still required. Her review of systems, physical examination, and laboratory studies do not reveal another likely cause, and she is already sleeping and exercising regularly. Therefore, pharmacologic therapy can reasonably be offered. Amantadine has been shown in several small placebo-controlled studies to reduce MS-related fatigue. The mechanism is not clearly understood but is likely related to the drug's stimulant properties. Although a brief course of corticosteroids may provide a temporary boost of energy, this strategy is not a practical solution because the beneficial effects will by definition be brief. Longer-term use of corticosteroids is also not advised because of the high risk of adverse effects. Discontinuation of interferon beta-1a is unnecessary because the patient does not perceive any adverse effects from the drug. Flu-like symptoms and fatigue related to interferon beta-1a are usually temporally related to individual injections. Although fatigue can also be related to vitamin B12 deficiency, injection of the vitamin is not indicated for this patient because the patient's physical examination and normal laboratory values show that she is not vitamin B12 deficient.
question
Neuro 32 A 35-year-old woman is evaluated in the office for a 5-month history of right-hand numbness and tingling. She says that these symptoms involve the entire hand, seem to be worse when she drives or holds a book or newspaper, and have been awakening her at night. She reports no history of neck pain or hand weakness. Personal and family medical history is noncontributory, and she takes no medication. General physical examination reveals no abnormalities. Neurologic examination shows normal strength but sensory loss in the distribution of the median nerve in the right hand. Percussion elicits paresthesias at the wrist (Tinel sign). An electromyogram shows a mild right median neuropathy at the wrist, with no evidence of cervical radiculopathy. Which of the following is the most appropriate initial step in treatment? A Corticosteroid injection into the carpal tunnel B Gabapentin C Surgical referral for carpal tunnel release D Wrist splints
answer
D Wrist splinting is an effective, conservative treatment of carpal tunnel syndrome in patients with mild symptoms. This patient should be treated with wrist splints. She reports symptoms typical of carpal tunnel syndrome. The electromyogram (EMG) confirms the presence of a mild median neuropathy at the wrist. Given the mild symptoms and the presence of only mild changes on the EMG, a trial of conservative therapy is most appropriate. Conservative treatment approaches offer advantages over surgical treatment in terms of decreased short-term morbidity and surgical risk; also, the adverse effects of drug therapy may be avoided if nondrug modalities are used and prove to be beneficial. Conservative therapies include wrist splints, physical therapy, ergonomic adjustments, and corticosteroid injections. The role of occupational and recreational activities using the hands is highly variable, but ergonomic education and modification can be considered if thought to be relevant for individual patients. For this patient with mild signs and symptoms of carpal tunnel syndrome, a trial of conservative therapy using wrist splints is indicated. A recently reported randomized controlled trial, which identified workers through a carpal tunnel syndrome surveillance protocol, assigned them to either the control group, who received ergonomic education, or the treatment group, who participated in a 6-week nocturnal splinting trial. Participants in the splinting group showed greater improvement than the control group, and this benefit was still present at 1 year. The initial response rate to corticosteroid injections at or near the wrist is 70%. The duration of response is quite variable, however, and patients often require repeat injections or, ultimately, carpal tunnel surgery. Patients with mild to moderate symptoms who do not respond to wrist splinting should be considered for corticosteroid injection. Gabapentin and other medications used to treat neuropathic pain are typically not indicated in carpal tunnel syndrome. Carpal tunnel surgery should be considered in patients who have not responded to conservative therapy and in patients with moderate to severe signs or symptoms, unless there is a contraindication to surgery. A recent systematic review of the different surgical treatment options for carpal tunnel syndrome failed to reveal improved outcomes in open versus endoscopic carpal tunnel release.
question
Neuro 33 A 28-year-old woman in her first trimester of pregnancy is evaluated in the office for severe and frequent migraine attacks occurring up to four times per week. These attacks are associated with severe pain, nausea, and emesis and last up to 24 hours; acetaminophen and NSAIDs do not relieve her symptoms. The patient has a 10-year history of migraine. Results of physical examination, including a neurologic examination, are normal. Which of the following is the most appropriate treatment for this patient? A Amitriptyline at bedtime B Ergotamine tartrate as needed C Metoclopramide as needed D Valproic acid (extended release) twice daily E Verapamil three times daily
answer
C Migraines during pregnancy that do not respond to simple analgesia can be safely and effectively treated with metoclopramide. Metoclopramide has been shown to have efficacy for both the pain and nausea associated with migraine. This drug has a U.S. Food and Drug Administration (FDA) rating of pregnancy category B. Acute medications with an FDA rating of pregnancy category A or B carry the lowest risk of harm to the fetus and should be used preferentially for the treatment of migraine during pregnancy. Amitriptyline and verapamil are used for migraine prophylaxis. Both have an FDA rating of pregnancy category C (risk to the fetus cannot be ruled out) and are not contraindicated for pregnant women. However, prophylactic migraine therapy should be avoided whenever possibly during pregnancy, especially in the first trimester. Often, migraine improves during the second and third trimesters of pregnancy, so acute therapy during the first trimester is preferred. If migraine continues to be disabling and occurs frequently during the second and third trimesters, prophylactic therapy could be considered, especially if the health of the mother and/or the fetus is compromised. Ergotamine tartrate and valproic acid have an FDA rating of pregnancy category X (studies have shown fetal abnormalities) and are contraindicated during pregnancy.
question
Neuro 34 A 65-year-old man is evaluated for possible epilepsy. He reports three similar "spells" over the past month of which he recalls a feeling of déjà vu followed by a loss of awareness. He says that family members who witnessed these episodes have described him as being unresponsive during them, recounting that he stares and repetitively smacks his lips for 1 minute, followed by a few minutes of confusion and garbled speech before he returns to normal awareness. He has no significant personal or family medical history and takes no medications. Physical examination, including neurologic examination, reveals no abnormalities. Which of the following is the most likely diagnosis? A Absence seizure B Generalized tonic-clonic seizure C Myoclonic seizure D Partial complex seizure E Simple partial seizure
answer
D Epilepsy that presents in adulthood is usually partial in onset; partial seizures that involve altered awareness are classified as complex partial seizures. This patient's "spells" were most likely partial complex epileptic seizures. The incidence of epilepsy is greatest in infants and older adults, with half of new-onset seizures presenting in persons older than 65 years. In large part, the latter fact reflects the increased prevalence of underlying brain diseases—particularly cerebrovascular disease, neurodegenerative conditions (such as Alzheimer disease), and brain tumors— that increase seizure risk in this segment of the population. Epilepsy is classified as partial or generalized on the basis of the areas of the brain involved at onset. Seizures that present in adulthood are usually partial in onset; partial seizures in which the patient maintains full awareness are classified as simple partial, whereas those involving an alteration of consciousness, as in this patient, are classified as partial complex. The most common site of onset for partial seizures is the temporal lobe. As described by this patient, temporal lobe events often begin with an aura of déjà vu, a rising epigastric sensation, or autonomic disturbances. Although automatisms, such as lip smacking, are occasionally reported in absence seizures, these features are atypical for that type of seizure and are more suggestive of partial complex ones. In this patient, the age of onset, duration (several minutes) of the event, presence of oral automatisms, and reported postictal confusion and speech impairment all suggest a partial complex seizure. Absence, generalized tonic-clonic, and myoclonic seizures are all manifestations of generalized epilepsy and are correlated with generalized epileptiform discharges on an electroencephalogram. Absence seizures are characterized by brief periods of staring and unresponsiveness, typically lasting seconds, with an immediate return to normal awareness. Absence seizures usually present in childhood. The length of this patient's episodes, his confusion before returning to normal awareness, and his age argue against absence seizure being the diagnosis. Generalized tonic-clonic (grand mal) seizures are characterized by stiffening of the trunk and extremities followed by generalized symmetric jerking. Such features were not reported in this patient. Myoclonic seizures consist of brief, shock-like muscle jerks without loss of awareness. These features are not consistent with the history this patient provides.
question
Neuro 35 A 53-year-old woman is evaluated in the office for a 4-month history of tremor. The tremor affects both upper extremities and is present "most of the time." She has a 15-year history of type 2 diabetes mellitus; she also has a history of hypertension, gastroparesis, and chronic kidney disease. Medications are insulin glargine, insulin lispro, lisinopril, hydrochlorothiazide, and metoclopramide. On examination, she has diminished pedal pulses. Speech, language, and mental status are normal. Cranial nerve function is normal, although a paucity of facial expression is noted. Movements are slow, and there is mild bilateral upper and lower extremity rigidity. Deep tendon reflexes are normal, as are results of manual muscle strength testing. Sensory examination reveals distal sensory loss. She had a mildly stooped posture but no postural instability. A 4-Hz resting tremor in both upper extremities is noted, as is a prominent postural tremor. Which of the following is the most likely diagnosis? A Dementia with Lewy bodies B Drug-induced parkinsonism C Multiple system atrophy D Parkinson disease
answer
B Drug-induced parkinsonism is a potential complication of dopamine-blocking medications, including metoclopramide. Drug-induced parkinsonism is the most likely diagnosis in this patient. This disorder has classically been associated with neuroleptic medications but can occur with any dopamine-blocking medications, including metoclopramide. Although metoclopramide causes drug-induced parkinsonism in one third of all patients using it, the disorder is particularly underdiagnosed in such patients. Establishing a diagnosis of drug-induced parkinsonism is critical because stopping dopamine-blocking medications can reverse or improve parkinsonian features in these patients. Cognitive impairment in conjunction with parkinsonism occurs in patients with dementia with Lewy bodies, but the latter diagnosis is unlikely in this patient, given her apparently normal cognition. Multiple system atrophy is a heterogeneous, progressive, and ultimately fatal neurodegenerative disorder associated with parkinsonian features and with cerebellar and autonomic signs and symptoms of variable severity. Early multiple system atrophy would be a consideration in this patient if she were not on a medication known to induce signs and symptoms of parkinsonism. The parkinsonism in some patients with early multiple system atrophy cannot be distinguished from Parkinson disease and may even be responsive initially to levodopa. Most patients with multiple system atrophy, however, have bilateral parkinsonian signs and lack significant tremor, findings that are atypical of Parkinson disease. Parkinson disease should be part of the differential diagnosis but is an unlikely cause of the symptoms in this patient. Although there are parkinsonian signs and symptoms, there are several atypical features that should prompt consideration of an alternative diagnosis. The presence of symmetric signs and symptoms (tremor and rigidity) and the postural tremor in this patient suggest a condition other than Parkinson disease. Other features that suggest an alternative condition in patients with parkinsonian signs and symptoms include early falls, rapid progression, poor or waning levodopa response, dementia, early autonomic failure, and ataxia. Moreover, this patient is taking metoclopramide, a medication known to cause parkinsonism.
question
Neuro 36 An 81-year-old woman is evaluated in the office for increasing difficulty with activities of daily living, including dressing and feeding herself, over the past 6 months. She has had gradually progressive cognitive decline for the past 5 years and now needs 24-hour help from a caregiver; Alzheimer dementia was previously diagnosed. The patient also has hypertension and hypothyroidism. Her mother had onset of Alzheimer dementia at age 80 years and died at age 87 years. Current medications are donepezil, 10 mg/d; lisinopril, 5 mg/d; levothyroxine, 0.1 mg/d; and a daily multivitamin. On physical examination, temperature is 36.4 °C (97.5 °F), blood pressure is 120/78 mm Hg, pulse rate is 68/min, respiration rate is 14/min, and BMI is 24. Her level of alertness, speech, and gait are normal. The patient scores only 12/30 on the Folstein Mini-Mental State Examination. Results of a complete blood count, a basic metabolic panel, a serum vitamin B12 measurement, and thyroid function tests are normal. A CT scan of the head without contrast shows no evidence of tumor, hemorrhage, or infarction. Which of the following is the most appropriate next step in treatment? A Add memantine B Add quetiapine C Add sertraline D Increase the dosage of donepezil E Stop all medications
answer
A Memantine is a first-line agent for treatment of moderate to advanced Alzheimer dementia. This patient has Alzheimer dementia that is moderately advanced and now has difficulties with basic activities of daily living. The N-methyl-D-aspartate receptor antagonist memantine is the only drug approved by the U.S. Food and Drug Administration as first-line treatment of moderate to advanced Alzheimer dementia. Although evidence is limited, there is some suggestion that the stepped approach of adding memantine to a regimen that includes a cholinesterase inhibitor (such as donepezil) results in a modest additional benefit over substituting memantine for the cholinesterase inhibitor. Quetiapine is an antipsychotic medication and sertraline is an antidepressant agent. Although both drugs can be used in patients with Alzheimer dementia, their use is limited to the treatment of the behavioral symptoms of psychosis and depression, respectively, neither of which this patient has at this time. There is no approved indication for using donepezil beyond its recommended maximum dosage of 10 mg/d. In fact, data suggest that doing so is associated with increased adverse effects and not with increased efficacy. Discontinuing the donepezil taken by this patient without substituting another drug to manage her functional decline would not help slow or otherwise improve the course of her disease. Although the potential cognitive side effects of the other prescription (and nonprescription) medications taken by this patient must also be considered, lisinopril, levothyroxine, and daily multivitamins have no association with such effects.
question
Neuro 37 A 75-year-old man is evaluated in the emergency department for a 2-day history of impaired balance, left-leg weakness, and urinary urgency and a 3-day history of constant midthoracic pain with occasional shooting pain in the left thorax at approximately the T7 dermatome. There is no history of trauma. The patient has degenerative arthritis of the lumbosacral spine. He is a current smoker with a 50-pack-year smoking history. The patient takes occasional ibuprofen or acetaminophen for back pain. On physical examination, temperature is normal, blood pressure is 140/85 mm Hg, pulse rate is 80/min, and respiration rate is 14/min. Neurologic examination shows left lower extremity weakness, hyperreflexia, an extensor plantar response, bilateral sensory impairment below the T7 dermatome, and gait ataxia. Findings from mental status, cranial nerve, and upper extremity motor and sensory examinations are normal. Results of a complete blood count, coagulation panel, and erythrocyte sedimentation rate measurement are also normal. A radiograph of the chest shows a parenchymal lesion in the right lung apex. Which of the following is the most likely diagnosis? A Acute L5 disk herniation B Epidural abscess C Epidural hematoma D Epidural metastases
answer
D New and progressive symptoms referable to the spinal cord represent a neurologic emergency that should prompt evaluation for a compressive lesion. The most likely diagnosis is epidural spinal cord compression from metastatic cancer of the spine. The clinical feature of weakness accompanied by upper motoneuron signs (such as hyperreflexia and an extensor plantar response) and the detection of a sensory level localize the process to the thoracic spinal cord. The shooting thoracic pain is a localizing symptom that indicates involvement of the left T7 spinal nerve root. The patient is a long-standing smoker and has a lung mass suspicious for malignancy. Lung cancer is among the most common primary malignancies that metastasize to the spine. New and progressive symptoms referable to the spinal cord represent a neurologic emergency that should prompt evaluation for a compressive lesion. Clinical outcome, including ambulatory ability and mortality, depends on neurologic status at diagnosis. A lumbar disk herniation at the L5 level would typically cause low back pain and radicular pain (sciatica) in one lower extremity because most herniations are unilateral or asymmetric. The thoracic location of the pain and sensory level and the presence of upper motoneuron signs in this patient exclude that diagnosis. Other compressive lesions, such as an abscess or a hematoma, could result in the clinical symptoms and signs exhibited by this patient. However, epidural abscesses are usually accompanied by fever and an elevated erythrocyte sedimentation rate, both of which were absent in this patient. Likewise, epidural hematoma development is strongly associated with anticoagulant use; this patient was not taking any such medications, and he had normal results on a coagulation panel.
question
Neuro 38 A 68-year-old man is admitted to the intensive care unit because of an exacerbation of chronic obstructive pulmonary disease (COPD). His course becomes complicated over the next 2 days by pneumonia and acute kidney injury, and he requires noninvasive positive-pressure ventilation with bilevel positive airway pressure. After 10 days, his medical condition stabilizes, but profound weakness of the extremities is noted. Besides COPD, the patient has a history of hypertension, hypothyroidism, and hyperlipidemia. One week before admission, he was started as an outpatient on prednisone, 60 mg/d, for his COPD; other medications include piperacillin-tazobactam, metoprolol, levothyroxine, and simvastatin. On physical examination, blood pressure is 134/90 mm Hg, pulse rate is 90/min, and arterial oxygen saturation is 96% on nasal oxygen, 2 L/min. Neurologic examination shows profound symmetric weakness of bilateral upper and lower extremity muscles. Deep tendon reflexes and cranial nerve function are normal. Laboratory studies show an elevated serum creatinine level at 1.5 mg/dL (114.5 µmol/L). Serum levels of creatine kinase and electrolytes are normal, as are results of liver chemistry studies. Which of the following is the most appropriate therapy for this patient? A Increased dosage of prednisone B Intravenous administration of immune globulin C Physical and occupational therapy D Plasma exchange
answer
C Therapy for critical illness myopathy, a frequent complication in sepsis or multiorgan failure that can cause failure to wean from mechanical ventilation and limb weakness, involves physical and occupational rehabilitative therapies and avoidance of corticosteroids. This patient should have physical and occupational therapy. His history is quite typical of critical illness myopathy. Critical illness refers to the syndrome of sepsis and multiple organ failure. Critical illness myopathy or neuropathy is usually recognized in the intensive care unit, either in patients who have difficulty weaning from mechanical ventilation (up to 30% of patients) or in patients who have developed severe limb weakness during or after recovery from critical illness. Electrodiagnostic testing with electromyography should be considered in patients who develop generalized weakness or fail to wean from ventilation in this setting. Such testing can help exclude other conditions, such as neuromuscular junction disorders or polyradiculoneuropathy (Guillain-Barré syndrome), and confirm the diagnosis of critical illness myopathy or neuropathy. Physical and occupational therapy is ideally initiated immediately on recognition of this syndrome. Respiratory therapy is important to minimize the risk of superimposed pulmonary infection. Stretching and passive range-of-motion exercises help maintain joint mobility and prevent contractures. Later, as the patient improves, strengthening of upper and lower extremity muscle groups and training in activities of daily living can be performed. Most patients will transition into the inpatient rehabilitation unit once medically stable. Corticosteroids, such as prednisone, should be avoided, if possible, in patients with critical illness who develop peripheral neuropathy or myopathy during the course of their illness. Corticosteroids may play a role in the pathogenesis of critical illness myopathy, but myopathy can develop in patients who are critically ill even with no history of corticosteroid exposure. Treatment of critical illness polyneuropathy with intravenous immune globulin has been attempted, but this therapy has not been shown to be of benefit in this disease or in critical illness myopathy. Some attempts to target and interrupt the cascade of humoral factors associated with sepsis have been promising, but treatment of critical illness myopathy is supportive at this time. The primary, initial approach is directed toward the prevention and management of sepsis, the systemic inflammatory response syndrome, and multiple organ dysfunction. Aggressive treatment of infection, hypoxemia, and hypotension is critical. There is no evidence of a role for plasma exchange in the treatment of critical illness myopathy or neuropathy at this time.
question
Neuro 39 A 40-year-old woman is evaluated in the emergency department 30 minutes after having a 1-minute episode of involuntary jerking of the right hand that spread up the right arm and a subsequent 2-minute episode of loss of consciousness and witnessed generalized tonic-clonic seizure activity. She has a 1-month history of increasing confusion and low-grade headache. Two years ago, she was treated for cutaneous melanoma. On physical examination, the patient is awake and alert. Vital signs are normal. There are no signs of meningismus and no papilledema. She has mild right-sided facial droop and only antigravity strength in the right arm; strength is normal elsewhere. Laboratory studies, including a complete blood count, measurement of serum electrolyte and plasma glucose levels, and a urine toxicology screen, show no abnormalities. Which of the following is the most appropriate next diagnostic test for this patient? A CT of the head B Electroencephalography C Gadolinium-enhanced MRI of the brain D Lumbar puncture E Positron emission tomography
answer
C Contrast-enhanced MRI is the diagnostic modality of choice when brain metastasis is suspected. This patient should have a gadolinium-enhanced MRI. Her melanoma has most likely metastasized to her brain. Brain metastases are 10 times more common than primary brain tumors, and metastatic disease of the central nervous system (CNS) is estimated to occur in 20% to 40% of adults with cancer. The tumors most likely to metastasize to the CNS in adults are lung and breast carcinoma and melanoma. Presenting signs and symptoms vary according to the size and location of the metastatic lesion(s) and occur as a result of local mass effect or increased intracranial pressure; common symptoms include seizure, headache, behavioral changes, and subacute progressive focal neurologic deficits. A high level of clinical suspicion for CNS metastasis is therefore justified when new neurologic deficits develop in patients such as this one with a known primary tumor. Contrast-enhanced MRI is the diagnostic modality of choice when brain metastasis is suspected. MRI has a higher sensitivity and specificity than CT scanning and is also safer because MRI contrast does not cause nephrotoxicity, and allergies are extremely rare. Furthermore, normal findings on a CT scan do not exclude the presence of a brain tumor, so a follow-up MRI would be required. Despite her witnessed tonic-clonic seizure, electroencephalography is not appropriate as the next diagnostic step because brain imaging is more likely to reveal a potential underlying cause for the seizure. MRI is the study of choice to detect tumors, gliosis associated with encephalitis, head trauma, and stroke that may be causing seizures. Lumbar puncture is also inappropriate in this patient because seizures are almost never present as the sole manifestation of a CNS infection or subarachnoid hemorrhage; lumbar puncture is indicated only if there are other indicators of these conditions. CNS infection is indicated by the presence of fever or altered mental status. Patients with subarachnoid hemorrhage may also have seizures, but this type of hemorrhage is associated with severe headache, altered mental status, syncope, and neurologic deficits and usually will be identified by CT or MRI imaging. Positron emission tomography (PET) can be used to identify malignant tumors with high metabolic rates and may be helpful in planning the next diagnostic step (which lesion to biopsy) and in prognosis. However, the use of PET before the establishment of a brain tumor with MRI is premature and not indicated.
question
Neuro 40 A 48-year-old woman is evaluated in the office for memory loss of gradual onset and progression over the past year. She says she has difficulty recalling names of familiar people, has misplaced her glasses on numerous occasions, and is slower to find her car in large or crowded parking lots. The patient now requires help from her daughter to manage her finances and prepare large meals. She has no other problems or personal medical history, but several members of her family developed dementia between age 46 and 54 years, including her mother, maternal uncle, and maternal grandfather. Her only medication is a daily multivitamin. On physical examination, temperature is 36.7 °C (98.1 °F), blood pressure is 116/74 mm Hg, pulse rate is 72/min, respiration rate is 14/min, and BMI is 24. Her level of alertness, speech, and gait are normal. She scores 24/30 on the Folstein Mini-Mental State Examination, losing two points on the orientation section for misstating the date and year, one point in the serial calculation portion, and all three points on the recall portion. Results of a complete blood count, basic metabolic panel, serum vitamin B12 measurement, and thyroid function tests are normal. Genetic testing is positive for the presenilin-1 mutation. An MRI of the brain without contrast shows no abnormalities. Which of the following is the most likely diagnosis? A Autosomal recessive Parkinson disease with dementia B Creutzfeldt-Jakob disease C Early-onset familial Alzheimer dementia D Frontotemporal dementia E Vascular dementia
answer
C The presenilin-1 mutation is specific for early-onset familial Alzheimer dementia. This patient's personal and family medical history is striking for dementia at an unusually young age. Her maternal family history in particular is strongly suggestive of early-onset familial Alzheimer dementia, which typically has symptomatic onset in a person's forties or fifties. Although genetic testing is not routine in the evaluation of dementia, all known causes of early-onset familial Alzheimer dementia involve an autosomal dominant mutation. Therefore, genetic testing was appropriate for this patient and should be considered in all patients with early-onset dementia that seems to follow an autosomal dominant pattern of inheritance. The presenilin-1 mutation is specific for early-onset familial Alzheimer dementia. Although mutations of the amyloid precursor protein and presenilin-2 gene are other known autosomal dominant mutations that can cause the disease, the presenilin-1 mutation is by far the most common cause, and testing for this mutation is commercially available. Because the mutation has been identified in this patient, it is appropriate to offer genetic counseling to her daughter. Because the presenilin-1 mutation is specific for early-onset familial Alzheimer dementia and because no evidence of parkinsonism was found on examination, it is unlikely that this patient has autosomal recessive Parkinson disease with dementia. In addition to evidence of parkinsonism, mutations in the parkin, α-synuclein, LRRK2, and other genes are associated with this disorder. The autosomal dominant mutations seen in early-onset familial Alzheimer dementia are not. Besides the specificity of presenilin-1 for Alzheimer dementia, this patient is also unlikely to have Creutzfeldt-Jakob disease or vascular dementia because characteristic MRI findings in those disorders are absent. Specifically, there is no "cortical ribbon" sign typical of Creutzfeldt-Jakob disease and no sign of previous cerebral infarction, which is seen in most patients with vascular dementia. No details are provided that suggest a frontotemporal pattern of dementia, although patients with Alzheimer dementia may occasionally exhibit such a pattern. Frontotemporal dementia and Alzheimer dementia are thus sometimes difficult to distinguish on clinical grounds alone. Even in the presence of a frontotemporal pattern of cognitive deficits, however, a presenilin-1 mutation is specific for early-onset familial Alzheimer dementia.
question
Neuro 41 A 79-year-old woman is to be transferred from the emergency department to a hospital ward for ongoing care. She awoke at home 5 hours ago with slurred speech, difficulty swallowing food and drink, and left hemiparesis. A right hemispheric ischemic stroke was diagnosed in the emergency department after a CT scan of the head confirmed a right hemispheric infarction. Because the time of stroke onset could not be determined, no recombinant tissue plasminogen activator was administered. The patient has no other medical problems and takes no medications. On physical examination, blood pressure is 168/86 mm Hg, pulse rate is 80/min, and respiration rate is 18/min. Neurologic assessment reveals dysarthria, dysphagia, left facial droop, and left hemiparesis. Laboratory studies show a plasma LDL cholesterol level of 158 mg/dL (4.09 mmol/L) but no other abnormalities. Which of the following is the most appropriate first step in management after transfer is completed? A Bedside screening for dysphagia B Oral administration of an angiotensin-converting enzyme inhibitor C Oral administration of a statin D Physical therapy and rehabilitation consultation
answer
A In a patient with stroke, dysphagia screening should be performed before food, oral medication, or liquids are administered. On admission to a hospital ward, a patient with stroke should be given nothing by mouth (kept NPO) until a swallowing assessment is conducted. Dysphagia screening is especially appropriate for this patient, who had difficulty swallowing when she first awoke with stroke symptoms. Dysphagia occurs in 45% of all hospitalized patients with stroke and can lead to poor outcomes, including aspiration pneumonia and death. Bedside screening of swallowing ability should thus be completed before oral intake of any medication or food; if the screening results are abnormal, a complete examination of swallowing ability is recommended. The American Heart Association/American Stroke Association recommends a water swallow test performed at the bedside by a trained observer as the best bedside predictor of aspiration. A prospective study of the water swallow test demonstrated a significantly decreased risk of aspiration pneumonia of 2.4% versus 5.4% in patients who were not screened. Angiotensin-converting enzyme inhibitors, statins, and aspirin are appropriate treatments for secondary stroke prevention in some patients, but they should not be orally administered before ruling out the risk of aspiration. Like most patients with stroke, this patient will undoubtedly require physical therapy and rehabilitation during her recovery. However, consulting with the department(s) responsible for such care is not an immediate concern and should not be the first step taken when the patient arrives in the hospital ward.
question
Neuro 42 A 50-year-old man is evaluated for a 12-year history of slowly progressive left leg weakness and trouble ambulating. There is no history of transient neurologic symptoms. He has a history of hypertension, coronary artery disease, and chronic low back pain. Current medications are sublingual nitroglycerin, atenolol, aspirin, and occasional NSAIDs. On physical examination, vital signs are normal. The patient has moderately severe spastic paraparesis that is worse on the left, with prominent circumduction of the left leg during ambulation. He requires a cane to ambulate 100 meters. Cerebrospinal fluid analysis reveals the presence of oligoclonal bands. MRIs of the brain and spine show lesions consistent with chronic multiple sclerosis. Which of the following is the most appropriate treatment for this patient? A Glatiramer acetate B Interferon beta-1a C Natalizumab D Physical therapy
answer
D No therapies have convincing effects on the neurodegenerative processes that underlie progressive forms of multiple sclerosis. This patient has primary progressive multiple sclerosis (MS); no treatments have been shown to affect the disease course. Therefore, physiatry consultation for evaluation and treatment of his spasticity and back pain are most appropriate at this time. His gradually worsening neurologic function is very likely due to a degenerative loss of axons in the central nervous system; active inflammation, the hallmark of clinical relapses, plays a much less significant role in primary progressive MS than in the relapsing-remitting type. This patient's function and pain could likely be improved substantially with a focus on symptomatic therapies, beginning with physiatry consultation for evaluation and treatment of his spasticity, impaired mobility, and musculoskeletal pain. Symptomatic therapy in patients with multiple sclerosis can have a marked beneficial effect on patient comfort, function, and quality of life, even when further disease progression cannot be effectively stopped. Beta interferons, glatiramer acetate, and natalizumab, the currently approved MS therapies, are all most effective in altering the immunologic mechanisms that underlie relapses and thus are only appropriate for relapsing-remitting disease.
question
Neuro 43 A 30-year-old man is evaluated in the office for an 8-month history of intensely painful headaches, which occur up to 10 times per day and last approximately 15 minutes each. The pain is most severe around the left eye, and he has no pain between attacks. Each attack is associated with rhinorrhea, lacrimation, and conjunctival injection. The patient has a 12-pack-year smoking history. He takes a combination of acetaminophen, caffeine, and aspirin, usually taking a total of five tablets daily. Results of a physical examination, including a neurologic examination, are normal. Which of the following is the most likely diagnosis? A Cluster headache B Medication overuse headache C Paroxysmal hemicrania D SUNCT syndrome (Short-lasting Unilateral Neuralgiform headache attacks with Conjunctival injection and Tearing)
answer
C The various trigeminal autonomic cephalalgias, characterized by pain referred to the first division of the trigeminal nerve and accompanying cranial autonomic symptoms, can be distinguished by the duration and frequency of each attack. This patient has paroxysmal hemicrania, one of the trigeminal autonomic cephalalgias. These cephalalgias are characterized by pain referred to the first division of the trigeminal nerve and cranial autonomic symptoms, including lacrimation and rhinorrhea. They differ from one another in the duration and frequency of attacks. A paroxysmal hemicrania attack has an intermediate duration (mean, 15 minutes) and an intermediate episode frequency (mean, 11 per day). Cluster headache, another trigeminal autonomic cephalalgia, closely resembles paroxysmal hemicrania except in the duration and frequency of attacks. Cluster headache is associated with one to eight attacks per day, and each attack has a mean duration of 60 minutes. Because this patient's headaches only last approximately 15 minutes each, cluster headache is unlikely. Although this patient overuses over-the-counter analgesics, he does not have a background of diffuse low-grade headache associated with medication overuse headache. The syndrome of Short-lasting Unilateral Neuralgiform headache attacks with Conjunctival injection and Tearing, or SUNCT syndrome, is another trigeminal autonomic cephalalgia with a very similar phenotype to paroxysmal hemicrania. However, SUNCT syndrome is usually associated with more than 30 attacks per day and a duration of 30 to 120 seconds for each attack. These parameters do not match this patient's symptoms. It is important to distinguish between the trigeminal autonomic cephalalgias because they are treated differently. Verapamil, indomethacin, and lamotrigine are, respectively, the treatments of first choice for cluster headache, paroxysmal hemicrania, and SUNCT syndrome.
question
Neuro 44 A 62-year-old woman is evaluated for a 1-year history of tremor that affects both upper extremities. She says that her handwriting has become sloppier since she first noticed the tremor and that she occasionally spills her morning coffee because of it. Although she feels otherwise healthy, she is concerned that she may have Parkinson disease. The patient has a history of hyperlipidemia controlled by diet and exercise but is otherwise healthy. Her mother, who died at age 79 years, had a similar tremor. Her only medication is a daily multivitamin. On examination, she has a mild tremor in the upper extremities that is present with the arms extended and during finger-to-nose testing. No resting tremor is apparent. Muscle tone and gait and limb coordination are normal. Administration of which of the following drugs is the most appropriate treatment of this patient? A Carbidopa-levodopa B Clonazepam C Propranolol D Ropinirole
answer
C First-line medications used to treat essential tremor include propranolol, primidone, gabapentin, and topiramate. This patient should be treated with propranolol. She has a history and examination findings consistent with the presence of essential tremor. Essential tremor primarily occurs when a patient maintains a posture, such as when the hands are outstretched. Essential tremor also may be present during movement, particularly postural adjustments. Autosomal dominant transmission occurs in approximately half of patients with this condition. Essential tremor most commonly affects the upper extremities; however, the legs, head, trunk, face, and vocal cords may be involved. Up to 15% of patients with essential tremor have major disability associated with this condition. Progression of essential tremor is typically slow, with intermittent lengthy periods of stable symptoms. Features that may be predictive of a more severe essential tremor include a positive family history of tremor, longer tremor duration, voice tremor, and unilateral tremor onset. Alcoholic beverage consumption suppresses symptoms in most patients with this condition. Treatment options for essential tremor are often limited and frequently only partially effective. It has been estimated that 50% of patients with essential tremor have no response to medical treatment. First-line medications used to treat essential tremor include propranolol, primidone, gabapentin, and topiramate. Propranolol is typically the drug of choice in most patients with essential tremor because of its effectiveness, which has been established in multiple well-designed randomized clinical trials. Essential tremor is distinguished from Parkinson disease by its lack of parkinsonian features, such as rigidity, bradykinesia, postural instability, and resting tremor. Carbidopa-levodopa can be an appropriate choice to treat Parkinson disease but is not useful in the treatment of essential tremor. This tremor is typically postural and kinetic, with a frequency of 6 to 12 Hz. Clonazepam is considered a second-line medication in patients with refractory essential tremor and thus should not be used as the initial pharmacotherapy, given its questionable effectiveness in two randomized clinical studies and its frequent association with drowsiness. Ropinirole is a dopamine agonist medication used to treat Parkinson disease. Given the absence of any other signs of Parkinson disease, this medication would not be indicated in this patient.
question
Neuro 45 A 50-year-old man is admitted to the hospital after having two generalized tonic-clonic seizures in a 24-hour period. He has had seizures in the past, which were always attributed to alcohol withdrawal. Ten years ago, he was in a major motor vehicle collision that was related to alcohol intoxication. He has end-stage liver disease secondary to alcoholic cirrhosis but has been sober for 2 years and is awaiting a liver transplant. His kidney function is normal. His current medications include nadolol, spironolactone, and furosemide. On physical examination, the patient is awake and alert, is afebrile, and has a normal blood pressure and pulse rate. Neurologic examination findings are normal. The general physical examination reveals changes consistent with chronic liver disease, including jaundice and ascites. Laboratory studies show a serum creatinine level of 0.6 mg/dL (45.8 µmol/L) and no blood ethanol. Serum electrolyte levels are normal. An MRI of the brain shows an area of chronic encephalomalacia in the left frontotemporal head region, consistent with old trauma. An electroencephalogram shows left temporal sharp waves. Which of the following is the best treatment for this patient? A Levetiracetam B Oxcarbazepine C Phenytoin D Valproic acid
answer
A For patients who should avoid hepatically metabolized antiepileptic drugs, either because of drug interaction or underlying liver disease, levetiracetam, gabapentin, and pregabalin can be used. Given his clinical history, MRI findings, and electroencephalographic findings, this patient is likely to have epilepsy. Treatment of epilepsy in a patient with an underlying hepatic disease requires careful selection and management of the antiepileptic medication. Levetiracetam, gabapentin, and pregabalin are the preferred choices in patients with significant liver disease because they do not undergo significant hepatic metabolism and have low protein binding. Therefore, levetiracetam is most appropriate for this patient. For antiepileptic drugs that are hepatically metabolized or highly protein bound, alterations of hepatic enzymatic pathways and hypoalbuminemia can result in unexpected drug toxicity. For these reasons, oxcarbazepine, phenytoin, and valproic acid would be less favored options for this patient. Additionally, some antiepileptic drugs should be avoided because of their potential hepatoxicity, particularly valproic acid and felbamate. In patients who have undergone or are expected to undergo organ transplantation, it is particularly important to consider potential drug interactions that might alter the effectiveness of their immunosuppression regimen. Cytochrome P450 enzyme inducers, including phenytoin and oxcarbazepine, and inhibitors, including valproic acid, can be problematic in this population. Highly protein-bound drugs, such as phenytoin, are also best avoided when possible. Levetiracetam, gabapentin, and pregabalin are preferred for these patients because of the lack of significant drug interactions. Because these three medications are renally excreted, the dosage may need to be lowered in the presence of renal insufficiency. Dosing should be based on clinical response rather than the serum drug level because the therapeutic range for these agents is quite broad.
question
Neuro 46 A 74-year-old woman is admitted to the hospital after sustaining a severe left hemispheric ischemic stroke while alone at home. Her son found her collapsed in the living room when he went to visit her. The patient has hypertension for which she takes enalapril but no history of ischemic heart disease or heart failure. On physical examination, blood pressure is 190/105 mm Hg, pulse rate is 80/min, and respiration rate is 16/min. The patient has right hemiparesis, right facial droop, aphasia, and dysarthria. The remainder of the physical examination, including the cardiovascular examination, is normal. Results of laboratory studies, including serum creatinine level, are normal. A CT scan shows frank ischemic changes that occupy most of the left middle cerebral artery territory. An electrocardiogram and chest radiograph show normal findings. Which of the following is the most appropriate treatment of her hypertension at this time? A Intravenous labetalol B Intravenous nicardipine C Oral nifedipine D Withholding (Pausieren) of all antihypertensive medications
answer
D For uncomplicated ischemic strokes in patients without concurrent acute coronary artery disease or heart failure, antihypertensive medications should be withheld if the systolic blood pressure is less than 220 mm Hg or the diastolic blood pressure is less than 120 mm Hg, unless there are other manifestations of end-organ damage. For uncomplicated ischemic strokes in patients without concurrent acute coronary artery disease or heart failure, consensus exists that antihypertensive medications, such as intravenous labetalol or nicardipine, should be withheld if the systolic blood pressure is less than 220 mm Hg or the diastolic blood pressure is less than 120 mm Hg, unless there are other manifestations of end-organ damage. This patient's systolic and diastolic blood pressure levels are below these limits. Many such patients have spontaneous declines in blood pressure during the first 24 hours after stroke onset. Oral nifedipine is an inappropriate treatment for this patient not only because of its antihypertensive qualities, but also because of its route of administration. Given the severity of her stroke deficits, in particular the dysarthria, she should receive nothing by mouth until a swallowing evaluation is carried out because of the high risk of aspiration. Notably, the patient is not eligible for recombinant tissue plasminogen activator therapy because the time interval between now and her previous symptom-free state is unknown. Aspirin (160 to 325 mg/d) administered within 48 hours of stroke onset results in a small but significant reduction in the risk for recurrent stroke during the first 2 weeks after the stroke and improves outcome at 6 months. Therefore, aspirin is recommended as initial therapy for most patients with acute stroke. However, aspirin should not be administered for at least 24 hours after administration of thrombolytics.
question
Neuro 47 A 44-year-old woman is evaluated for a 4-month history of worsening gait and bilateral leg numbness. She underwent gastric bypass surgery 8 years ago but has no other relevant personal or family medical history. Her only medications are a daily multivitamin and vitamin B12. On physical examination, temperature is normal, blood pressure is 130/80 mm Hg, pulse rate is 90/min, respiration rate is 14/min, and BMI is 29. The patient has moderate gait ataxia, lower extremity spasticity, hyperreflexia, and bilateral extensor plantar responses. Strength is normal, but vibration and proprioceptive sensation is impaired in both feet. Results of laboratory studies show a mild normocytic anemia. Serum vitamin B12, vitamin E, methylmalonic acid, and homocysteine levels are all normal. Nerve conduction studies and an electromyogram reveal a moderate axonal peripheral neuropathy. MRIs of the cervical spinal cord and the thoracic spinal cord show no abnormalities. Which of the following is the most appropriate next diagnostic study? A Antinuclear antibody testing B Measurement of serum copper and zinc levels C MRI of the brain D Serum neuromyelitis optica (NMO)-IgG autoantibody test
answer
B Vitamin B12 and copper deficiencies are associated with malabsorption syndromes related to gastric bypass surgical procedures and can cause anemia and a syndrome of progressive myeloneuropathy. Serum copper and zinc levels should be measured in this patient. She has a progressive myeloneuropathy syndrome and mild anemia, both caused by copper deficiency. Gastric bypass surgery is a risk factor for this clinical syndrome, which can be caused by vitamin B12 deficiency, copper deficiency, or both. Given that her serum vitamin B12 level is normal, copper deficiency is the most likely cause. Overingestion of zinc can also impair copper absorption. Therefore, both serum copper and zinc levels should be evaluated. In patients with copper deficiency, the spinal cord MRI can be normal or demonstrate signal changes involving the dorsal columns. Antinuclear antibody testing is useful for detection of an inflammatory or a rheumatologic abnormality, but such a disorder is unlikely in the absence of any clinical symptoms or signs suggesting a systemic inflammatory disease. An MRI of the brain may be appropriate in some patients with myelopathy to investigate for multifocal diseases, such as multiple sclerosis (MS). This patient, however, has no cerebral symptoms or signs, and the clinical syndrome exhibited and electromyography results suggest a myeloneuropathy. Neuromyelitis optica (NMO) can be distinguished from MS by the NMO-IgG autoantibody test. Long considered an MS variant, NMO is now recognized as a distinct demyelinating disease with a predilection for the optic nerves and spinal cord. An MRI of the spinal cord typically shows extensive lesions in patients with NMO. This patient's normal results on spine MRI make the diagnosis of NMO unlikely and the NMO-IgG autoantibody test unnecessary.
question
Meirp 48 A 68-year-old man is evaluated for fatigue. He says that for the past 2 weeks, he has been awakening at approximately 2 AM with a left-sided tremor and left-sided stiffness. Parkinson disease was diagnosed 3 years ago after he noted a left-hand tremor and a change in his handwriting; examination at that time showed mild parkinsonian signs, and he was started on carbidopa-levodopa. He currently takes immediate-release carbidopa-levodopa, which results in near-resolution of his parkinsonian symptoms; he notes, however, that the medication wears off if too much time passes between doses. During this recent period of early-morning awakening, he has occasionally taken extra carbidopa-levodopa, which has allowed him to fall asleep again. Results of a general medical examination are normal. Neurologic examination reveals slurred speech and a paucity of facial expression. Deep tendon reflexes are normal, as are results of manual muscle strength testing and sensory examination. He has mild upper extremity rigidity that is greatest in the left arm and a very mild resting tremor of the left upper limb. No appendicular or truncal ataxia is noted. Which of the following should be added to this patient's drug regimen to treat his fatigue? A Clonazepam, before bedtime B Donepezil C Extended-release carbidopa-levodopa, before bedtime D Fluoxetine
answer
C Motor symptoms of Parkinson disease, such as tremor, rigidity and dystonia, can develop nocturnally and cause sleep disturbances; such symptoms may respond to extended-release carbidopa-levodopa therapy. This patient should take extended-release carbidopa-levodopa before bedtime. Patients with Parkinson disease develop both motor and nonmotor complications. One of the major nonmotor complications is fatigue, which occurs in roughly half of patients with Parkinson disease. The first step in evaluating fatigue in these patients is to ensure that their dosage of levodopa or a dopamine agonist is high enough to adequately treat their parkinsonian symptoms. Standing blood pressures should also be obtained in these patients, when symptomatic, to exclude orthostatic hypotension as the cause of fatigue. Sleep disorders are common in patients with Parkinson disease, and poor sleep is the likely cause of fatigue in this patient. The patient reports increasing stiffness and tremor at night, symptoms that reflect a "wearing-off" effect of levodopa. Taking extended-release carbidopa-levodopa immediately before bedtime should allow a more restful night's sleep and lessen his fatigue. The extended-release version of this medication has a longer duration of efficacy than the immediate-release version. Sleep disorders are also common in patients with Parkinson disease. Its motor manifestations (such as tremor, rigidity, and dystonia), as exhibited by this patient, can disrupt sleep. Restless legs syndrome, periodic limb movement disorder, rapid eye movement (REM) sleep behavior disorder, and obstructive sleep apnea are other sleep disorders that can all occur in patients with Parkinson disease and disrupt sleep. REM sleep behavior disorder is characterized by the lack of normal muscle atonia during REM sleep and may result in patients physically acting out their dreams, such that they may yell, punch, grab, shout, or even jump out of bed. REM sleep behavior disorder may precede other clinical signs of Parkinson disease. Although this disorder and periodic limb movements of sleep may respond to clonazepam, this drug is not indicated in this patient, who lacks a history of such movements. Donepezil is indicated in the treatment of patients with dementia and is therefore not appropriate for this patient who has no evidence of dementia. Psychiatric disorders can develop in up to 60% of patients with Parkinson disease. After dopaminergic therapy is optimized, most clinicians use selective serotonin reuptake inhibitors, such as fluoxetine, to treat depression in patients with Parkinson disease. These drugs and serotonin-norepinephrine reuptake inhibitors are first-line agents for those requiring drug treatment. Given the absence of any other symptoms of depression, fluoxetine is not indicated in this patient. Moreover, although depression is often associated with fatigue, this patient's fatigue is more likely the result of an inadequate dosage of medication.
question
Neuro 49 A 38-year-old woman is evaluated in the office for a 10-month history of increasingly frequent headache. The headache is often worse in the morning on awakening. She has recently started keeping a headache diary, which reveals episodes on approximately 25 days of each month. The headache varies from a near-daily bilateral frontal dull throbbing to a severe left hemicranial throbbing associated with nausea, photophobia, and phonophobia. The patient has a 20-year history of migraine without aura and a history of depression. Her mother also has a history of migraine and depression, and her sister has a history of migraine. The patient has been taking propranolol for 3 months; a mixed analgesic containing butalbital, caffeine, and acetaminophen for mild or moderate headache at least 3 days per week for 9 months; rizatriptan for severe headache at least 2 days per week for 4 months; and citalopram for 1 year. Rizatriptan has become increasingly ineffective over the past month. Physical examination findings, including neurologic examination findings, are normal. Which of the following is the most likely diagnosis for her current symptoms? A Chronic migraine B Chronic tension-type headache C Idiopathic intracranial hypertension D Medication overuse headache
answer
D Medication overuse headache is generally defined as a headache for more than 15 days per month and the use of acute headache medication on more than 10 days per month. This patient has medication overuse headache. She has a 20-year history of migraine but a 10-month history of chronic daily headache on more than 15 days per month. She has been using an acute headache medication (butalbital, caffeine, and acetaminophen) more than 10 days per month and a combination of this medication and rizatriptan on some of these days. These features define a medication overuse headache. Although the patient does have chronic migraine, her current symptoms most likely result from her overuse of acute medications and not from her long history of migraine. Medication overuse headache typically presents when or soon after a patient awakens, and the efficacy of migraine-specific therapy in patients with medication overuse headache is intermittent or poor. Furthermore, some of this patient's headaches lack the classic features of migraine, including a pounding, unilateral headache of approximately 1-day's duration associated with nausea and disability (taking to bed). Despite the patient's depression, her headaches are not fully characteristic of chronic tension-type headache, which is typically mild to moderate in severity, lasts from 30 minutes to 7 days, and is often described as a "band-like" constriction around the head. Tension-type headaches are not associated with nausea and vomiting, photophobia, or phonophobia. Idiopathic intracranial hypertension is a disturbance of increased intracranial pressure without evidence of intracranial disease, such as mass lesion, hydrocephalus, or venous sinus thrombosis. This disorder occurs most commonly in obese women of childbearing age but also may be associated with tetracycline therapy, oral contraceptive use, and hypervitaminosis A. Affected patients typically develop new onset of daily nonthrobbing headaches that may worsen with coughing and sneezing or in the supine position. Other clinical symptoms may include diplopia, transient episodes of monocular or binocular visual loss, and pulsatile tinnitus. Characteristic findings in patients with this condition are papilledema, an enlarged blind spot or visual field abnormalities, and possible sixth cranial nerve palsy. This patient's findings are not consistent with idiopathic intracranial hypertension.
question
Neuro 50 A 68-year-old man is admitted to the hospital for evaluation of a transient 15-minute episode of left facial droop, slurred speech, and left arm weakness. Three years ago, the patient had a radical neck dissection as treatment of head and neck cancer and subsequently had radiation therapy. He also has hypertension and hyperlipidemia and has a 40-pack-year smoking history. Family history is noncontributory. Current medications are lisinopril, atorvastatin, and aspirin. On physical examination, temperature is normal, blood pressure is 156/88 mm Hg, pulse rate is 88/min, and respiration rate is 14/min. A right carotid bruit is heard on auscultation. No abnormal findings are noted on neurologic examination. Results of laboratory studies are normal. An MRI of the brain reveals a small wedge-shaped, cortical diffusion-weighted positive region of signal change occupying the right hemisphere. A magnetic resonance angiogram of the neck shows 80% stenosis of the right internal carotid artery. In addition to aspirin therapy, which of the following is the most appropriate next step in treatment? A Carotid angioplasty and stenting B Carotid endarterectomy C External carotid to internal carotid artery bypass surgery D Intravenous administration of heparin
answer
A Carotid angioplasty and stenting should be used in patients with symptomatic severe (>70%) internal carotid artery stenosis who are not eligible for surgical treatment with carotid endarterectomy. This patient, who has had an ischemic stroke and has symptomatic severe internal carotid artery stenosis (>70% stenosis), should undergo carotid angioplasty and stenting. Although carotid endarterectomy is still considered the gold standard of surgical therapies for patients with such stenosis, it cannot be performed in those who have stenosis that is difficult to access surgically (above the C2 level), medical conditions that greatly increase the risk of surgery, or other specific conditions, such as radiation-induced stenosis or restenosis after carotid endarterectomy. For such patients, the less invasive combination of carotid angioplasty and stenting is preferable. The U.S. Food and Drug Administration has approved carotid angioplasty and stenting for patients with symptomatic severe carotid artery stenosis who are classified as high surgical risk or who have unfavorable anatomy that precludes a surgical approach. Other candidates for nonsurgical treatment of severe stenosis include patients with a history of radical neck surgery, spinal immobility, dissection, an ostial lesion below the clavicle, the presence of a tracheostomy stoma, and contralateral laryngeal nerve paralysis. External carotid to internal carotid artery bypass surgery was shown not to be effective for the surgical treatment of carotid artery stenosis, but this surgery may be considered for symptomatic carotid artery occlusion (that is, 100% occlusion). Studies of unfractionated and low-molecular-weight heparin have not shown any benefit for the vast majority of patients with acute ischemic stroke. Overall, the small reduction in recurrent ischemic stroke associated with anticoagulants is counterbalanced by an increase in hemorrhagic strokes. The American Heart Association and the American Academy of Neurology recommend that patients with ischemic stroke should not be treated with anticoagulation. Therefore, heparin administration is not appropriate for this patient.
question
Neuro 51 A 69-year-old woman is evaluated in the office for visual hallucinations that have occurred several times over the past 3 months. During the past year, she and others have noticed that she is walking more slowly, has a mildly stooped posture, and occasionally has saliva accumulating at the corner of her mouth. Over the past 3 years, she has had increasing difficulty organizing financial records, has forgotten appointments, and twice became lost in crowded but familiar parts of her town. According to her husband, who accompanied her to this appointment, her sleep over the past 10 years has been marked by semipurposeful and flailing limb movements and occasional clear speech, once taking the form of a speech to employees. Her mother had onset of Alzheimer dementia at age 77 years and died at age 82 years. Her only medication is a daily multivitamin. On physical examination, temperature is 36.6 °C (97.9 °F), blood pressure is 142/76 mm Hg supine and 110/68 mm Hg standing, pulse rate is 80/min supine and does not change with standing, respiration rate is 14/min, and BMI is 22. There is trace cogwheeling of the right upper limb. When the patient walks, her posture is slightly stooped, right arm swing is slightly diminished, and gait is slow without frank shuffling. Rapid alternating movements are mildly reduced in speed and amplitude in the right hand. She has mildly reduced facial expression and mildly reduced inflection of speech. She scores 27/30 on the Folstein Mini-Mental State Examination, losing one point for misstating the date and two points on the memory portion of the test. Her level of alertness is normal, and she has no tremor. Results of a complete blood count, basic metabolic panel, serum vitamin B12 measurement, and thyroid function tests are normal. An MRI of the brain without contrast shows no abnormalities. Which of the following is the most likely diagnosis? A Alzheimer dementia B Creutzfeldt-Jakob disease C Dementia with Lewy bodies D Frontotemporal dementia E Vascular dementia
answer
C Dementia with Lewy bodies is characterized by dream-enactment behavior, cognitive decline, parkinsonism, and visual hallucinations. This patient has symptoms of dream-enactment behavior, cognitive decline, parkinsonism, and visual hallucinations, which together are the hallmark features of dementia with Lewy bodies. Also characteristic of this type of dementia are the physical findings of orthostatic hypotension and features of mild parkinsonism, such as a reduced degree of facial expression (hypomimia), reduced arm swing, stooped posture, and mild cogwheeling. Alzheimer dementia is a common comorbidity in patients with dementia with Lewy bodies. However, this diagnosis clearly does not in itself sufficiently account for the additional features of parkinsonism, dream enactment behavior, visual hallucinations, and dysautonomia. Creutzfeldt-Jakob disease is a rapidly progressive dementia that is associated with early age at onset and prominent myoclonus and typically results in death within a year of onset. This patient's 10-year history of symptoms and absence of myoclonus rule out that diagnosis. Because the hallmark features of frontotemporal dementia, such as apathy, perseveration, hoarding, disinhibition, and other personality changes, are lacking in this patient, that diagnosis is unlikely. This patient has no cerebrovascular risk factors for or history of stroke. Additionally, her MRI has no features to suggest a vascular etiology of her symptoms. Therefore, vascular dementia is very unlikely.
question
Neuro 52 A 32-year-old woman is seen for a follow-up evaluation. She had a witnessed generalized tonic-clonic seizure 1 week ago and was evaluated in the emergency department, where results of physical examination, complete blood count, measurement of serum electrolyte levels, and urine toxicology screen were all normal. She is otherwise healthy, has no significant personal or family medical history, and takes no medications. Results of a repeat physical examination are also normal. In addition to electroencephalography, which of the following diagnostic tests should be performed next? A CT of the head B Lumbar puncture C MRI of the brain D Positron emission tomography
answer
C MRI is superior to CT for detection of epileptogenic lesions. For patients whose clinical history includes new onset of a seizure for which no obvious provocative cause is identified, the standard evaluation consists of electroencephalography and MRI of the brain. These tests will help not only to confirm the diagnosis, but also to predict the risk of future recurrence and rule out any underlying condition (such as a brain tumor) that might require treatment in itself. MRI has been shown to be clearly superior to CT in detecting potentially epileptogenic lesions. For example, in one retrospective case series of 117 patients with refractory epilepsy, MRI detected 95% of histologically identifiable lesions, whereas CT only detected 32%. CT is the imaging modality of choice for new-onset seizures only when there is a suspicion of an acute cerebral hemorrhage or when a contraindication to MRI is present. Neither condition pertains to this patient. For adults with new-onset seizures, lumbar puncture is generally only indicated when the patient's history or physical examination findings lead to a significant clinical suspicion of an underlying infection or inflammatory cause. Infection of the central nervous system is indicated by the presence of fever or altered mental status. Patients with a subarachnoid hemorrhage, for whom lumbar puncture is sometimes appropriate, may also have seizures, but such a hemorrhage is associated with severe headache, altered mental status, syncope, and neurologic deficits and usually will be identified on a CT scan or MRI. Given this patient's uneventful prior history and normal results on physical examination, lumbar puncture is inappropriate. Positron emission tomography is sometimes used as part of a surgical evaluation for medically refractory epilepsy. It does not presently have any role in the evaluation of new-onset seizures.
question
Neuro 53 A 50-year-old man is evaluated because of abnormal findings on a brain MRI. Two weeks ago, he was involved in a motor vehicle accident in which he sustained shoulder, upper back, and neck injuries and head trauma without loss of consciousness. Results of an MRI of the cervical spine were normal, but an MRI of the brain revealed white matter abnormalities. The patient has a 15-year history of hypertension but no personal or family history of neurologic disease. His only medication is quinapril. On physical examination, blood pressure is 155/90 mm Hg; all other vital signs are normal. Results of neurologic examination are also normal. Which of the following is the most appropriate next step in diagnosis? A Lumbar puncture B Repeat MRI in 3 months C Visual evoked potential studies D No further testing
answer
D Misinterpretation of nonspecific white matter lesions discovered on brain MRIs of patients without specific symptoms is a leading cause of the misdiagnosis of multiple sclerosis. This patient requires no further testing. He has MRI evidence of white matter lesions but no history or examination findings that suggest a diagnosis of multiple sclerosis (MS). Misinterpretation of white matter abnormalities incidentally discovered on the MRIs of patients with nonspecific symptoms is a leading cause of MS misdiagnosis. The lesions noted on this patient's MRI are not typical of those seen in MS; given his age and long-standing and incompletely controlled hypertension, they are most likely related to small-vessel cerebrovascular disease. MS lesions are typically larger, ovoid, and periventricular in location, as in the MRI shown . They sometimes enhance with gadolinium. The patient can be reassured that MS is extremely unlikely and counseled that he needs to improve his blood pressure control and be aware of other modifiable vascular risk factors. Lumbar puncture and visual evoked potential studies might be useful if MS were a serious clinical consideration but are unnecessary for this patient. There is currently no indication to repeat the MRI in 3 months for diagnostic purposes; additionally, findings will not influence management of his hypertension or other modifiable risk factors.
question
Neuro 54 A 56-year-old man is evaluated in the office for a 1-month history of intermittent weakness of the left foot, a 6-month history of progressive right arm weakness, and 1-year history of muscle cramps. He says he feels lately as if he is "catching" his foot on things when ambulating. He has noticed no shortness of breath, dysphagia, or other bulbar symptoms and reports no other pain, sensory loss, or bowel or bladder impairment. The patient is otherwise healthy, has no history of disease, and is unaware of any family history of neurologic disorders. He takes no medications. Results of a general medical examination are normal. Neurologic examination reveals normal speech, language, and mental status. His tongue appears atrophic with fasciculations. He has diffuse weakness and atrophy of the proximal muscles in the right arm; fasciculations are noted. Left arm strength and muscle bulk are normal. Moderate weakness of the distal muscles in the left leg is noted, with fasciculations present in both lower extremities. Deep tendon reflexes are brisk in the upper and lower limbs, and the plantar response is extensor bilaterally. Sensory examination reveals no abnormalities, and there is no appendicular ataxia. Laboratory studies show a serum creatine kinase level of 602 U/L. Results of a complete blood count; measurement of serum creatinine, electrolyte, and vitamin B12 levels; and liver chemistry studies are normal. A radiograph of the chest shows no abnormalities. Which of the following is the most likely diagnosis? A Amyotrophic lateral sclerosis B Cervical myelopathy C Chronic inflammatory demyelinating polyradiculoneuropathy D Primary lateral sclerosis
answer
A Both upper and lower motoneuron findings are typically present in amyotrophic lateral sclerosis, which helps distinguish this disorder from its mimickers, such as multifocal motor neuropathy, chronic inflammatory demyelinating polyradiculoneuropathy, and primary lateral sclerosis. Amyotrophic lateral sclerosis (ALS) is the most likely diagnosis in this patient. Patients with ALS typically have progressive, asymmetric, painless extremity or bulbar weakness on presentation. The absence of sensory loss and the lack of bowel or bladder impairment are also suggestive of ALS. The combination of upper motoneuron findings (hyperreflexia, extensor plantar response) and lower motoneuron signs (atrophy, fasciculations) seen on neurologic examination strongly suggests ALS, which is a fatal, neurodegenerative motoneuron disease that affects both the upper and lower motoneurons. The term motoneuron disease is used to describe the heterogeneous group of disorders affecting the upper motoneuron, the lower motoneuron, or both; for example, progressive muscular atrophy is a motoneuron disease that affects the lower motoneuron, and primary lateral sclerosis is a motoneuron disease that affects the upper motoneuron. It is unclear whether different motoneuron disorders are distinct disorders or reflect different manifestations of a single disease. What is established, however, is that ALS has the worst prognosis of them all, with a mean survival of 3 to 5 years. Cervical myelopathy should be considered in patients presenting with arm and leg weakness but is an unlikely cause of this patient's symptoms. The presence of tongue atrophy and fasciculations, absence of sensory loss, and lack of bowel or bladder impairment would not be typical of a cervical myelopathy. Nonetheless, MRI studies are indicated in patients with suspected ALS to exclude myelopathy. Chronic inflammatory demyelinating polyradiculoneuropathy is a treatable neurologic condition that causes weakness, sensory loss, and depressed deep tendon reflexes. The absence of sensory loss and the hyperreflexia seen in this patient are not typical of this disorder. Chronic inflammatory demyelinating polyradiculoneuropathy is an immune-mediated, inflammatory disorder that causes demyelination of peripheral nerves and nerve roots. An elevated cerebrospinal fluid protein level, characteristic findings on an electromyogram showing demyelination and conduction block, or a sural nerve biopsy can establish the diagnosis. Chronic inflammatory demyelinating polyradiculoneuropathy is a neurologic disorder that is expected to respond to immune-modulating therapy. Corticosteroids, intravenous immune globulin, plasma exchange, mycophenolate mofetil, and azathioprine have been the primary therapies used in its treatment. Primary lateral sclerosis is not the diagnosis in this patient, given the lower motoneuron findings of atrophy and fasciculations on examination. A motoneuron disorder affecting the upper motoneuron, primary lateral sclerosis causes slowly progressive weakness and spasticity. In affected patients, spasticity predominates over weakness and is typically symmetric and lower limb predominant. Later in the disease course, most patients develop pseudobulbar features, with dysarthria and emotional lability. Primary lateral sclerosis is a clinical diagnosis that can be established only by excluding other diseases. Spinal cord MRIs are particularly useful in this regard. Primary lateral sclerosis is a slowly progressive neurodegenerative disorder with a much better prognosis than ALS; long-term survival is expected. There are no medications approved by the U.S. Food and Drug Administration for the treatment of primary lateral sclerosis.
question
Neuro 55 A 32-year-old man is evaluated in the office for a 6-month history of excruciating headaches, which occur up to 12 times per day and last approximately 10 minutes each. He is pain free between attacks. The pain is centered around and behind the left eye, and each attack is associated with conjunctival injection, lacrimation, and rhinorrhea. The patient has an 8-pack-year smoking history. He takes a combination of acetaminophen, caffeine, and aspirin, usually taking a total of four to six tablets daily. Results of a physical examination, including a neurologic examination, are normal. Which of the following is the most appropriate treatment for this patient's condition? A Indomethacin B Lamotrigine C Prednisone D Verapamil
answer
A Indomethacin is the treatment of choice for paroxysmal hemicrania. This patient should receive indomethacin. He most likely has paroxysmal hemicrania, one of the trigeminal autonomic cephalalgias, which are characterized by pain referred to the first division of the trigeminal nerve and by accompanying cranial autonomic symptoms, including lacrimation and rhinorrhea. An attack of paroxysmal hemicrania has an intermediate duration (mean, 15 minutes) and an intermediate episodic frequency (mean, 11 per day). Treatment with indomethacin can immediately and completely resolve the headache. Usually, the response occurs within the first 48 hours after treatment is initiated. Indomethacin is not effective for treating any of the other trigeminal autonomic cephalalgias. Therefore, a positive response to the drug helps distinguish between paroxysmal hemicrania and the other trigeminal autonomic cephalalgias. Lamotrigine is the most effective drug for the treatment of the syndrome of Short-lasting Unilateral Neuralgiform headache attacks with Conjunctival injection and Tearing (SUNCT syndrome), another trigeminal autonomic cephalalgia. There is no evidence supporting its usefulness in the treatment of paroxysmal hemicrania. Prednisone and verapamil are commonly used treatments for cluster headache, yet another trigeminal autonomic cephalalgia. As with lamotrigine, there is no evidence supporting their usefulness in the treatment of paroxysmal hemicrania.
question
Neuro 56 A 72-year-old man with a 3-year history of progressive dementia is evaluated in the emergency department for increased weakness, difficulty arising from a seated position, gross unsteadiness and slowness of pace while walking, and a 1-week history of urinary incontinence. His wife reports that after a fall 3 weeks ago in which he sustained no head trauma, he started to become weaker and more confused and now cannot ambulate without holding onto someone or using a walker. Although the patient has exhibited progressive mild gait change with shuffling and reduced left-sided arm swing over the past 2 years, he has not previously required a gait aid. At an evaluation 2 months ago, he scored 19/30 on the Folstein Mini-Mental State Examination. The patient also has chronic atrial fibrillation. There is no relevant family medical history. Current medications are donepezil, memantine, atenolol, and warfarin. On physical examination, temperature is 36.5 °C (97.8 °F), blood pressure is 96/60 mm Hg, pulse rate is 84/min and irregular, respiration rate is 16/min, and BMI is 23. The patient attends to the examiner but cannot rise from a seated position without assistance. His posture is stooped and he walks slowly with a shuffling gait, is very unsteady, and requires the assistance of the examiner to walk without falling. The patient fatigues rapidly after walking a few feet. There are no cranial bruises or other signs of trauma. He now scores 5/30 on the Folstein Mini-Mental State Examination. Laboratory studies show an INR of 2.3. Results of a complete blood count, a basic metabolic panel, measurement of serum vitamin B12 and thyroid-stimulating hormone levels, and a urinalysis are normal. A chest radiograph shows clear lung fields. An electrocardiogram shows atrial fibrillation, with a heart rate of 84/min. Which of the following is the most appropriate next diagnostic test? A Awake and asleep electroencephalography B Cerebrospinal fluid analysis C CT of the head D Echocardiography
answer
C In elderly patients with chronic dementia who take warfarin, minor, even indirect head trauma can lead to bleeding and cause abrupt worsening of confusion. This patient should undergo CT of the head. His chronic course seems most compatible with a degeneratively based etiology, such as dementia with Lewy bodies. Although impaired cognition and gait are expected in this setting, the disease course tends to be slow and steady. The subacute deterioration affecting both cognitive and motor skills that this patient has exhibited most likely resulted from his fall. Although there was no witnessed direct head trauma, minor, even indirect head trauma is possible because he takes warfarin and thus is at increased risk for bleeding. The manner in which the patient has declined is itself nonspecific, and common medical causes of such decline, such as urinary tract infection, pneumonia, and some common metabolic disturbances, are unlikely. The possibility of greatest concern is some form of intracranial disturbance, specifically, a subdural hematoma. A CT scan of the head is thus the most reasonable test to perform because other tests have not revealed a more obvious cause for the patient's decline. The patient's course does not suggest a seizure, and so electroencephalography (EEG) is not appropriate as the next diagnostic test. If a subdural hematoma is diagnosed, the patient and family should be counseled that seizures may complicate his future clinical course and that EEG is now reasonable to evaluate any epileptiform symptoms that may arise. In the absence of concern about meningitis, cerebrospinal fluid analysis should not be considered until it is known whether there is increased intracranial pressure related to a subdural hematoma or other trauma-induced intracranial hemorrhage. There is an increased risk of brainstem herniation with lumbar puncture and of cardioembolic cerebral infarction from the reversal of anticoagulation needed before performing a lumbar puncture. Echocardiography would be reasonable if brain imaging disclosed a recent cerebral infarction as the cause of the decline. In the absence of such evidence, this test is inappropriate.
question
Neuro 57 A 45-year-old woman is admitted to the hospital with mild left hemiplegia, left hemineglect, and dysarthria. A CT scan of the head reveals a large right hemispheric infarction due to an occluded right middle cerebral artery. She has a history of anti-phospholipid antibody syndrome diagnosed 3 years ago after an episode of iliofemoral venous thrombosis. She was treated for 18 months with warfarin but elected to discontinue treatment. There is no relevant family history. Her only medication is aspirin. Forty-eight hours later, a progressive deterioration of her mental status is noted. The patient had previously stated that she wants everything possible done to prolong her life ("full code"). Examination now shows that she has severe left hemiplegia and that her head and eyes are deviated to the right. A repeat CT scan of the head reveals a large hypodense region occupying the entire right middle cerebral artery territory, a local mass effect, and a 7-mm midline shift from right to left. Which of the following is the most appropriate therapy at this time? A Decompressive hemicraniectomy B Intra-arterial thrombolysis C Intravenous heparin D Use of an endovascular mechanical clot-retrieval device
answer
A Decompressive surgery can be a life-saving intervention in a patient who develops malignant brain edema after a hemispheric stroke. This patient has malignant brain edema as a result of her ischemic stroke and should be treated with decompressive hemicraniectomy. Young patients with major infarctions affecting the cerebral hemisphere or cerebellum have a heightened risk of brain edema and increased intracranial pressure. Reducing any edema and close monitoring for signs of neurologic worsening, particularly during the first 3 to 5 days after the stroke when the edema maximizes, are recommended. Meta-analyses of randomized, controlled trials have shown that decompressive hemicraniectomy for malignant stroke reduces morbidity and mortality. Intra-arterial thrombolysis is an option for the treatment of selected patients who have had a major stroke within the past 6 hours due to occlusion of a major intracranial artery. Although this patient's stroke was caused by an occluded right middle cerebral artery, she is long past the time window for this acute stroke therapy. Delay is associated with an increased risk of hemorrhagic conversion and reduced symptomatic benefit. Long-term anticoagulation may play a role in the prevention of recurrent ischemic stroke in a patient with anti-phospholipid antibody syndrome. However, acute intravenous administration of heparin in this patient who has a large acute cerebral infarction is more likely to accelerate the possibility of hemorrhagic conversion of the infarction and thus to worsen her current clinical state. The endovascular mechanical clot-retrieval device has been used to extract thrombi from occluded intracranial arteries. However, guidelines recommend that such a device must be used within 8 hours of a stroke. This patient is now beyond that 8-hour time window.
question
Neuro 58 A 58-year-old man is evaluated for a 5-month history of gait impairment and falls and a 1-week history of difficulty swallowing. His wife, who accompanied him, says that she has noticed a change in his speech over the past few weeks. He has no significant medical history, and there is no known family history of neurologic problems. General physical examination findings are normal; no orthostatic decrease in blood pressure is noted. Neurologic examination shows normal orientation and memory. Speech is slow and mildly dysarthric, and he appears to have decreased facial expression. Cranial nerve examination shows an impairment in vertical gaze. There is evidence of moderate axial and mild bilateral appendicular rigidity. No tremor is noted. Muscle strength and deep tendon reflexes are normal, as are sensory examination findings. There is no appendicular ataxia. Gait is slow, and there is significant postural instability. Which of the following is the most likely diagnosis? A Amyotrophic lateral sclerosis B Corticobasal degeneration C Parkinson disease D Progressive supranuclear palsy
answer
D Early falls, symmetric bradykinesia and rigidity, and lack of a resting tremor or levodopa responsiveness characterize progressive supranuclear palsy and help distinguish it from Parkinson disease. This patient's history and examination findings are most consistent with a diagnosis of progressive supranuclear palsy. A sporadic, neurodegenerative disorder, progressive supranuclear palsy typically manifests as gait impairment and falls, slurred speech, and impaired swallowing. The presence of reduced facial expression, axial rigidity, and impairment of vertical eye movements on examination further suggests the diagnosis. Amyotrophic lateral sclerosis is not associated with parkinsonian signs such as rigidity and postural instability. Patients with bulbar-onset amyotrophic lateral sclerosis present with slurred speech (dysarthria) and swallowing dysfunction, later followed by the development of diffuse extremity weakness, atrophy, and fasciculations. Corticobasal degeneration is also a rare, sporadic, neurodegenerative disorder that can manifest as gait impairment, slurred speech, dystonia, or myoclonus. Parkinsonian signs are evident on examination and are characteristically asymmetric. Alien limb phenomena can occur, whereby an extremity will move independently of voluntary, conscious control. The lack of a resting tremor, the symmetric bradykinesia and rigidity, and the early falls distinguish this patient's condition from Parkinson disease. Patients with progressive supranuclear palsy do not respond favorably to levodopa therapy, a mainstay of Parkinson disease management.
question
Neuro 59 A 58-year-old man is evaluated in the emergency department after having a generalized tonic-clonic seizure 1 hour ago while sleeping; the seizure was witnessed by his wife. The patient has postictal lethargy and confusion. Stage IV non-small cell lung cancer characterized by a large, surgically nonresectable lesion in the right lung and by liver and pancreatic metastases was diagnosed 6 months earlier. He has been receiving chemotherapy for the past 5 months. On examination, the patient is afebrile; other vital signs are also normal. He is lethargic but arousable and is oriented to self but not to time or place. Mild weakness and incoordination of the right upper extremity are noted. Results of laboratory studies are normal. An MRI of the brain with contrast shows nine ring-enhancing lesions at the gray-white junction that involve both cerebral hemispheres and are consistent with metastases. The largest of these is in the left precentral gyrus and is associated with edema in the surrounding white matter. Which of the following is the best management option? A Brain biopsy B Palliative whole-brain radiation therapy and corticosteroid administration C Stereotactic radiosurgery of the left precentral gyrus D Surgical resection of accessible cerebral metastases
answer
B Patients with multiple cerebral metastases and/or advanced systemic disease are not candidates for neurosurgical resection and should instead receive palliative treatment. The recommended treatment for this patient who has multiple cerebral metastases is palliative whole-brain radiation therapy and corticosteroid administration. Brain metastases are a common complication of systemic cancer and are associated with a poor prognosis. Treatment decisions are based on the location and number of cerebral lesions, the severity of neurologic symptoms, and the extent and prognosis of the systemic cancer. The main goal of therapy is to improve neurologic deficits by reducing the volume of the space-occupying metastases and the surrounding edema and to prevent symptom progression. Treatment response is directly related to the time from diagnosis to radiation therapy. In the presence of a known primary tumor with a likelihood of metastasizing to the central nervous system, such as lung cancer, brain biopsy to confirm the metastatic nature of the lesions is not needed. Stereotactic radiosurgery and surgical resection are generally restricted to patients with a single metastatic lesion and with reasonably controlled systemic disease. Neurosurgery will occasionally be considered for palliation in patients with multiple metastases, but only if there is a single lesion causing immediately life-threatening or severely disabling symptoms. In this patient, the presenting symptoms of seizure and mild weakness would not justify such aggressive intervention; these symptoms would be better treated with whole-brain radiation therapy and corticosteroids.
question
Neuro 60 A 33-year-old man is evaluated for a 3-day history of worsening weakness and numbness of the right arm and leg. He has a 5-year history of multiple sclerosis. His only current medication is glatiramer acetate. On physical examination, temperature is 36.5 °C (97.7 °F), blood pressure is 105/75 mm Hg, pulse rate is 68/min, and respiration rate is 14/min. Moderate right arm and leg weakness, hyperreflexia, an extensor plantar response, and vibratory sense impairment are noted. Which of the following should this patient receive to treat his acute relapse? A Empiric antibiotic therapy B Immune globulin, intravenously C Methylprednisolone, intravenously D Plasmapheresis E Prednisone, orally
answer
C Multiple sclerosis relapses may resolve more rapidly with intravenous methylprednisolone therapy. Evidence from placebo-controlled trials involving multiple sclerosis (MS) and optic neuritis supports the intravenous use of methylprednisolone to speed recovery from acute MS relapses. The long-term outcome of an individual MS attack, however, is not affected by the therapy chosen. Therefore, in light of the potential adverse effects of such drugs, intravenous administration of corticosteroids should be offered only for relapses that result in substantial discomfort or reduced function. This patient's 3-day history of worsening weakness and numbness of the right arm and leg qualifies him for corticosteroid therapy. It is necessary to rule out infection as a cause of neurologic worsening in patients with MS because infections can cause a "pseudoexacerbation"; this patient, however, is asymptomatic except for his neurologic symptoms and is afebrile. Patients with moderate or severe MS-related disability are more susceptible to the effects of mild infections, especially urinary tract infections, and investigation for such infections is warranted, even in the absence of systemic symptoms. However, empiric antibiotic therapy to treat a possible occult infection is not warranted in the absence of evidence or a high clinical suspicion of infection. Intravenously administered immune globulin is an effective treatment for a number of immune system-mediated diseases, but evidence of its efficacy for MS is lacking. Clinical trials have found no benefit from immune globulin, either as add-on treatment to methylprednisolone for acute MS attacks or as monotherapy for acute optic neuritis. Plasma exchange therapy may be beneficial in the treatment of fulminant attacks of multiple sclerosis that are unresponsive to corticosteroids. Such therapy appears to be particularly useful in patients with severe attacks of neuromyelitis optica that do not improve with corticosteroids. However, plasmapheresis is not indicated for this patient at this time before a trial of methylprednisolone. Oral administration of prednisone was not as effective as intravenous administration of methylprednisolone in the Optic Neuritis Treatment Trial and so should not be the first choice to treat acute MS relapses.
question
Neuro 61 A 17-year-old male high school student is evaluated in the emergency department 1 hour after having a generalized tonic-clonic seizure while eating breakfast with his family. He says he was out late the night before with classmates and drank six cans of beer over the course of the evening. He reports having sudden, involuntary jerks of his arms this morning before the convulsion and having had similar jerks on awakening over the past 2 months when sleep deprived. He reports no history of regular alcohol or illicit substance abuse. He takes no medications. Physical examination and neurologic examination findings are normal. Results of laboratory studies (including a complete blood count; measurements of serum electrolyte, plasma glucose, and serum ethanol levels; and a urine toxicology screen) are normal. A CT scan of the head shows no abnormalities. Which of the following is the most likely diagnosis? A Alcohol withdrawal seizure B Benign rolandic epilepsy C Juvenile myoclonic epilepsy D Temporal lobe epilepsy
answer
C Juvenile myoclonic epilepsy is characterized by myoclonic and generalized tonic-clonic seizures on awakening that are often provoked by sleep deprivation or alcohol. This patient most likely has juvenile myoclonic epilepsy. Recognizing the specific epilepsy syndrome affecting a patient is crucial in selecting the appropriate therapy, making the correct prognosis, and, in some cases, providing genetic counseling. A history of myoclonic (rapid, unprovoked jerks) and generalized tonic-clonic seizures on awakening with onset in adolescence strongly suggests a diagnosis of juvenile myoclonic epilepsy. One of the most commonly encountered forms of epilepsy, juvenile myoclonic epilepsy may affect 5% to 10% of all patients with epilepsy. Seizures are often provoked by sleep deprivation, alcohol, or exposure to flickering lights. Alcohol withdrawal seizures develop in chronic users of alcohol and are generally seen in combination with other signs and symptoms of alcohol withdrawal, such as delirium, tremor, tachycardia, and diaphoresis. This patient's history does not suggest alcohol withdrawal as the likely cause of his recurrent seizures. Benign rolandic epilepsy is a syndrome seen in younger children and adolescents who have seizures, usually during sleep, that begin with focal sensory and/or motor symptoms involving the face, mouth, and throat that can then secondarily generalize. This benign syndrome is not associated with myoclonic seizures and so is not the diagnosis in this patient. Temporal lobe epilepsy is the most common of the localization-related epilepsies, a type of epilepsy resulting from a focal brain abnormality. Temporal lobe epilepsy can often be further categorized as the result of a focal abnormality of the mesial temporal lobe. The most common seizure classification associated with mesial temporal lobe epilepsy is complex partial seizure. Characteristically, patients with complex partial seizures are awake but exhibit altered awareness, such as unresponsiveness or staring. Patients also exhibit automatisms—such as lip smacking, swallowing, picking, or manipulating objects—or automatic (purposeless, repetitive) behaviors. Patients often describe a preceding aura and, most commonly, autonomic symptoms. About one third of complex partial seizures will generalize as tonic-clonic seizures. This patient's myoclonic jerking is not compatible with temporal lobe epilepsy.
question
Neuro 62 A 78-year-old man is re-evaluated after a 5-mm left middle cerebral artery aneurysm is discovered incidentally on an MRI of the brain obtained because of a previous symptom of extreme dizziness, which has since resolved. The patient has no other relevant personal or family medical history and takes no medications. On physical examination, blood pressure is 138/82 mm Hg, pulse rate is 80/min, and respiration rate is 18/min. Results of physical examination, including neurologic examination, are normal. A magnetic resonance angiogram (MRA) shows a solitary unruptured aneurysm but no additional intracranial aneurysms. No hemorrhage, infarction, mass, or mass effect is evident. Which of the following is the most appropriate next step in the management of this patient's aneurysm? A Annual MRA B Endovascular coiling of the aneurysm C Nimodipine administration D Surgical clipping of the aneurysm
answer
A Annual magnetic resonance or CT angiography to monitor aneurysmal growth is appropriate as management of a low-risk unruptured intracranial aneurysm. This patient should have an annual magnetic resonance angiogram (MRA) or CT angiogram to monitor aneurysmal growth. For patients without a prior subarachnoid hemorrhage, the lowest-risk aneurysms are those in the anterior circulation and less than 7 mm in diameter. The annual risk of rupture for an aneurysm of the size of this patient's is 0.05% annually. The risk of neurologic disability associated with intervention exceeds the potential benefit. After 3 successive years of annual monitoring, an MRA or CT angiogram obtained once every 3 years is sufficient. The second report from the International Study of Unruptured Intracranial Aneurysms was a prospective observational study of patients who either underwent open or endovascular repair of asymptomatic intracranial aneurysms. Open surgery was associated with surgery-related death or poor neurologic outcome of nearly 13% at 1 year, compared with approximately 10% for endovascular repair. Complication rates increased with increasing age (30% at age 70 years or older), aneurysm size, and location of the aneurysm in the posterior circulation. Because the complication rate of intervention is likely to exceed the complication rate of observation alone, neither clipping nor coiling is indicated. Nimodipine is administered routinely in patients with subarachnoid hemorrhage in order to prevent vasospasm. This drug is not appropriate for a patient with an unruptured intracranial aneurysm. It is generally recommended that patients should refrain from smoking, heavy alcohol consumption, amphetamines, cocaine, and excessive straining and the Valsalva maneuver when taking this drug.
question
Neuro 63 A 64-year-old woman is evaluated for a 1-year history of increasing difficulty finding the right word in conversation and completing sentences; she sometimes says the wrong word accidentally. Her family now has difficulty understanding her, and she no longer has any interest in speaking on the telephone. Her ability to drive, shop, pay bills, and cook seems unimpaired. She has no other relevant personal or family medical history. Her only medication is aspirin, 81 mg/d. On physical examination, temperature is 36.6 °C (97.9 °F), blood pressure is 122/78 mm Hg, pulse rate is 80/min, respiration rate is 14/min, and BMI is 23. The patient is right-handed. Her level of alertness is normal, and her comprehension appears to be intact, with her correctly executing the commands to show the right thumb and two fingers on the left hand. Spontaneous speech is effortful, and she talks in short, telegraphic sentences filled with many mispronunciations (such as "posital" for "hospital"). She makes similar errors when trying to write words rather than speak them, can repeat no more than two words or four numbers at a time, and can repeat essentially no sentences. Results of a complete blood count, a basic metabolic panel, a serum vitamin B12 measurement, and thyroid function tests are normal. An MRI of the brain without contrast shows mild atrophy but is otherwise unremarkable. This patient's impairment in speech and writing is most likely due to which of the following disorders? A Alzheimer dementia B Creutzfeldt-Jakob disease C Dementia with Lewy bodies D Frontotemporal lobar degeneration E Vascular dementia
answer
D In frontotemporal lobar degeneration, which encompasses the syndromes of progressive nonfluent aphasia, semantic dementia, and frontotemporal dementia, symptom onset is insidious and progression gradual over the course of several years. This patient has progressive nonfluent aphasia that is most likely due to frontotemporal lobar degeneration. Progressive nonfluent aphasia, semantic dementia, and frontotemporal dementia comprise the three main syndromes of frontotemporal lobar degeneration. Symptom onset is insidious and progression is gradual over the course of several years. Her early decline in social interpersonal conduct is typical of this disorder, as is her aspontaneity and economy of speech. Approximately 10% of patients with frontotemporal lobar degeneration, especially frontotemporal dementia, have concurrent motoneuron disease. The most prominent early symptom of Alzheimer dementia is memory impairment. Alzheimer dementia can also produce aphasia, although usually not a purely nonfluent form and not typically as an initial symptom. Other diagnoses should also be considered when the prominent early finding is a symptom other than recent memory impairment, such as impaired social behavior, gait difficulty, or hallucinations and delusions. Clinical suspicion for additional diagnoses also should be raised when the disease course is not insidious and chronically progressive. Finally, Alzheimer dementia typically begins at a later age than 64 years, an age that is more typical of frontotemporal lobar degeneration. Creutzfeldt-Jakob disease is a rapidly progressive dementia producing death, typically within a year of onset. Although the specific symptoms of this disorder are highly variable, this patient's gradual disease course does not resemble the more rapid one of Creutzfeldt-Jakob disease. This patient does not have two of the core clinical features (parkinsonism, fluctuations in cognition or level of alertness, and visual hallucinations) required for a diagnosis of dementia with Lewy bodies, and she is considerably younger than most patients with this disorder. As with Alzheimer dementia, aphasia can occur within the context of this dementia. Progressive nonfluent aphasia is often mistaken for a stroke because of the obvious speech impairment produced. With stroke, however, the aphasia would be of sudden onset, and the patient would likely have MRI evidence of cerebral infarction, both of which are lacking in this instance. Additionally, the patient has exhibited no other signs or symptoms of stroke and has no history of vascular disease. Therefore, vascular dementia is unlikely.
question
Neuro 64 A 28-year-old man is evaluated in the office for a 3-day history of blurred vision in the left eye and pain with eye movement. He has a history of rare migraine headaches but no other disorders. There is no pertinent family history. The patient takes no medications. On physical examination, temperature is 36.8 °C (98.2 °F), blood pressure is 105/65 mm Hg, pulse rate is 78/min, respiration rate is 12/min, and BMI is 22. A left central scotoma is noted on visual field testing. Corrected visual acuity is 20/50 in the right eye and 20/20 in the left; red desaturation and a relative afferent pupillary defect are noted in the left eye. Other findings of physical and neurologic examinations, including funduscopy, are normal. Results of laboratory studies are normal. An orbital MRI reveals enhancement of the left optic nerve with intravenous administration of gadolinium. An MRI of the brain shows six periventricular white matter lesions, each measuring 3 to 5 mm in diameter and one enhancing with gadolinium administration. Ophthalmologic consultation confirms that the clinical features are compatible with optic neuritis. Which of the following best describes this patient's approximate risk of developing multiple sclerosis over the next 10 to 15 years? A 10% B 20% C 50% D 90%
answer
D If the brain MRI of a patient with a first-ever event of symptomatic central nervous system inflammatory demyelination shows lesions consistent with demyelination, the risk of developing multiple sclerosis approaches 90% over the next 10 to 15 years. The results of a brain MRI in a patient with new-onset neurologic symptoms suggestive of demyelination provide the most powerful prognostic information about his or her risk of developing multiple sclerosis (MS). Symptomatic patients with as few as one MRI brain lesion compatible with demyelination on clinical presentation have a risk of up to 90% that they will experience a second clinical event over the next 10 to 15 years, thereby confirming MS. Conversely, if the brain MRI reveals no white matter lesions, their risk over the same time period is slightly less than 20%. The brain MRI typical of MS reveals ovoid lesions in the periventricular white matter that sometimes enhance with gadolinium. This patient has also experienced left optic neuritis. When this occurs as a first-ever event of symptomatic central nervous system inflammatory demyelination, it is referred to as a clinically isolated syndrome, a term that implies some risk of developing MS in the future. Other such clinically isolated syndromes include brain stem or spinal cord events, such as myelitis. Long-term follow-up studies of all patients presenting with optic neuritis show that only approximately 50% of them later develop MS. This risk, however, is increased over that of the general population. Coupled with the implications of the brain MRI in this patient, the finding of optic neuritis confirms his very high risk.
question
Neuro 65 A 40-year-old man is evaluated in the emergency department for ongoing seizure activity. His wife says that he was having a generalized tonic-clonic seizure when she awoke 45 minutes ago and that she called the paramedics when the seizure continued for more than 5 minutes; she also reports that he had three seizures yesterday, each consisting of staring and unresponsiveness with repetitive lip smacking and each lasting 2 minutes. The patient has a known history of partial epilepsy secondary to a cavernous malformation in the left temporal lobe and has an average of four seizures each year. He is on phenytoin therapy. On examination, he is comatose with continuous rhythmic twitching of the right face and arm. Blood pressure is 120/70 mm Hg, pulse rate is 80/min, and arterial oxygen saturation rate by pulse oximetry is 85% (on ambient air). The patient is intubated. Which of the following is the most appropriate next step in management? A CT of the head B Electroencephalography C Intravenous administration of lorazepam D Measurement of serum phenytoin level
answer
C Benzodiazepines are the first-line treatment for status epilepticus. This patient should receive lorazepam. He most likely has status epilepticus, a neurologic emergency defined as seizures that persist or recur without interval recovery for a period of 30 minutes or longer. Approximately 50% of patients with status epilepticus have an established diagnosis of epilepsy, and among these patients, nonadherence to medication is a common precipitating event. Convulsive status epilepticus has a mortality rate of approximately 20%. When generalized convulsive status epilepticus continues, clinical signs of ongoing electrical seizure activity will often become increasingly subtle and may only involve subtle twitching of the eyes, face, or limbs, as seen in this patient. However minimal, this continuing electrical seizure activity must be corrected to avoid potential morbidity and mortality. Persistent seizure activity results in acute systemic complications, including fever, hemodynamic instability, acidosis, rhabdomyolysis, and pulmonary edema, all of which must be carefully managed. Airway, breathing, and circulatory status should be assessed at presentation and monitored continuously. Early diagnosis and therapeutic intervention are critical because an increase in the duration of status epilepticus is directly correlated with increased mortality. Treatment should not be delayed for diagnostic testing. Intravenous administration of a benzodiazepine (lorazepam or diazepam) is accepted as first-line therapy and may be administered by emergency medical personnel in the field. A CT of the head may ultimately be necessary to evaluate for bleeding of the cavernous malformation. Although emergent head imaging is often useful in the absence of a known underlying cause of status epilepticus, it should never delay treatment with a benzodiazepine. Because this patient's symptoms are quite consistent with status epilepticus, confirming the diagnosis with an electroencephalogram (EEG) and thus delaying treatment is unnecessary. In patients who are unresponsive or somnolent after status epilepticus, continuous EEG monitoring is strongly advocated to distinguish between ongoing nonconvulsive status and postictal states. Intravenous administration of phenytoin or phenobarbital is used as second-line therapy for status epilepticus not responsive to benzodiazepines. If administration of these agents is planned, it should not be delayed while awaiting results of serum drug levels.
question
Neuro 66 A 55-year-old man is evaluated in the postanesthesia care unit for an inability to move his right arm and leg and a marked delay in his ability to obey commands and instructions. Two hours ago, he underwent femoropopliteal bypass surgery because of severe lower extremity peripheral artery disease. The neurologic deficits were first noticed 15 minutes ago, immediately after extubation. In addition to peripheral artery disease, the patient has a history of hypertension, hyperlipidemia, coronary artery disease, and type 2 diabetes mellitus. Medications include aspirin, metoprolol, lisinopril, simvastatin, and metformin. On physical examination, blood pressure is 162/88 mm Hg, pulse rate is 80/min, respiration rate is 18/min, and BMI is 31. Pupils are 3 mm in diameter, symmetric, and reactive to light. He is aphasic, has right hemiparesis, and has a right extensor plantar response. An immediately obtained CT scan of the head shows a subtle hyperdensity of the left middle cerebral artery in the sylvian fissure but no other obvious abnormalities. Which of the following is the most appropriate next step in management? A Administration of naloxone B Intra-arterial clot extraction C Intravenous administration of recombinant tissue plasminogen activator D MRI of the brain
answer
B Recombinant tissue plasminogen activator as treatment of acute stroke is contraindicated in patients who have had major surgery in the past 14 days. This patient should undergo intra-arterial clot extraction. His presentation is not merely that of someone just emerging from anesthesia or still exhibiting the effects of narcotic analgesia. Rather, the patient is behaving clinically as if he sustained an intraoperative or postoperative stroke. A CT scan of the head obtained so soon after symptom onset will not yet reveal ischemic changes, although it will exclude hemorrhage as a cause of his neurologic symptoms. His CT scan strongly supports the presence of an occluded left middle cerebral artery. The ideal acute treatment for a perioperative ischemic stroke in this patient, whose recent surgery makes him ineligible for recombinant tissue plasminogen activator (rtPA), is consultation with an endovascular neurosurgeon for intra-arterial clot extraction with a device approved by the U.S. Food and Drug Administration. As with the intra-arterial administration of thrombolytics, the use of these devices is limited to comprehensive stroke centers that have the expertise to perform these procedures safely. Although the clot extraction device is recognized by American Heart Association/American Stroke Association guidelines as a reasonable intervention for extraction of intra-arterial thrombi in carefully selected patients for the management of acute stroke, the expert panel also recognizes that the utility of the device in improving outcomes after stroke is unclear. Although the narcotic analgesia he received during surgery may be contributing to this patient's small pupils, it is not the probable explanation for his hemiparesis. Therefore, administration of naloxone or any other opioid antagonist is not appropriate management. The patient, who was last known to be neurologically intact 2 hours ago at the start of the operation, awoke from anesthesia with the neurologic deficit. Although he is within the traditional 3-hour time window for intravenous administration of recombinant tissue plasminogen activator, this therapy is contraindicated within 14 days of major surgery. A diffusion-weighted MRI of the brain would be more sensitive than the CT scan was in detecting acute ischemia but is unnecessary in this patient. The diagnosis of acute ischemic stroke can satisfactorily be established clinically, and the MRI does not add anything of value to the patient's management.
question
Neuro 67 A 65-year-old man is hospitalized because of a 6-week history of progressive weakness and incoordination of the left arm and leg. He is otherwise healthy, with no relevant personal or family medical history. He takes no medications. Vital signs and other general physical examination findings are normal. Neurologic examination confirms mild weakness of the left arm and leg with decreased rapid alternating movements. A complete blood count and peripheral blood smear are normal. A contrast-enhanced MRI of the brain is shown . Lumbar puncture is ordered, and no abnormalities are found on subsequent analysis of the cerebrospinal fluid, including cytologic examination and flow cytometry. Which of the following is the most appropriate next step in management? A Chemotherapy with intrathecal methotrexate B Corticosteroid administration C Hospice referral D Stereotactic brain biopsy E Whole-brain radiation therapy
answer
D Primary central nervous system lymphoma is diagnosed by detection of a clonal B-cell population on cerebrospinal fluid analysis or by brain biopsy. This patient should undergo a stereotactic brain biopsy. He most likely has a primary central nervous system (CNS) lymphoma, given the distinctive radiographic signs. In immunocompetent patients, a primary CNS lymphoma is visualized most often as a solitary round mass with minimal surrounding edema that is situated in the deep white matter. Tumors are isointense to hypointense on T2-weighted MRI images and enhance homogeneously after contrast administration. Parenchymal brain metastases are rare in patients with systemic lymphoma. More commonly, parenchymal brain involvement occurs as an isolated site of disease (primary CNS lymphoma). Among patients presenting with a primary CNS lymphoma, occult systemic disease is present in less than 5%. The symptoms of CNS lymphoma are dependent on the site of tumor involvement. Primary CNS lymphoma is usually a B-cell non-Hodgkin lymphoma and is rare, accounting for less than 5% of intracranial tumors. It presents most commonly in the sixth and seventh decades of life in immunocompetent patients and in the fourth decade in HIV-infected patients. Despite its characteristic appearance, a primary CNS lymphoma cannot be accurately diagnosed by clinical presentation or neuroimaging studies. Because other conditions, both malignant and nonmalignant, can mimic primary CNS lymphoma, a tissue diagnosis is required before treatment can be appropriately initiated. Diagnostic modalities include stereotactic brain biopsy or cerebrospinal fluid analysis to detect a clonal B-cell population. Standard treatment of primary CNS lymphoma in immunocompetent patients consists of chemotherapy (usually high-dose methotrexate) followed by whole-brain radiation therapy if the patient's tumor is refractory to chemotherapeutic agents. However, neither of these treatments is appropriate before a biopsy establishes a tissue diagnosis. When lymphoma is suspected but not confirmed, corticosteroids should be avoided unless mandated by high risk of immediate morbidity and mortality from mass effect because they may cause the lesion to dramatically shrink or even disappear. This corticosteroid response is temporary and can complicate diagnosis by reducing the yield of diagnostic biopsy and delay therapy. The median survival of patients with CNS lymphoma is 3 years with combination chemotherapy and radiation therapy. Therefore, hospice referral is not indicated, particularly before the establishment of a tissue diagnosis.
question
Neuro 68 A 24-year-old woman is evaluated in the office for an 8-week history of severe daily headache. The headache is generalized and throbbing in quality and is associated with intermittent nausea and vomiting, episodes of visual blurring provoked by coughing and laughing, and the perception that she can hear her heartbeat. The headache has been unresponsive to oral administration of two different triptans and to a 1-month course of amitriptyline. The patient has a history of migraine without aura since age 13 years. Her only current medication is ibuprofen, as needed. On physical examination, her temperature is 36.4 °C (97.5 °F), blood pressure is 128/80 mm Hg, pulse rate is 88/min, respiration rate is 20/min, and BMI is 34. She has impaired lateral gaze with the left eye, indistinct disc margins, and no retinal venous pulsations. There is no impairment of visual acuity or visual fields to confrontation. Results of a complete blood count, liver chemistry studies, and measurements of serum creatinine, electrolyte, and thyroid-stimulating hormone levels are normal. An MRI of the brain and magnetic resonance venography of the brain show no abnormalities. Lumbar puncture reveals an opening pressure of 380 mm H2O. Results of cerebrospinal fluid analysis are normal. Which of the following is the most appropriate treatment at this time? A Acetazolamide B Nortriptyline C Propranolol D Sumatriptan, subcutaneously E Valproic acid
answer
A Acetazolamide is the option of first choice for the medical treatment of idiopathic intracranial hypertension. This patient should be treated with acetazolamide. She has idiopathic intracranial hypertension, a disease that primarily affects young obese women. She has a progressive daily headache associated with pulsatile tinnitus and transient visual obscurations, the most common symptoms of this disease. She also has papilledema and impaired lateral gaze to the left on presentation. These signs and symptoms suggest the possibility of a left sixth cranial nerve palsy. Abducens nerve palsies are sometimes seen in patients with idiopathic intracranial hypertension because of compression of the sixth cranial nerve as a result of elevated intracranial pressure. Definitive diagnosis is established by an elevated cerebrospinal fluid (CSF) pressure and normal results on CSF analysis. Because there is not yet evidence of visual field or visual acuity impairment, urgent surgical intervention is not necessary and medical therapy is appropriate. However, all patients with suspected idiopathic intracranial hypertension must undergo a thorough ophthalmologic evaluation, including formal visual perimetry testing, to detect enlargement of the blind spots or visual field defects that are not detected by confrontation visual field testing at the bedside. Acetazolamide is the medical option of first choice for the treatment of idiopathic intracranial hypertension. Although its exact mechanism of action is unclear, acetazolamide is a carbonic anhydrase inhibitor that decreases the production of CSF and relieves intracranial hypertension. No evidence suggests that nortriptyline, propranolol, and subcutaneously administered sumatriptan are appropriate treatments of idiopathic intracranial hypertension. Nortriptyline also has the potential adverse effect of weight gain, which could worsen the clinical course in this patient. Valproic acid is approved by the U.S. Food and Drug Administration for the preventive treatment of migraine. Although this patient has a history of migraine and her current headache is associated with features often seen during acute migraine attacks (nausea, emesis, severe headache, throbbing quality), the headache does not otherwise resemble migraine because it is persistent, progressive, unresponsive to triptan medications, and associated with some abnormal findings on examination. Valproic acid is therefore not appropriate in this circumstance, especially with its potential for weight gain as an adverse effect.
question
Neuro 69 A 54-year-old man with a 1-year history of Parkinson disease is brought to the office by his wife, who is concerned about her husband's recent excessive gambling. She says that in the past 6 months, he has been spending increasing amounts of time at a casino, where he rarely enjoyed going before the diagnosis of Parkinson disease. His behavior is otherwise unchanged. The patient has been taking ropinirole since the diagnosis and has had a marked diminution in tremor as a result; he has had no difficulties with or change in mood, cognition, or sleep. General physical examination findings are normal. Neurologic examination shows normal speech, language, mood, and mental status. There is mild left upper limb rigidity and a minimal resting tremor, but no other abnormalities are detected. Which of the following is the most likely cause of this patient's gambling problem? A Bipolar disorder B Frontotemporal dementia C Medication-related compulsive behavior D Parkinson-related dementia
answer
C A potential adverse effect of dopamine agonist therapy is the development of compulsive behaviors, such as pathologic gambling, shopping, and hypersexuality. This patient has developed an excessive gambling behavior after receiving treatment with the dopamine agonist ropinirole for Parkinson disease. Patients who are initiated and maintained on dopamine agonist medications to control Parkinson disease should be warned about the potential for developing abnormal, compulsive behaviors, such as excessive gambling, excessive shopping, and hypersexuality. These adverse effects, which can also develop in patients taking such medications for restless legs syndrome, are likely due to effects on the dopaminergic reward centers in the brain. Factors that can increase the risk of these behaviors include a young age at diagnosis in men and a history of mood disorders, alcohol abuse, or obsessive-compulsive behaviors. Other potential adverse effects of dopamine agonist therapy include orthostatic hypotension, nausea, vomiting, hallucinations, and sleep attacks. These potential adverse effects of dopamine agonist medications should be discussed with patients. Sleep attacks, or the sudden irresistible urge to sleep, have garnered considerable medicolegal and social attention. There is controversy about whether such attacks are distinct from excessive daytime somnolence, an established potential side effect of dopamine agonist medications. However, there have been rare reports of sleep attacks occurring while driving and resulting in motor vehicle accidents. The current recommendation is to warn patients taking these medications about the risk of sleep attacks while driving. Bipolar disorder is an illness characterized by periods of mood elevation and one or more episodes of depression. The period of mood elevation is characterized by a distinct period of abnormal and persistently elevated, expansive, or irritable mood that lasts at least 1 week. This patient's mood is described as normal and he lacks a history of depression, both of which make bipolar disorder unlikely. Patients with frontotemporal dementia may develop compulsive behaviors but would be expected to also exhibit other signs of personality or behavioral change. This patient's only sign of such a change is his compulsive gambling. The patient has no history or examination findings suggesting that Parkinson-related dementia has developed, which makes it an unlikely cause of his excessive gambling. Parkinson-related dementia is not associated with compulsive gambling behavior.
question
Neuro 70 A 33-year-old woman is evaluated in the emergency department for paresthesia that began in the left face and spread over 30 minutes to the left arm and leg, clumsiness of the left hand that began 30 minutes ago, and an emerging right-sided throbbing headache. She is otherwise healthy but has a family history of migraine. Her only medication is a daily oral contraceptive pill. On physical examination, temperature is normal, blood pressure is 140/82 mm Hg, pulse rate is 110/min, and respiration rate is 20/min. All other examination findings are normal. Results of laboratory studies and a CT scan of the head are also normal. Which of the following is the most likely diagnosis? A Migraine with aura B Multiple sclerosis C Sensory seizure D Transient ischemic attack
answer
A When assessing a patient with the acute onset of focal neurologic deficits, the examiner should include stroke mimickers, such as migraine with aura, in the differential diagnosis. This patient with stroke symptoms is most likely experiencing a migraine with aura and not a stroke. Migraine with aura is a stroke mimicker; stroke mimickers account for nearly one third of all stroke-alert calls in an emergency department. The clinical clues supporting a diagnosis of migraine are the patient's young age, the absence of vascular risk factors, the family history of migraine, and the presence and spread of the sensory symptoms. An MRI with diffusion-weighted imaging can rule out an acute ischemic stroke and thus help confirm the diagnosis of migraine with aura. Although multiple sclerosis (MS) should be in the differential diagnosis, this patient is less likely to have MS than a migraine or stroke because her presentation was more acute than would be typical in MS. Finding evidence of central nervous system demyelination on an MRI is the usual way MS is diagnosed; when such evidence is lacking, demyelination can sometimes be suggested by abnormal findings in the cerebrospinal fluid. Symptoms of stroke and transient ischemic attack (TIA) are described as negative or are said to involve loss of function. For example, there may be hemiparesis (a motor deficit affecting half the body) or bland sensory loss (numbness, loss of sensation, diminished sense of touch) in half the body. In contrast, partial seizures account for positive motor symptoms—such as involuntary unilateral muscle movement, twitching, and jerking—or positive sensory symptoms—such as paresthesia, tingling, or a feeling of "pins and needles." Sensory seizure symptoms generally reflect the anatomic organization of the sensory homunculus on the contralateral primary sensory cortex, whereas migraine symptoms may not. The sensory aura of a migraine generally spreads slowly over half the body. Rapidity of onset is another helpful clue in distinguishing migraine from TIA and seizure. A TIA comes on very rapidly (seconds), and seizures generally manifest in less than 1 minute. Migraine with aura, on the other hand, presents more slowly (over minutes, as with this patient), and symptoms spread slowly from region to region.
question
Neuro 71 A 25-year-old woman with epilepsy comes to the office seeking advice about pregnancy. She first developed seizures after sustaining a head injury in a motor vehicle collision at age 16 years. MRIs obtained since then have shown an area of encephalomalacia in the right temporal lobe. Her seizures were initially refractory to carbamazepine and valproic acid monotherapy. Carbamazepine was stopped, and lamotrigine was added to the valproic acid 1 year ago. She has not had any seizures since that time. Which of the following is the most appropriate management? A Advise the patient not to become pregnant B Continue the valproic acid and lamotrigine C Discontinue the valproic acid and continue the lamotrigine D Discontinue the valproic acid and lamotrigine E Substitute phenobarbital for her current medications
answer
C Infants exposed to antiepileptic medication during the first trimester of pregnancy have a 4% to 6% chance of having a major congenital malformation; malformation rates are greatest in infants exposed to valproic acid and polytherapy antiepileptic regimens. This patient should discontinue taking the valproic acid but continue taking the lamotrigine. All current antiepileptic drugs are classified by the U.S. Food and Drug Administration (FDA) as pregnancy risk category C or D. Infants exposed to antiepileptic drugs during the first trimester of pregnancy have a risk of major congenital malformation that is twice that observed in the general population (4%-6% versus 2%-3%). This teratogenic risk is greatest in those whose mothers were on a polytherapy antiepileptic drug regimen or valproic acid monotherapy. Furthermore, there is early evidence indicating that maternal antiepileptic drug use during pregnancy can have an adverse effect on the long-term cognitive and behavioral development of the children born to these mothers. However, the potential for harm is not great enough to justify counseling every woman with epilepsy against becoming pregnant. In light of the potential risk to their offspring, women with epilepsy should discuss the risks and benefits of treatment with their care providers and modify any risk, if possible. Because there is no antiepileptic drug in FDA pregnancy risk category A or B, the potential of safely discontinuing antiepileptic drugs should be assessed when a woman with epilepsy desires pregnancy. This patient has a known underlying structural lesion, has a history of seizures that were initially difficult to control, and has been seizure free for only 1 year. All these factors indicate it is unlikely that she could be safely taken off antiepileptic drugs at this time. Most women with epilepsy will require continued drug therapy during pregnancy, with the goal of reducing the medications to the greatest extent that can still reasonably be expected to maintain seizure control. Low-dose monotherapy is optimal; whenever possible, valproic acid should be discontinued or replaced because of its particularly high teratogenic potential. This patient has been seizure free since lamotrigine was added to her regimen. The most appropriate next step in management, therefore, is to discontinue the valproic acid and attempt to control her epilepsy with lamotrigine monotherapy. There continues to be a common misconception that phenobarbital is safer than other antiepileptic drugs during pregnancy. However, current data from large pregnancy registry studies do not support this assumption but demonstrate instead a risk of major malformations and adverse effects on intelligence in children whose mothers took phenobarbital during pregnancy.
question
Neuro 72 A 79-year-old woman was hospitalized 4 days ago after sustaining a right hip fracture in a fall. She underwent surgical repair with right hip replacement 3 days ago and did not awaken from general anesthesia until 12 hours after extubation. As her alertness has increased, she has become increasingly agitated, yelling at the nurses and flailing her arms; mechanical four-limb restraints were placed 2 days ago. The patient has a 4-year history of progressive cognitive decline diagnosed as Alzheimer dementia. She also has chronic atrial fibrillation treated with chronic warfarin therapy. She has no other pertinent personal or family medical history. Current medications are donepezil, memantine, atenolol, warfarin, and low-molecular-weight heparin. On physical examination today, temperature is 37.2 °C (99.0 °F), blood pressure is 100/68 mm Hg, pulse rate is 100/min and irregular, respiration rate is 18/min, and BMI is 21. The patient can move all four limbs with guarding of the right lower limb. She is inattentive and disoriented to time and place and exhibits combativeness alternating with hypersomnolence. The remainder of the neurologic examination is unremarkable, without evidence of focal findings or meningismus. Which of the following is the most likely diagnosis? A Acute cerebral infarction B Acute worsening of Alzheimer dementia C Meningitis D Postoperative delirium
answer
D Patients with chronic dementia, such as Alzheimer dementia, are at greater risk for delirium after surgery with general anesthesia. Abrupt worsening of confusion in elderly patients with chronic dementia usually results from an acute medical problem. In addition, patients with chronic dementia from almost any cause are at greater risk for delirium after surgery with general anesthesia. This patient with a hip fracture who underwent right hip surgery with general anesthesia and did not recover from the anesthesia until 12 hours after extubation most likely has postoperative delirium. Such delirium is highly predictable and often easily managed by identification and correction of any underlying disorders and the removal or reduction of contributing factors. In a patient with chronic atrial fibrillation who is confused postoperatively, the possibility of acute stroke must be considered. However, this patient has no clinical evidence of such an event, making this diagnosis extremely unlikely. Surgery does not exacerbate the dementia of Alzheimer dementia (or of any other cause) but rather produces a superimposed delirium. This patient has had dementia for 4 years that has abruptly gotten worse after surgery. Although not impossible, meningitis is highly unlikely in this setting, especially given the absence of any supporting physical examination findings, including meningeal irritation.
question
Neuro 73 A 36-year-old woman is evaluated in the emergency department for a 3-day history of confusion and falls. Her husband, who accompanied her, says that her symptoms seem to be getting worse. She has a 20-year history of Crohn disease that necessitated a partial small bowel resection because of stricture formation 5 years ago; she had a partial colectomy for fistulae 2 years ago and has had recent weight loss due to diarrhea. Current medications include prednisone and azathioprine. On physical examination, temperature is 35.6 °C (96.0 °F), blood pressure is 142/76 mm Hg, pulse rate is 90/min, respiration rate is 14/min, and BMI is 17. Temporal muscle wasting, sunken supraclavicular fossae, and absent adipose stores are noted. Abdominal examination reveals surgical scars and a few low-pitched bowel sounds, but results are otherwise normal. On neurologic examination, the patient is confused; she is unable to state the date and does not know the name of the hospital. Marked nystagmus is noted. There is no nuchal rigidity or obvious motor weakness. Deep tendon reflexes are reduced, and plantar responses are flexor. The patient has a markedly ataxic gait. Which of the following is the best initial management? A Electroencephalography B Haloperidol C Thiamine D Vancomycin, ampicillin, and ceftriaxone
answer
C Wernicke encephalopathy is due to thiamine deficiency; may result in mental status changes, ophthalmoplegia, nystagmus, and unsteady gait; and is best treated with thiamine. This patient should receive thiamine. She has Wernicke encephalopathy, a syndrome that results from deficiency of vitamin B1, an important coenzyme in several biochemical pathways of the brain. Typical clinical manifestations of the disorder include mental status changes, nystagmus, ophthalmoplegia, and unsteady gait, all varying in intensity from minor to severe. The typical clinical triad of ataxia, areflexia, and ophthalmoplegia is seen in only 19% of affected patients. Conditions associated with Wernicke encephalopathy include AIDS, alcohol abuse, cancer, hyperemesis gravidarum, prolonged total parenteral nutrition, postsurgical status (particularly gastric bypass), and glucose loading (in a predisposed patient). The disorder develops in patients who are malnourished as a result of malabsorption, a poor diet, increased metabolic requirements during illness, or thiamine deficiency. Recognition of the disorder and treatment with intravenous administration of thiamine are essential. Establishing a diagnosis of Wernicke encephalopathy can be difficult, but treatment with thiamine should not be withheld while considering other disorders. The diagnosis remains a clinical one and should be considered in any patient with poor nutrition or a disorder that can result in impaired absorption of food who exhibits one or more of the classic features of mental status change, gait impairment, and ocular signs. Measurements of serum thiamine level and erythrocyte transketolase activity lack specificity as diagnostic tests and may not be readily available. An MRI of the brain can help rule out other conditions and may show some characteristic changes of Wernicke encephalopathy, such as paraventricular signal changes in the thalamus, mamillary bodies, periaqueductal region, and cerebellum. Although these MRI changes seem to be quite specific for this disorder, their sensitivity is only 53%. Because Wernicke encephalopathy remains a clinical diagnosis, other neurologic disorders should be considered in this patient after thiamine has been administered. Electroencephalography can help exclude a seizure disorder, such as nonconvulsive status epilepticus. Infections, including encephalitis and meningitis, for which intravenous administration of broad-spectrum antibiotic drugs (such as vancomycin, ampicillin, and ceftriaxone) may be appropriate also should be part of the differential diagnosis and can be excluded with cerebrospinal fluid analysis. Haloperidol is not indicated in this patient, who is confused but has no apparent history of psychosis or agitation. Some patients with Wernicke encephalopathy do have agitation, hallucinations, and behavioral disturbances that can mimic an acute psychosis.
question
Neuro 74 A 65-year-old woman comes to the office for her annual examination. She reports that she had a tonic-clonic seizure at age 24 years after the birth of her daughter but has been seizure free on phenytoin since that time. The patient also has osteoporosis, diagnosed after a screening bone density scan. Current medications include phenytoin, alendronate, calcium, and vitamin D. Physical examination findings are normal. Which of the following is the most appropriate next step in management? A Check the serum phenytoin level B Continue the current dosage of phenytoin C Discontinue the phenytoin in a tapered fashion D Substitute lamotrigine for the phenytoin
answer
C Patients on antiepileptic medication who have been seizure free for 2 years should be considered for medication withdrawal. Phenytoin therapy should be discontinued in this patient in a tapered fashion. Although lifelong antiepileptic drug therapy is required for some patients and for some types of epilepsy, this is by no means always the case. As a general rule, discontinuation of antiepileptic drugs should be considered for patients who have been seizure free for 2 or more years. Medications should not be withdrawn in patients with epilepsy syndromes known to be lifelong, with underlying structural brain lesions, with symptomatic neurologic disorders, or (in most cases) with a history of medically refractory seizures. The risk of recurrent seizure when the patient is no longer taking the medication must always be balanced against the risks associated with continued antiepileptic drug treatment. Unfortunately, too many patients are treated unnecessarily for years because of the common misconception that antiepileptic drug therapy can never be safely discontinued. This patient has been seizure free for more than 40 years; in fact, the decision to initiate therapy was questionable because she only had a single event. Now she has osteoporosis, a condition which can be worsened by continued exposure to phenytoin. Therefore, the most appropriate next step in management is to gradually withdraw the medication. Because the phenytoin will be withdrawn, there is no need to determine a blood level prior to tapering the medication; the results will not affect when or how the medication will be tapered. This patient has been seizure free for more than 2 years and thus meets the criteria for careful withdrawal of the antiepileptic medication. There is no indication to substitute lamotrigine for the phenytoin or to continue the phenytoin.
question
Neuro 75 A 25-year-old woman is evaluated in the office for a 1-week history of numbness of the left leg. She also had a 2-week history of diplopia and vertigo 1 year ago; an MRI obtained at that time is shown . There is no family history of neurologic problems. She takes no medications. On physical examination, vital signs are normal. Plantar response is extensor on the left. There is mild loss of vibratory sense in both feet with a patchy, reduced pain sensation throughout the left lower extremity. Laboratory studies are noncontributory. A repeat MRI of the brain shows only the same periventricular white matter lesion. MRIs of the cervical spine and thoracic spine show no abnormalities. Which of the following is the most appropriate next step in diagnosis? A Electronystagmography B Lumbar puncture C Magnetic resonance angiography D Magnetic resonance spectroscopy
answer
B Cerebrospinal fluid analysis is useful when the clinical setting is suspicious for multiple sclerosis but neuroimaging is inconclusive. This patient should next undergo lumbar puncture. There is already a high clinical suspicion of multiple sclerosis (MS) because of the patient's age, her prior transient neurologic symptoms, the abnormal findings on neurologic examination, and the periventricular white matter lesion seen on two MRIs of the brain. However, these imaging and examination results are insufficient to confirm that multiple regions of the central nervous system are affected at different times (dissemination in time and space), which is one of the diagnostic criteria of MS. Up to 85% of patients with MS have an abnormal finding on cerebrospinal fluid analysis, such as the presence of oligoclonal bands or elevation of the IgG index. Therefore, lumbar puncture is the most appropriate next step in diagnosis. Confirmation of the diagnosis of MS at this stage would allow intervention with immunomodulatory therapy and result in a lower risk of both future relapses and accumulation of neurologic impairment. In the assessment of vestibular function, electronystagmography (ENG) uses electrodes to record eye movements to help discriminate between central and peripheral causes of vertigo. However, this patient's current leg numbness, history of diplopia and vertigo, and findings of an extensor plantar response, a loss of vibratory sense, and reduced pain sensation of the leg point to a process not confined to vestibular function but associated with manifestations that are separate in both time (two neurologic events over 2 years) and space (different parts of the central nervous system). This is most compatible with MS. Furthermore, in a patient without symptoms of vertigo, the ENG is likely to be normal. For similar reasons, magnetic resonance angiography, a noninvasive imaging technique used to detect vascular lesions, is unlikely to be helpful in this patient with probable MS. Magnetic resonance spectroscopy is at present a research technique that is not useful for MS diagnosis but may in the future play a role in monitoring disease progression and determining whether therapeutic interventions have neuroprotective effects.
question
Neuro 76 An 88-year-old man is brought to the emergency department within 45 minutes of the witnessed onset of dysarthria and right face, arm, and leg weakness. Recombinant tissue plasminogen activator is administered intravenously 105 minutes after symptom onset. The patient is then admitted to the intensive care unit, and 4 hours after thrombolysis, his neurologic symptoms and signs are rapidly improving. On physical examination, vital signs are normal, except for a blood pressure of 190/105 mm Hg. There is right pronator drift, right facial droop, and mild residual dysarthria. Which of the following is the most appropriate treatment of this patient's elevated blood pressure? A Intravenous labetalol B Intravenous nitroprusside C Oral nifedipine D Withholding of antihypertensive medications
answer
A In a patient with ischemic stroke treated with recombinant tissue plasminogen activator (rtPA), systolic blood pressure should be kept below 180 mm Hg and diastolic below 105 mm Hg for 24 hours after rtPA treatment; intravenous labetalol or nicardipine can best achieve this goal. In patients who have received recombinant tissue plasminogen activator as treatment of stroke, systolic blood pressure should be maintained below 180 mm Hg and diastolic blood pressure below 105 mm Hg for at least the first 24 hours after thrombolysis treatment. According to current clinical guidelines, intravenous administration of labetalol or nicardipine can best achieve this goal (class II recommendation). Therefore, of the options listed, intravenous administration of labetalol is most appropriate for this patient whose systolic blood pressure is 190 mm Hg and whose diastolic blood pressure is 105 mm Hg. Intravenous administration of nitroprusside should be instituted only if either labetalol or nicardipine proves unsuccessful in controlling this patient's blood pressure. Intravenous nitroprusside is considered second-line therapy because it may be associated with increased intracranial pressure. Oral administration of nifedipine is inappropriate because of its rapid absorption, which can result in a secondary precipitous decline in blood pressure. Withholding antihypertensive medications is inappropriate in this patient. Excessively high blood pressure is associated with an increased risk of symptomatic hemorrhagic transformation after thrombolytic therapy and may be prevented with careful adjustment of the blood pressure to target levels recommended by treatment guidelines.
question
Neuro 77 A 34-year-old woman is evaluated in the office for right-sided facial paralysis that she noticed on awakening 1 hour ago. She has a 10-pack-year smoking history. Personal and family medical history is noncontributory. Her only medication is a daily oral contraceptive. On physical examination, temperature is 36.5 °C (97.7 °F), blood pressure is 110/70 mm Hg, pulse rate is 82/min, respiration rate is 14/min, and BMI is 26. Limb strength, reflexes, and tone are normal bilaterally. Findings from a sensory examination, which included her face, are also normal. When asked to raise her eyebrows, the patient does not elevate the right side. When asked to shut her eyes, she cannot close the right one, but the globe rotates upward, partially covering the iris. When asked to smile, the patient does not move the right side of her face. Which of the following is the most likely diagnosis? A Graves ophthalmopathy B Left cerebral infarction C Right facial nerve (Bell) palsy D Right trigeminal neuralgia
answer
C Any cause of a complete facial neuropathy will impair the entire hemiface, including the forehead muscles. This patient's physical examination findings most strongly suggest right facial nerve (Bell) palsy. The precise cause of Bell palsy is not known, and it is still considered an idiopathic disorder. Research strongly suggests it may be the result of herpes simplex virus infection of the facial nerve. Bell palsy is not considered contagious. The seventh cranial nerve innervates all muscles of facial expression (the mimetic muscles). Any cause of a complete facial neuropathy will therefore impair the entire hemiface, including the forehead corrugators typically spared by cerebral lesions. Bell phenomenon describes the reflexive rolling upwards of the globe during eye closure. When a patient is asked to close the eyes, forced eyelid opening will reveal this phenomenon, as will the selective paralysis of the orbicularis oculi due to a facial neuropathy. Facial neuropathies will otherwise spare the extraocular muscles that govern globe movement. Because Bell palsy is a diagnosis of exclusion, clinicians need to make every effort to exclude other identifiable causes of facial paralysis, such as Lyme disease, acute and chronic otitis media, cholesteatoma, and multiple sclerosis. Other common causes of acute peripheral facial paralysis will often have findings on history or physical examination that suggest the correct diagnosis. Graves ophthalmopathy can cause proptosis or extraocular muscle edema with consequent eye movement abnormalities but is not associated with the facial hemiparalysis typical of facial nerve (Bell) palsy. Cerebral infarction, brain hemorrhage, or any structural brain lesion can cause weakness of the lower face but not of the forehead because the bilateral cortical representation of the midline forehead spares the forehead corrugators. Some limb or sensory abnormality is also often, but not universally, observed in the setting of cerebral infarction; no such abnormality was observed in this patient. Therefore, despite her cerebrovascular risk factors of oral contraception and cigarette smoking, this patient is unlikely to have had a cerebral infarction. The trigeminal nerve provides sensation, not movement, to the muscles of facial expression, so trigeminal neuralgia is not a likely diagnosis in this patient with normal sensation.
question
Neuro 78 A 46-year-old man is evaluated in the office for a 6-month history of a resting right-arm tremor. He says that his writing has gotten smaller during this time and that he has had difficulty buttoning his dress shirts. The patient reports no prior medical problems and is not aware of any neurologic problems in his family. He takes no medications. Results of a general medical examination are normal. Neurologic examination shows a paucity of facial expression (hypomimia). Cranial nerve function is normal. Motor examination shows normal strength but mild left upper limb rigidity and a 5-Hz resting tremor of the right upper limb. Deep tendon reflexes are normal, as are results of sensory examination. There is no truncal or appendicular ataxia. Diminished arm swing is noted bilaterally, but it is worse on the right. A tremor in the right upper limb is noted during ambulation. Left upper limb alternating motion rates are diminished. Which of the following is the best treatment for this patient? A Amantadine B Pramipexole C Primidone D Propranolol
answer
B Dopamine agonist medications are used as first-line treatment of Parkinson disease in patients younger than 65 years, whereas levodopa is used in patients age 65 years or older. This patient should be treated with pramipexole. He has classic signs of Parkinson disease, including tremor, rigidity, and bradykinesia. There are no effective neuroprotective agents to treat this disorder. Dopamine agonist medications, either pramipexole or ropinirole, are indicated for the initial treatment of the parkinsonian symptoms in this young patient with apparent Parkinson disease. Motor complications, such as dyskinesias (abnormal involuntary movements), an end-of-dose "wearing-off" effect, and fluctuations, may be less frequent and less severe with dopamine agonist medications than with levodopa. Levodopa, a precursor of dopamine, is the most efficacious medication used to treat the symptoms of Parkinson disease but is typically initiated only in patients older than 65 years. The associated development of motor fluctuations occurs at a rate of 10% annually in these patients but may develop more rapidly and be more severe in younger patients taking levodopa as an initial medication. Carbidopa is administered in conjunction with levodopa to prevent the peripheral conversion of levodopa to dopamine. Amantadine has been used in the treatment of Parkinson disease since the 1970s but is not currently a medication of choice for initial therapy. Its mechanism of action in the treatment of Parkinson disease is unknown, but amantadine may increase dopamine release via antagonism at the N-methyl-D-aspartic acid receptor. The drug also has anticholinergic properties. Amantadine may reduce dyskinesias, one of the motor complications of Parkinson disease. However, any initial improvement in their parkinsonian symptoms that patients taking amantadine experience may be modest and not sustained. Primidone is used in patients with essential tremor. Essential tremor is distinguished from Parkinson disease by its occurrence with limb movement and the lack of parkinsonian signs on examination. Likewise, propranolol would not be of any benefit in this patient with the typical resting tremor of Parkinson disease. Propranolol remains the drug of choice in the treatment of patients with essential tremor.
question
Neuro 79 A 70-year-old man is seen in the office for routine follow-up of partial seizures that began 2 years ago after a stroke. At that time, he was started on phenytoin, 300 mg/d, and has had no subsequent seizures; he tolerates the medication well, with no reported adverse effects. His current medications are phenytoin, an angiotensin-converting enzyme inhibitor, a statin, and aspirin. Results of physical examination, including a neurologic examination, are normal. Laboratory studies show a total serum phenytoin level of 9 mg/L (35.6 µmol/L) (therapeutic range, 10-20 mg /L [39.6-79.2 µmol/L]). Results of a complete blood count and liver chemistry tests are normal. Which of the following is the most appropriate next step in management? A Continue the phenytoin at the current dosage B Increase the phenytoin dosage C Measure the free serum phenytoin level D Substitute gabapentin for the phenytoin
answer
A Adjustments to the dosage or type of antiepileptic drug used by a patient with epilepsy should be based on clinical seizure control and drug side effects rather than strict serum drug levels. The most important considerations that should guide decisions about adjusting epilepsy medication dosage or changing the type of drug used are the current clinical seizure control and the presence of adverse side effects. This patient reports a prolonged period of freedom from seizures and the absence of adverse effects while on a stable dosage of phenytoin. Although the patient's total serum phenytoin level is mildly subtherapeutic, this in itself is not a sufficient indication to make a change in the type or dosage of medication. Therefore, the patient should be kept on phenytoin at his present dosage of 300 mg/d as long as a seizure or adverse effect of medication does not occur. Monitoring of serum antiepileptic drug levels can be useful when it is necessary to confirm that a level is therapeutic in a patient with uncontrolled seizures, when toxicity is suspected, or when medication adherence needs to be confirmed; none of these scenarios applies to this patient. Serum drug level monitoring can be particularly helpful with phenytoin because of its narrow therapeutic window and its nonlinear pharmacokinetics, which can result in unexpectedly large changes in serum concentration from small adjustments in oral dosage. For phenytoin and other highly protein-bound drugs, checking the free (as well as the total) serum drug levels may be necessary if there is concern about therapeutic effectiveness or toxicity in the presence of a low serum protein level or polypharmacy with other highly protein-bound drugs; again, these indications do not apply to this patient.
question
Neuro 80 A 26-year-old woman is evaluated in the office for a change in migraine symptoms. She began having migraine attacks shortly after menarche at age 13 years, experiencing an attack approximately every 2 months. For the past 8 weeks, these attacks have been associated with visual aura. The neurologic symptoms evolve over a period of 10 minutes, last less than 60 minutes, and are followed within 30 minutes by a severe, unilateral throbbing headache associated with nausea. Sumatriptan relieves the headache within 30 minutes. The patient also has asthma. Her mother and sister have a history of migraine. Current medications are an oral contraceptive pill started 9 weeks ago, sumatriptan as needed, a daily inhaled corticosteroid, and an inhaled β-agonist as needed. Results of physical examination, including neurologic examination, are normal. Complete blood count results, erythrocyte sedimentation rate, serum chemistry study results, thyroid-stimulating hormone level, and anticardiolipin and antinuclear antibody levels are normal. An MRI of the brain shows no abnormalities. Which of the following is the most appropriate next step in management? A Add propranolol B Add verapamil C Discontinue the oral contraceptive pill D Discontinue the sumatriptan E Measure serum lactate and pyruvate levels
answer
C Oral contraceptive pills are contraindicated in women with migraine with aura, especially if the aura involves more than just simple visual aura, if there are additional stroke risk factors, or if the aura begins after the initiation of oral contraception. The American Academy of Obstetrics and Gynecology considers migraine with aura an absolute contraindication to the use of oral contraception. The International Headache Society strongly advises against the use of oral contraceptive pills in women who have migraine with aura that involves more than just the visual system and recommends stopping the pill in women who develop an aura after the pill is started. Patients who have migraine with aura have up to an eight-fold increased risk of ischemic stroke; this risk is tripled by smoking and quadrupled by the use of oral contraception. This patient, therefore, should discontinue using oral contraception. There is little evidence to support the use of propranolol or verapamil for the prevention of migraine in this patient. In any case, the frequency of her migraines (approximately one every 2 months) does not justify the use of preventive medication. Triptans are not contraindicated in patients with migraine with aura. Therefore, the sumatriptan can continue to be used for acute migraine relief in this patient. Mitochondrial Encephalopathy with Lactic Acidosis and Stroke-like episodes (MELAS) is a maternally inherited mitochondrial disorder associated with lactic acidosis and a high prevalence of migraine with aura. The hallmark of MELAS is the occurrence of nonischemic stroke manifesting as hemiparesis, hemianopia, or cortical blindness. The course is progressive and leads to progressive neurologic impairment and dementia. Other features may include short stature, hearing loss, and muscle weakness. However, this patient has no clinical or imaging features of this disorder, and measuring her serum lactate and pyruvate levels would not be appropriate.
question
Neuro 81 An 82-year-old man is evaluated in the office for an episode of hesitancy in speech, word-finding difficulty, right facial droop, and weakness and awkwardness of the right hand and arm. The episode occurred early yesterday, lasted 20 minutes, and was witnessed by his wife. The patient has a history of coronary artery disease, hypertension, and hyperlipidemia. Current medications are metoprolol, aspirin, hydrochlorothiazide, and lovastatin. On physical examination, temperature is normal, blood pressure is 148/88 mm Hg, pulse rate is 70/min, and respiration rate is 12/min. Neurologic examination reveals no abnormalities. Which of the following is the most appropriate next step in management? A Add clopidogrel B Admit to the hospital C Order outpatient diagnostic studies D Schedule a follow-up visit in 1 week
answer
B Patients with a diagnosis of a recent transient ischemic attack are at an appreciably high short-term risk of stroke and should be evaluated in a hospital in an expedited and emergent fashion. This patient should be admitted to the hospital. Given his clinical history, he most likely has had a recent transient ischemic attack (TIA). His ABCD2 score (based on Age, Blood pressure, Clinical features, the Duration of symptoms, and the presence of Diabetes) is 5: one point is for his age (>60 years), one point for his hypertension, one point for a symptom duration of greater than 10 minutes, and two points for the focal weakness he described. This score is moderately high and carries an estimated stroke risk of 5% over the next 2 days, 7% over the next week, 10% over the next 30 days, and 12% over the next 3 months. Therefore, the most appropriate response is for this patient to undergo urgent evaluation within the next 24 hours at an emergency department, at a hospital during a brief admission, or at an organized urgent TIA clinic. It is reasonable to review the stroke prevention regimen of a patient with risk factors and make adjustments to any antiplatelet medications in the context of a new stroke or TIA. However, the priority is an expedited evaluation to determine the cause and mechanism of the stroke or TIA, such as symptomatic extracranial carotid artery stenosis amenable to endarterectomy, intracranial stenosis amenable to angioplasty and stenting, or cardioembolism with requirements for long-term anticoagulation. Therefore, adding clopidogrel to this patient's drug regimen is not the most appropriate next step in management. Outpatient diagnostic studies may play a role in the assessment of this patient, but only if they occur and results are back within 24 hours. Because this scenario is unlikely, such studies are clearly not the most appropriate next step in management. Given the high probability of an acute ischemic stroke event and the high short-term risk of stroke in this patient, scheduling a follow-up appointment in 1 week in the absence of diagnostic testing or evaluation could be life threatening.
question
Neuro 82 A 33-year-old woman is admitted to the hospital for evaluation and treatment of new-onset transverse myelitis that has resulted in severe paraparesis. The patient also has had two recent episodes of optic neuritis. She reports no systemic symptoms, such as fever, rash, arthralgias, or pulmonary problems. She has a history of hypothyroidism but no family history of any neurologic disorders. Her only medication is levothyroxine. Vital signs are normal on physical examination. Visual acuity is 20/200 in the right eye and 20/30 in the left. Bilateral optic disc pallor and severe spastic paraparesis with loss of all sensory modalities below T10 are noted. The patient requires bilateral assistance to ambulate 5 meters. Results of laboratory studies are normal, including a complete blood count, liver chemistry and renal function tests, and measurement of erythrocyte sedimentation rate and C-reactive protein level. The antinuclear antibody is positive, but anti-double-stranded DNA and anti-SSA/SSB antibodies are negative. Analysis of the cerebrospinal fluid shows a normal IgG index and no abnormalities in oligoclonal banding. An MRI of the spinal cord reveals an increased signal extending over five vertebral segments with patchy gadolinium enhancement. An MRI of the brain shows no abnormalities. Which of the following is the most appropriate next diagnostic test? A Electromyography B Serum antineutrophil cytoplasmic antibody test C Serum neuromyelitis optica (NMO)-IgG autoantibody test D Testing of visual evoked potentials
answer
C Neuromyelitis optica (NMO) is a severe demyelinating disease of the central nervous system that is distinct from multiple sclerosis and associated with the autoantibody marker NMO-IgG (anti-aquaporin-4). This patient very likely has neuromyelitis optica (NMO), a severe demyelinating disease of the central nervous system that is distinct from multiple sclerosis (MS). She should be tested for the autoantibody marker NMO-IgG (anti-aquaporin-4). NMO occurs more commonly in nonwhite persons, is often associated with serum autoantibodies or other autoimmune diseases, and has a predilection for the optic nerves and spinal cord with relative sparing of the brain. This patient's spinal cord lesion is also characteristic of NMO because it extends over more than three vertebral segments; cord lesions in typical MS are usually less than two segments in length. The finding of the NMO-IgG autoantibody marker is approximately 75% sensitive and more than 90% specific for NMO. Differentiating between NMO and MS as early in the disease course as possible is important because the prognosis and treatment of the two diseases are different. NMO is a more severe disease treated with immunosuppressive drugs, whereas MS is initially treated with immunomodulatory therapies, such as β-interferon and glatiramer acetate. There are no symptoms or signs of peripheral nerve or muscle involvement in this patient. Therefore, electromyography is not indicated. Cytoplasmic and perinuclear-staining antinuclear cytoplasmic antibodies (ANCAs) may be detected in patients with systemic vasculitis. Although vasculitis is a rare cause of transverse myelitis, it is very unlikely to explain multiple episodes of optic neuritis and transverse myelitis in the absence of systemic symptoms. Visual evoked potential testing will confirm the known involvement of the optic nerves in this patient but will not otherwile aid diagnosis and so is unnecessary.
question
Neuro 83 A 30-year-old man with epilepsy is evaluated for disabling seizures that occur once a month despite adherence to his drug therapy. He also reports excess sedation and cognitive slowing related to his medication. The patient has had partial complex seizures since age 16 years and has been previously treated with appropriate dosages of phenytoin, carbamazepine, and valproic acid; he currently takes lamotrigine, 200 mg twice daily (dose range, 100-600 mg/d). He is otherwise in excellent health and has no other medical problems. Physical examination, including neurologic examination, reveals no abnormal findings. An MRI of the brain shows an increased T2 signal and atrophy of the right hippocampus, both consistent with mesial temporal sclerosis. An electroencephalogram shows right temporal sharp waves. Which of the following is the most appropriate next step in management? A Add another anticonvulsant to his drug regimen B Increase the dosage of lamotrigine C Refer him for epilepsy surgery evaluation D Refer him for implantation of a vagus nerve stimulator
answer
C Patients with disabling partial seizures that have not responded to treatment with two appropriate anticonvulsant drugs should be considered for epilepsy surgery. For patients with medically refractory partial epilepsy whose seizures are adversely affecting their quality of life, referral for epilepsy surgery evaluation is the most appropriate next step in management. Seizures arising from mesial temporal sclerosis, as is suggested by this patient's MRI, are particularly amenable to surgical cure, with seizure-free rates reported to be as high as 80% after resection. The most common surgical procedure is surgical resection of mesial temporal lobe sclerotic lesions. Quality of life and social functioning improve after successful surgery, and surgical morbidity and mortality are low. In fact, surgery may improve long-term mortality. Approximately 30% of patients with epilepsy have seizures that cannot be adequately controlled with currently available medical therapies. If seizures have not responded to two medications that are appropriate for the seizure disorder and are used at an adequate dosage, the likelihood of achieving complete control with subsequent medication trials is less than 10%. The patient described has continued disabling seizures despite trials of four medications, all appropriate for treating partial complex seizures and all given at adequate dosages. Therefore, adding another drug to his current regimen is unlikely to improve his condition. His current dosage of lamotrigine, although not the theoretical maximal daily dosage, represents a reasonable trial of the drug. A further increase in the lamotrigine dosage is not the best choice for this patient who reports limiting adverse effects on the current regimen. The vagus nerve stimulator is an approved therapy for medically refractory partial epilepsy that is best used as a palliative option for patients who are not candidates for resective surgery. This patient has no contraindications to such surgery.
question
Neuro 84 A 73-year-old woman is evaluated in the emergency department for the onset of a severe, "explosive" headache 8 hours ago. She initially rested in a dark bedroom after headache onset, but when the pain did not abate, her husband drove her to the hospital. The patient has hypertension controlled with lisinopril. Family medical history is noncontributory. As assessment in the emergency department begins, she becomes nauseated, vomits, and then becomes rapidly and progressively more obtunded. Intubation and mechanical ventilation are required. On physical examination, temperature is normal, blood pressure is 188/102 mm Hg, pulse rate is 120/min, and respiration rate is 20/min. The patient exhibits a flaccid quadriplegia, and meningismus is present. Both pupils are 4 mm in diameter and nonreactive; the oculocephalic reflex is absent, and the corneal reflex is absent bilaterally. She has a depressed level of consciousness, with a Glasgow Coma Scale score of 3. Subhyaloid hemorrhages are noted on funduscopy. Results of a complete blood count (with differential) are normal, as are blood urea nitrogen, serum creatinine, and serum electrolyte levels. A CT scan of the head shows an extensive acute subarachnoid hemorrhage and mild prominence of the temporal tips of the lateral ventricles. Which of the following neurologic complications is most likely to have caused this patient's rapid deterioration? A Hydrocephalus B Rebleeding C Syndrome of inappropriate antidiuretic hormone secretion D Vasospasm
answer
B Rebleeding is the most imminent danger after a subarachnoid hemorrhage. The most likely complication to have caused this patient's rapid deterioration is rebleeding. In the first few hours after an initial hemorrhage, up to 15% of affected patients have a sudden deterioration of consciousness, which strongly suggests rebleeding. In patients who survive the first day, the rebleeding risk is evenly distributed during the next 4 weeks, with a cumulative risk of 40% without surgical or endovascular interventions. Occlusion of the responsible aneurysm is thus the first aim in the management of a subarachnoid hemorrhage and is usually performed by coiling or clipping. Typically, patients who develop hydrocephalus after having a subarachnoid hemorrhage are initially alert but then experience a gradual reduction in consciousness over the next 24 hours. Downward deviation of the eyes and small, unreactive pupils indicate dilatation of the proximal part of the cerebral aqueduct with dysfunction of the pretectal area. Although this patient may have secondarily developed hydrocephalus, her symptoms and examination findings suggest that the principal reason for her precipitous decline is early rebleeding. On average, one in five patients who sustain subarachnoid hemorrhages will have mildly enlarged ventricles on the initial CT scan but not frank hydrocephalus. The syndrome of inappropriate antidiuretic hormone secretion is recognized as a potential complication in patients with subarachnoid hemorrhages and other critical care neurologic conditions. Its symptoms and signs are muscle cramps, weakness, altered sensorium, coma, and seizures. This syndrome causes the electrolyte disturbance of hyponatremia. This patient's serum electrolytes were determined to be in the normal range at the time of her evaluation in the emergency department. Vasospasm-induced cerebral ischemia after a subarachnoid hemorrhage has a more gradual onset than occurred in this patient. It often involves more than the territory of a single cerebral artery. The clinical manifestations evolve gradually over several hours and consist of hemispheric focal deficits, a reduction of consciousness, or both. The peak frequency of vasospasm is from 5 to 14 days after the subarachnoid hermorrhage.
question
Neuro 85 A 32-year-old woman is evaluated for a gradual increase in migraine frequency and severity over the past 6 months. Migraine attacks, which formerly occurred two or three times each month, are now occurring approximately three times each week, with each attack lasting at least 12 hours. She has a 15-year history of asthma, a 10-year history of migraine without aura, and a 4-year history of type 2 diabetes mellitus controlled by diet. Her mother and sister also have a history of migraine. Current medications are almotriptan as needed for acute migraine, an inhaled corticosteroid, and an albuterol inhaler as needed for wheezing. On physical examination, blood pressure is 130/80 mm Hg, pulse rate is 88/min, and BMI is 34. Results of a physical examination, including a neurologic examination, are normal. An MRI of the brain shows no abnormalities. Which of the following is the most appropriate treatment for this patient? A Gabapentin B Nortriptyline C Propranolol D Topiramate E Valproic acid
answer
D Prophylactic medication should be initiated in patients with two or more migraine attacks per week. Prophylactic treatment should generally be initiated in patients with two or more migraine attacks per week. There is level 1 evidence to support the use of topiramate for the prevention of migraine, and the U.S. Food and Drug Administration has approved the drug for this purpose. This patient is obese (BMI of 34) and has type 2 diabetes mellitus. Any medication with the potential for weight gain must, therefore, be used with caution, given the morbidity associated with obesity and the potential to worsen her underlying hyperglycemia. Topiramate is associated with weight loss. Gabapentin is a second-tier drug because of the lower level of evidence supporting its use. It is also not approved by the FDA for the preventive treatment of migraine. There is no evidence of nortriptyline's efficacy in the preventive treatment of migraine. Moreover, it is also associated with weight gain. Propranolol, a nonselective β-blocker, is contraindicated in patients with persistent asthma and has a relative contraindication in patients with diabetes mellitus. Valproic acid, although also supported by level 1 evidence and FDA approval for migraine prevention, is associated with weight gain and is not the best treatment for this patient. Additionally, in light of the potential teratogenicity associated with this drug, it is often avoided in women of childbearing potential.
question
Neuro 86 A 24-year-old woman is evaluated for a 2-month history of abnormal movements affecting both arms. She describes these movements as intermittent and irregular writhing movements and says she cannot suppress them. The patient had an unprovoked deep venous thrombosis in her right leg 2 years ago and a miscarriage 8 months ago but has had no other medical problems. She has no known family history of neurologic disease or thrombophilia and takes no medications. General physical examination findings are normal. Neurologic examination reveals random, irregular movements of the hands and arms but is otherwise unremarkable. Which of the following is the most likely diagnosis? A Anti-phospholipid antibody syndrome B Huntington disease C Parkinson disease D Wilson disease
answer
A Chorea can occur in patients with anti-phospholipid antibody syndrome or systemic lupus erythematosus. This patient most likely has anti-phospholipid antibody syndrome. She has movements consistent with chorea, with no other identifiable neurologic signs or symptoms. Chorea is characterized by random, nonstereotyped movements that can affect virtually any body part. Anti-phospholipid antibody syndrome is characterized by a history of a thrombotic event (including recurrent fetal loss) in association with a persistent lupus anticoagulant or persistently elevated levels of anticardiolipin or B2-glycoprotein I antibodies. Lupus anticoagulants or elevated levels of anti-phospholipid antibodies are often present in patients with systemic lupus erythematosus; they also occur in patients with cancer or infections and in association with the use of certain drugs (for example, hydralazine, procainamide, phenothiazines). A multitude of neurologic disorders can occur in patients with anti-phospholipid antibody syndrome, including chorea. Chorea has also been associated with systemic lupus erythematosus and may occur early in the course of either disorder, even before other signs develop. Levels of anticardiolipin antibodies, antinuclear antibodies, and anti-double-stranded DNA antibodies should be measured in the diagnostic evaluation of chorea. Huntington disease is not likely in this patient. A progressive, autosomal dominant disorder, Huntington disease has a mean age of onset of approximately 40 years. A history of psychiatric disturbance may precede the motor manifestations of the disease, and cognitive abnormalities are present in nearly all affected patients, appearing early in the course of the disease. Chorea is also present in nearly all patients, and other extrapyramidal findings, such as dystonia and parkinsonian signs, are commonly seen. Chorea commonly occurs in patients with Parkinson disease who are on dopamine therapy and is thought to reflect high levels of dopamine in the basal ganglia. Typically, these choreiform movements, also known as dyskinesias, occur at the peak of the dopamine medication effect and, if mild, may not be bothersome to the patient. Given that this patient has no symptoms of Parkinson disease, such as a resting tremor, and is not on medication, this diagnosis is very unlikely. Wilson disease is an autosomal recessive disorder that most commonly results in psychiatric symptoms and parkinsonian signs, neither of which are present in this patient. Chorea occurs in approximately 15% of patients with Wilson disease.
question
Neuro 87 A 31-year-old woman with a history of migraine is evaluated in the office for several severe migraine attacks that are not well controlled with over-the-counter analgesics. The headaches are severe, are associated with severe nausea and vomiting, and began 2 weeks ago after she delivered a healthy baby boy. She is currently breast feeding. Results of physical examination, including neurologic examination, are normal. Which of the following is the most appropriate treatment for this patient? A Butalbital B Extra-strength acetaminophen C Frovatriptan D Metoclopramide E Prochlorperazine
answer
C Using frovatriptan or sumatriptan for treatment of migraine is compatible with breast feeding, according to the American Academy of Pediatrics. This patient should be taking frovatriptan for her migraine attacks. Frovatriptan and sumatriptan are both considered compatible with breast feeding by the American Academy of Pediatrics. The patient has not responded to first-line therapy with simple analgesics and is reporting severe migraine attacks, for which a triptan is indicated. The American Academy of Pediatrics has given butalbital and acetaminophen a rating of caution and metoclopramide and prochlorperazine a rating of concern for mothers who are breast feeding. Therefore, these drugs should be avoided in such mothers. In addition, the patient has already reported a lack of response to over-the-counter analgesics, so extra-strength acetaminophen is not likely to result in marked improvement.
question
Neuro 88 A 30-year-old man is reevaluated for management of partial epilepsy. He first developed seizures 1 year ago after surgical treatment of a ruptured aneurysm of the right middle cerebral artery. He was started on phenytoin (400 mg/d) at that time but continues to have monthly episodes in which he experiences a rising epigastric sensation followed by loss of awareness lasting 1 to 2 minutes and then several hours of confusion and significant fatigue. He also reports gait imbalance and excessive fatigue related to the phenytoin. He takes no other medications. Physical and neurologic examination findings are noncontributory. Laboratory studies show a current serum phenytoin level of 18 mg/L (71.3 µmol/L) (therapeutic level, 10-20 mg/L [39.6-79.2 µmol/L]). Which of the following is the most appropriate next step in management? A Add another antiepileptic drug to his current regimen B Gradually transition from phenytoin to a new antiepileptic drug C Increase the dosage of phenytoin D Refer for epilepsy surgery evaluation E Refer for vagus nerve stimulation
answer
B Monotherapy is preferred over polytherapy drug regimens as treatment of epilepsy. This patient should gradually transition to a new antiepileptic drug. He continues to have disabling seizures, despite taking phenytoin for 1 year and despite having a high therapeutic serum drug level. Only approximately half of the patients prescribed a first antiepileptic drug for seizure control will continue on that drug. For the others, initial therapy will be unsuccessful, either from lack of efficacy or from unacceptable side effects. When patients are unsuccessfully treated with a first antiepileptic drug, the recommendation is to try a second antiepileptic drug as monotherapy. It is generally recommended to gradually transition between drugs, titrating the new drug to therapeutic range before tapering off the initial agent. There is no compelling evidence that polytherapy improves seizure control over monotherapy. However, the primary rationale for continued monotherapy in a patient unsuccessfully treated with a first antiepileptic drug is the increased rate of adverse effects and drug interactions when antiepileptic drugs are used in combination. Concerns about adherence and increased expense also play a role. Therefore, adding another antiepileptic drug to this patient's drug regimen is not appropriate as the next step in management. Although an increase in phenytoin dosage might be considered in some cases, this patient has already reported clinically significant adverse effects from the drug. These would likely limit his ability to tolerate a higher dosage. Epilepsy surgery and vagus nerve stimulation are reserved for patients with medically refractory seizure disorders and are generally not considered until at least two antiepileptic drugs have been shown to be ineffective in controlling seizures. This patient has thus far tried only phenytoin therapy. A referral for either of these treatments is premature at this point.
question
Neuro 89 A 68-year-old man is evaluated in the office for a 4-week history of continuous left leg pain and weakness. The pain is severe, burning, and in the distribution of the anterior thigh. He has had a 4.5-kg (10-lb) weight loss in the last 3 weeks. The patient had prostate cancer 1 year ago and underwent radiation therapy to the pelvis. He also has hypertension treated with lisinopril. General physical examination findings are normal, except for a BMI of 31. Neurologic examination shows normal speech, language, and cranial nerve function. Strength and deep tendon reflexes are normal in the upper extremities and in the right leg. There is diffuse weakness, greatest in the quadriceps, hip adductor, and iliopsoas muscles, and absent knee and ankle jerks in the left leg; plantar response is flexor. Sensory loss is evident in the distribution of the saphenous sensory nerve on the left. Laboratory studies show an elevated fasting plasma glucose level of 132 mg/dL (7.3 mmol/L). All other laboratory results, including erythrocyte sedimentation rate; serum electrolyte, creatinine, and thyroid-stimulating hormone levels; and liver chemistries, are normal. Which of the following is the most likely diagnosis? A Cauda equina syndrome B Diabetic lumbosacral radiculoplexus neuropathy C Myelopathy D Radiation-induced lumbosacral plexopathy
answer
B Diabetic lumbosacral radiculoplexus neuropathy (diabetic amyotrophy) is characterized by severe, initially unilateral lower limb pain and weakness. Diabetic lumbosacral radiculoplexus neuropathy, or diabetic amyotrophy, is the most likely diagnosis in this patient. This neuropathy is a subacute, progressive disorder that causes asymmetric leg pain, sensory loss, and weakness. Weight loss of 4.5 kg (10 lb) or more occurs in most affected patients. Many patients with this disorder are unaware that they have diabetes mellitus before the development of diabetic lumbosacral radiculoplexus neuropathy, and in most patients, glycemic control is not severely compromised. This disorder usually begins with unilateral leg pain followed by weakness and sensory loss, which spread to involve the contralateral leg nearly all the time. Weakness is often greatest proximally initially, but over time, diffuse weakness involving proximal and distal muscles ensues. Electromyographic studies characteristically show dysfunction at the level of multiple peripheral nerves, the lumbosacral plexus, and multiple nerve roots. MRI studies of the lumbosacral plexus are typically normal in this disorder but are most helpful in excluding an infiltrative neoplastic process, which can present similarly. Cauda equina syndrome due to a compressive, infiltrating, or inflammatory process affecting multiple lumbosacral nerve roots should be considered in this patient. However, the absence of abnormal signs or symptoms in the right leg would be unusual in this syndrome. Patients with cauda equina syndrome typically have bilateral (but sometimes asymmetric) leg pain, weakness, and areflexia. Bowel and bladder impairment can occur, and saddle anesthesia is expected in affected patients. Myelopathy due to demyelinating disease or other disorders should be considered in patients with lower limb weakness, sensory loss, and pain but is unlikely in this patient. Absent reflexes in the left leg suggest a lower motoneuron problem affecting either multiple nerve roots or the lumbosacral plexus in that limb. A myelopathy would be expected to cause hyperreflexia, spasticity, and an extensor plantar response in the lower limbs. When there is a history of pelvic irradiation, radiation-induced lumbosacral plexopathy should be considered in patients with progressive leg weakness. However, the rapid progression of symptoms and the presence of pain in this patient are not typical of radiation-induced lumbosacral plexopathy. Moreover, the mean time to symptom onset after radiation exposure is 3 to 6 years, although it has been reported to range from 1 month to 18 years after radiation exposure. Radiation-induced damage can affect the brachial plexus (typically, in breast cancer) or lumbosacral plexus. The frequency of brachial plexus injury after radiation therapy has been reported to range from 1.8% to 9.0%. Typical symptoms include gradually progressive weakness and sensory loss.
question
Neuro 90 A 69-year-old woman is evaluated in the emergency department for the sudden onset of headache, nausea, vomiting, and imbalance. The patient has hypertension for which she takes lisinopril. She has no other relevant personal or family medical history. On physical examination, blood pressure is 160/90 mm Hg, pulse rate is 80/min, and respiration rate is 18/min. Her Glasgow Coma Scale score is 12 (of a possible 15), which indicates moderate brain injury. Right finger-to-nose and heel-to-shin testing reveals dysmetria. Results of laboratory studies, including a complete blood count, a metabolic profile, and coagulation tests, are normal. A CT scan of the head shows an acute right cerebellar hemorrhage (>4 cm in diameter), perihematoma edema, compression of the right pons, and effacement but no compression of the fourth ventricle. Which of the following is the most appropriate next step in management? A Hematoma evacuation B Labetalol, intravenously C Recombinant factor VII, intravenously D Ventriculostomy
answer
A Emergent surgical evacuation of the hematoma is indicated for patients with a cerebellar hemorrhage greater than 3 cm in diameter who are deteriorating neurologically or who have brain stem compression and/or hydrocephalus from ventricular obstruction. This patient has signs of neurologic deterioration, given her score on the Glasgow Coma Scale. She has an acute cerebellar hemispheric hemorrhage that is greater than 3 cm in diameter and thus should be considered for neurosurgical intervention. The priority in such cases is to decompress the posterior fossa by surgically evacuating the hematoma. Patients with posterior fossa hematomas are at risk for life-threatening complications, including herniation and hydrocephalus, without such intervention. Blood pressure management in intracranial hemorrhage remains controversial. No trial has demonstrated that blood pressure control in this setting affects outcome, and there is concern about reducing cerebral perfusion pressure in patients with elevated intracranial pressure. The American Heart Association guidelines recommend that mean arterial blood pressure be kept between 70 mm Hg and 130 mm Hg. This patient's mean arterial blood pressure is 113 mm Hg, and thus intravenous administration of labetalol is not indicated. More importantly, urgent evacuation of the hematoma is likely to be life-saving and thus is the treatment of choice. Recombinant factor VII was promising as an experimental therapy for acute intracerebral hemorrhages, but the definitive phase 3 trial did not yield efficacious results. It is not approved by the U.S. Food and Drug Administration for treatment of such hemorrhages. A ventriculostomy may be indicated in a patient with a cerebellar hemorrhage who develops hydrocephalus. Because this patient's CT scan does not yet demonstrate that complication, ventriculostomy is inappropriate as the next step in management.
Get an explanation on any task
Get unstuck with the help of our AI assistant in seconds
New